You are on page 1of 246

Chapter 11

Service Department and Joint Cost Allocation


 

True / False Questions


 

1. The human resource department in a manufacturing company would be considered a service department. 
 
True    False
 
2. One reason to allocate service department costs to user departments is to encourage the user departments to
monitor their use of the service department costs. 
 
True    False
 
3. The direct method makes no cost allocations between or among service departments. 
 
True    False
 
4. The selection of an allocation base in the direct method is easier than the selection of an allocation base in
the step method. 
 
True    False
 
5. The step method allocates some, but not all, service department costs to other service departments. 
 
True    False
 
6. One advantage of the step method is that all reciprocal services are recognized between service
departments. 
 
True    False
 
7. With the reciprocal method, the total service department costs less the direct costs of the service department
equals the cost allocated to the service department. 
 
True    False
 
8. One potential disadvantage of the reciprocal method is it could overstate the cost of running the
organization's service departments. 
 
True    False
 

11-1
Copyright © 2017 McGraw-Hill Education. All rights reserved. No reproduction or distribution without the prior written consent of
McGraw-Hill Education.
9. In deciding whether to outsource a service department or not, the cost of the service department should be
estimated using the step method of allocation. 
 
True    False
 
10. Joint products are outputs from common inputs and a common production process. 
 
True    False
 
11. Joint costs are processing costs incurred after the split-off point in a common production process. 
 
True    False
 
12. The estimated net realizable value for a product is its estimated selling price after processing the product
beyond the split-off point. 
 
True    False
 
13. In general, it is better to use a product's market value at the split-off point than its estimated net realizable
value in allocating joint costs. 
 
True    False
 
14. The estimated net realizable value at the split-off point is calculated by taking the sales value after further
processing and deducting the additional processing costs. 
 
True    False
 
15. If a company's two joint products can be sold at the split-off point, there is no reason for allocating the joint
costs to the products. 
 
True    False
 
16. The physical quantities method of allocating joint costs is often used when the output sales prices are
highly volatile. 
 
True    False
 
17. The physical quantities method allocates joint costs so that each joint product has the same gross margin as
a percentage of sales. 
 
True    False
 
18. In a sell-or-process-further decision, the additional costs incurred after the split-off point are irrelevant. 
 
True    False
 
19. In a sell-or-process-further decision, the common costs incurred prior to the spilt-off point are irrelevant. 
 
True    False
 

11-2
Copyright © 2017 McGraw-Hill Education. All rights reserved. No reproduction or distribution without the prior written consent of
McGraw-Hill Education.
20. Since by-products have minor sales value, alternative methods of accounting for them will not have a
material effect on the financial statements. 
 
True    False
 
 

Multiple Choice Questions


 

21. Allocation of factory service department costs to the production departments is necessary to: 
 

A. measure use of plant capacity.


B. make sure that machines are operating efficiently.
C.  calculate cost per unit for purposes of external financial reporting.
D. control costs.
 
22. Which of the following statements is false regarding the use of multiple cost pools? 
 

A. Using more pools generally results in better cost allocations.


B. Designing and maintaining a system with multiple pools can be complex and costly.
C.  The use of Activity-Based-Accounting will almost always require multiple cost pools.
D. All support service costs are generally allocated using a single cost pool.
 
23. Which of the following statements is true regarding the use of multiple cost-pools? 
 

A. Organizations might implement multiple cost pools when the uses of the service resources have both
facility-level and unit-level components.
B. Designing and maintaining multiple cost pools is a relatively simple undertaking.
C.  The benefits of using multiple cost pools always outweigh the costs of setting them up.
D. Both resources supplied and resources used are allocated with multiple cost pools.
 
24. Which of the following would be an appropriate cost-allocation base for allocating the cost of the company
cafeteria? 
 

A. Square footage occupied by departments.


B. Number of hours of use.
C.  Number of meals served.
D. Salaries of personnel purchasing meals.
 

11-3
Copyright © 2017 McGraw-Hill Education. All rights reserved. No reproduction or distribution without the prior written consent of
McGraw-Hill Education.
25. Which of the following is the least practical reason for allocating service department costs to user
departments? 
 

A. To ascertain profitability of user departments.


B. To evaluate performance of managers and divisions.
C.  To make user departments aware that services are costly.
D. To provide the best possible service to users.
 
26. Service department costs are: 
 

A. generally treated as period costs rather than product costs.


B. reported as selling and administrative expenses on the income statement.
C.  eventually applied by the user departments to the units produced.
D. seldom found in manufacturing organizations.
 
27. A management purpose for allocating joint costs of a processing center to the various products produced is
to:  
 

A. establish inventory values for unsold units.


B. record accurate cost of sales by-product line.
C.  compute total processing cost variances by-product.
D. report correct standard product costs for comparative analysis.
 
28. Which of the following service departments could logically use space occupied (square footage) to allocate
its costs to user departments?  
 

A. Material Handling.
B. Cafeteria.
C.  Custodial Services.
D. Cost Accounting.
 
29. Which of the following departments is not a service department in a typical manufacturing company? 
 

A. Assembly.
B. Accounting.
C.  Human resources.
D. Information processing.
 

11-4
Copyright © 2017 McGraw-Hill Education. All rights reserved. No reproduction or distribution without the prior written consent of
McGraw-Hill Education.
30. Criteria for selecting allocation bases for service department allocations should not include:  
 

A. direct, traceable benefits from the service.


B. the extent of facilities provided.
C.  the ease of making an allocation.
D. sales dollars generated during the period.
 
31. Dreamland University has 20 departments. Two of its best departments are the (1) College of Innovation
(COI) and (2) Testing Services. The College of Innovation (COI) attempts to teach students the difficult,
but useful, skill of innovation. Testing Services grades examinations for professors. How would these two
departments be classified?
 
College of Testing
 
Innovation Services
A. Service Service
B. User Service
C. User User
D. Service User
 
 

A. Option A
B. Option B
C.  Option C
D. Option D
 
32. Which of the following is not a reason to justify the allocation of support services? 
 

A. Tax reporting requirements.


B. Influencing behavior of employees.
C.  To trace costs to the activity that created the costs.
D. Cost based contracts.
 
33. Which of the following statements is(are) false regarding the direct method of allocating service
department costs?
(A) The selection of an allocation base in the direct method is easier than the selection of an allocation
base in the step method.
(B) Once an allocation is made from a service department using the direct method, no further allocations
are made back to that department.  
 

A. Only A is false.
B. Only B is false.
C.  Neither A nor B is false.
D. Both A and B are false.
 

11-5
Copyright © 2017 McGraw-Hill Education. All rights reserved. No reproduction or distribution without the prior written consent of
McGraw-Hill Education.
34. Jamison Company has two service departments and two producing departments. Square footage of space
occupied by each department follows:
 
Custodial services 1,000 ft
General administration 3,000 ft
Producing department A 8,000 ft
Producing department B  8,000 ft
  20,000 ft

The department costs of Custodial Services are allocated on a basis of square footage of space. If Custodial
Services costs are budgeted at $38,000, the amount of cost allocated to General Administration under the
direct method would be:  
 

A. $0.
B. $7,125.
C.  $6,000.
D. $5,700.
 
35. Veneer Company has two service departments and two producing departments. The number of employees
in each department is:
 
Personnel 10
Cafeteria 25
Producing Department A 265
Producing Department B 250
  550

The department costs of the Personnel Department are allocated on a basis of the number of employees. If
these costs are budgeted at $37,125 during a given period, the amount of cost allocated to Department B
under the direct method would be:  
 

A. $0.
B. $17,187.50.
C.  $16,875.00.
D. $18,021.84.
 
36. Which of the following is not a benefit of cost allocation? 
 

A. Instilling responsibility for all costs of the company in division managers.


B. Constructing performance measures that may be more meaningful than contribution margins.
C.  Relating indirect costs to contracts, jobs and products.
D. Additional bookkeeping costs incurred to provide cost allocation information.
 

11-6
Copyright © 2017 McGraw-Hill Education. All rights reserved. No reproduction or distribution without the prior written consent of
McGraw-Hill Education.
37. Tenet Engineering, Inc. operates two user divisions as separate cost objects. To determine the costs of each
division, the company allocates common costs to the divisions. During the past month, the following
common costs were incurred:
 
Computer services (85% fixed) $260,000
Building occupancy 600,000
Personnel costs  110,000
Total common costs $970,000

The following information is available concerning various activity measures and service usages by each of
the divisions:
 
Division
  Division B
A
Area occupied (square feet) 20,000 40,000
Payroll $380,000 $180,000
Computer time (hours) 200 220
Computer storage (megabytes) 4,050 -0-
Equipment value $200,000 $250,000
Operating profit (pre-allocations) $555,000 $495,000

If common computer service costs are allocated using computer time as the allocation basis, what is the
computer cost allocated to Division B?  
 

A. $136,190.
B. $137,647.
C.  $144,444.
D. $173,333.
 

11-7
Copyright © 2017 McGraw-Hill Education. All rights reserved. No reproduction or distribution without the prior written consent of
McGraw-Hill Education.
38. Tenet Engineering, Inc. operates two user divisions as separate cost objects. To determine the costs of each
division, the company allocates common costs to the divisions. During the past month, the following
common costs were incurred:
 
Computer services (85% fixed) $260,000
Building occupancy 600,000
Personnel costs  110,000
Total common costs $970,000

The following information is available concerning various activity measures and service usages by each of
the divisions:
 
Division
  Division B
A
Area occupied (square feet) 20,000 40,000
Payroll $380,000 $180,000
Computer time (hours) 200 220
Computer storage (megabytes) 4,050 -0-
Equipment value $200,000 $250,000
Operating profit (pre-allocations) $555,000 $495,000

Using the most appropriate allocation basis, what is the personnel cost allocated to Division A?  
 

A. $58,143.
B. $74,643.
C.  $76,463.
D. $110,000.
 

11-8
Copyright © 2017 McGraw-Hill Education. All rights reserved. No reproduction or distribution without the prior written consent of
McGraw-Hill Education.
39. Tenet Engineering, Inc. operates two user divisions as separate cost objects. To determine the costs of each
division, the company allocates common costs to the divisions. During the past month, the following
common costs were incurred:
 
Computer services (85% fixed) $260,000
Building occupancy 600,000
Personnel costs  110,000
Total common costs $970,000

The following information is available concerning various activity measures and service usages by each of
the divisions:
 
Division
  Division B
A
Area occupied (square feet) 20,000 40,000
Payroll $380,000 $180,000
Computer time (hours) 200 220
Computer storage (megabytes) 4,050 -0-
Equipment value $200,000 $250,000
Operating profit (pre-allocations) $555,000 $495,000

If all common costs are allocated using operating profit as the allocation basis, what is the total cost
allocated to Division B?  
 

A. $457,286.
B. $512,714.
C.  $555,000.
D. $1,087,576.
 
40. Which of the following methods provides no data for service departments to monitor each other's costs? 
 

A. Direct method.
B. Reciprocal method.
C.  Step method.
D. All three methods, Direct, Reciprocal, and Step, provide data for monitoring costs.
 

11-9
Copyright © 2017 McGraw-Hill Education. All rights reserved. No reproduction or distribution without the prior written consent of
McGraw-Hill Education.
41. The Maryville Construction Company occupies 85,000 square feet for construction of mobile homes. There
are two manufacturing departments, finishing and assembly, and four service departments labeled S1, S2,
S3, and S4. Information relevant to Maryville is as follows:
 
Allocatio
Area n
Dept S1 S2 S3 S4 Assembly
used Finishing
S1 17,000 --- .10 .20 --- .20 .50
S2 4,250 --- --- .30 .30 --- .40
S3 8,500 .20 .20 --- .30 .20 .10
S4 4,250 .30 .10 .30 --- .20 .10
Fin. 21,250 --- --- --- --- --- ---
Asm. 29,750 --- --- --- --- --- ---

Rent paid for the area used is $720,000.

How much rent is allocable to the assembly department using the direct method of allocation?  
 

A. $420,000.
B. $332,500.
C.  $300,000.
D. $252,000.
 

11-10
Copyright © 2017 McGraw-Hill Education. All rights reserved. No reproduction or distribution without the prior written consent of
McGraw-Hill Education.
42. The Maryville Construction Company occupies 85,000 square feet for construction of mobile homes. There
are two manufacturing departments, finishing and assembly, and four service departments labeled S1, S2,
S3, and S4. Information relevant to Maryville is as follows:
 
Allocatio
Area n
Dept S1 S2 S3 S4 Assembly
used Finishing
S1 17,000 --- .10 .20 --- .20 .50
S2 4,250 --- --- .30 .30 --- .40
S3 8,500 .20 .20 --- .30 .20 .10
S4 4,250 .30 .10 .30 --- .20 .10
Fin. 21,250 --- --- --- --- --- ---
Asm. 29,750 --- --- --- --- --- ---

Rent paid for the area used is $720,000.

How much rent would be charged to S4 using the step method of allocation and a S3-S4-S1-S2 sequence
for the allocations?  
 

A. $36,000.
B. $40,000.
C.  $54,000.
D. $90,000.
 
43. If two service departments service the same number of departments, which service department's costs
should be allocated first when using the step method? 
 

A. The service department that provides the most service to other service departments.
B. The service department that provides the most service to the user departments.
C.  The service department with the least cost.
D. The service department that provides the least service to other service departments.
 
44. Which of the following is a weakness of the step method of service cost allocations?  
 

A. Computations are more complex than the reciprocal method.


B. All interdepartmental services are ignored.
C.  All intradepartmental services are ignored.
D. The order of service department allocation has to be determined.
 

11-11
Copyright © 2017 McGraw-Hill Education. All rights reserved. No reproduction or distribution without the prior written consent of
McGraw-Hill Education.
45. The Hsu Manufacturing Company has two service departments: Maintenance and Accounting. The
Maintenance Department's costs of $300,000 are allocated on the basis of machine hours. The Accounting
Department's costs of $120,000 are allocated on the basis of the number of employees within a specific
department. The direct departmental costs for A and B are $300,000 and $500,000, respectively.
 
  Maint Acctg A B
Machine hours 480 20 2,300 200
Number of employees 2 2 8 4

What is the Maintenance Department's cost allocated to Department A using the direct method?  
 

A. $92,000.
B. $230,000.
C.  $276,000.
D. $386,400.
 
46. The Hsu Manufacturing Company has two service departments: Maintenance and Accounting. The
Maintenance Department's costs of $300,000 are allocated on the basis of machine hours. The Accounting
Department's costs of $120,000 are allocated on the basis of the number of employees within a specific
department. The direct departmental costs for A and B are $300,000 and $500,000, respectively.
 
  Maint Acctg A B
Machine hours 480 20 2,300 200
Number of employees 2 2 8 4

What is the Accounting Department's cost allocated to Department B using the direct method?  
 

A. $40,000.
B. $80,000.
C.  $20,000.
D. $10,000.
 

11-12
Copyright © 2017 McGraw-Hill Education. All rights reserved. No reproduction or distribution without the prior written consent of
McGraw-Hill Education.
47. The Hsu Manufacturing Company has two service departments: Maintenance and Accounting. The
Maintenance Department's costs of $300,000 are allocated on the basis of machine hours. The Accounting
Department's costs of $120,000 are allocated on the basis of the number of employees within a specific
department. The direct departmental costs for A and B are $300,000 and $500,000, respectively.
 
  Maint Acctg A B
Machine hours 480 20 2,300 200
Number of employees 2 2 8 4

What is the Maintenance Department's cost allocated to Department B using the step method and assuming
the Maintenance Department's costs are allocated first?  
 

A. $276,000.
B. $230,000.
C.  $322,000.
D. $23,810.
 
48. The Hsu Manufacturing Company has two service departments: Maintenance and Accounting. The
Maintenance Department's costs of $300,000 are allocated on the basis of machine hours. The Accounting
Department's costs of $120,000 are allocated on the basis of the number of employees within a specific
department. The direct departmental costs for A and B are $300,000 and $500,000, respectively.
 
  Maint Acctg A B
Machine hours 480 20 2,300 200
Number of employees 2 2 8 4

What is the cost of the Accounting Department's cost allocated to Department A using the step method and
assuming the Maintenance Department's costs are allocated first?  
 

A. $81,333.
B. $81,587.
C.  $80,000.
D. $68,571.
 

11-13
Copyright © 2017 McGraw-Hill Education. All rights reserved. No reproduction or distribution without the prior written consent of
McGraw-Hill Education.
49. Steven Parker owns and operates Steven's Septic Service and Legal Advice. Steven's two revenue
generating (production) operations are supported by two service departments: Clerical and Janitorial. Costs
in the service departments are allocated in the following order using the designated allocation bases:

Clerical:

Variable cost: expected number of work orders processed


Fixed cost: long-run average number of work orders processed

Janitorial:

Variable cost: labor hours


Fixed cost: square footage of space occupied
Average and expected activity levels for next month (June) are as follows:
 
No. of Work
Labor Square
  Orders
Average Hours Footage
Expected
Septic
50 80 560 1,800
Service
Legal advice 25 20 840 2,200
Clerical 20 20 400 1,600
Janitorial 5 20 200 1,000

Expected costs in the service departments for June are as follows:


 
  Clerical Janitorial
Variable costs $12,000 $4,200
Fixed costs $8,400 $800

Under the step method of allocation, how much Clerical service cost should be allocated to the Septic
Service operation for June? (Assume Clerical costs are allocated before Janitorial costs and round all
calculations to the nearest whole dollar.)  
 

A. $12,689.
B. $13,100.
C.  $13,620.
D. $15,596.
 

11-14
Copyright © 2017 McGraw-Hill Education. All rights reserved. No reproduction or distribution without the prior written consent of
McGraw-Hill Education.
50. Steven Parker owns and operates Steven's Septic Service and Legal Advice. Steven's two revenue
generating (production) operations are supported by two service departments: Clerical and Janitorial. Costs
in the service departments are allocated in the following order using the designated allocation bases:

Clerical:

Variable cost: expected number of work orders processed


Fixed cost: long-run average number of work orders processed

Janitorial:

Variable cost: labor hours


Fixed cost: square footage of space occupied
Average and expected activity levels for next month (June) are as follows:
 
No. of Work
Labor Square
  Orders
Average Hours Footage
Expected
Septic
50 80 560 1,800
Service
Legal advice 25 20 840 2,200
Clerical 20 20 400 1,600
Janitorial 5 20 200 1,000

Expected costs in the service departments for June are as follows:


 
  Clerical Janitorial
Variable costs $12,000 $4,200
Fixed costs $8,400 $800

Under the direct method of allocation, what is the total amount of service cost allocated to the Legal Advice
operation for June? (Round all calculations to the nearest whole dollar.)  
 

A. $6,231.
B. $7,720.
C.  $8,640.
D. $9,330.
 

11-15
Copyright © 2017 McGraw-Hill Education. All rights reserved. No reproduction or distribution without the prior written consent of
McGraw-Hill Education.
51. Cordner Corporation has two production Departments: P1 and P2 and two service departments: S1 and S2.
Direct costs for each department and the proportion of service costs used by the various departments for the
month of July are as follows:
 
Proportion of Services Used
   
by:
Direct
Department S1 S2 P1 P2
costs
S1 $60,000   0.70 0.10 0.20
S2 $100,000 0.20   0.30 0.50
P1 $160,000        
P2 $140,000        

Under the direct-method of cost allocation, the amount of S1 costs allocated to the S2 would be:  
 

A. $42,000.
B. $20,000.
C.  $0.
D. $6,000.
 
52. Cordner Corporation has two production Departments: P1 and P2 and two service departments: S1 and S2.
Direct costs for each department and the proportion of service costs used by the various departments for the
month of July are as follows:
 
Proportion of Services Used
   
by:
Direct
Department S1 S2 P1 P2
costs
S1 $60,000   0.70 0.10 0.20
S2 $100,000 0.20   0.30 0.50
P1 $160,000        
P2 $140,000        

Under the direct-method of cost allocation, the amount of S1 costs allocated to P1 would be:  
 

A. $20,000.
B. $6,000.
C.  $30,000.
D. $62,500.
 

11-16
Copyright © 2017 McGraw-Hill Education. All rights reserved. No reproduction or distribution without the prior written consent of
McGraw-Hill Education.
53. Cordner Corporation has two production Departments: P1 and P2 and two service departments: S1 and S2.
Direct costs for each department and the proportion of service costs used by the various departments for the
month of July are as follows:
 
Proportion of Services Used
   
by:
Direct
Department S1 S2 P1 P2
costs
S1 $60,000   0.70 0.10 0.20
S2 $100,000 0.20   0.30 0.50
P1 $160,000        
P2 $140,000        

Under the step-method of cost allocation, the amount of S2 costs allocated to S1 would be:  
 

A. $40,000.
B. $20,000.
C.  $0.
D. $42,000.
 
54. Cordner Corporation has two production Departments: P1 and P2 and two service departments: S1 and S2.
Direct costs for each department and the proportion of service costs used by the various departments for the
month of July are as follows:
 
Proportion of Services Used
   
by:
Direct
Department S1 S2 P1 P2
costs
S1 $60,000   0.70 0.10 0.20
S2 $100,000 0.20   0.30 0.50
P1 $160,000        
P2 $140,000        

Under the step-method of cost allocation, the amount of costs allocated from S2 to P2 would be:  
 

A. $88,750.
B. $50,000.
C.  $62,500.
D. $53,250.
 

11-17
Copyright © 2017 McGraw-Hill Education. All rights reserved. No reproduction or distribution without the prior written consent of
McGraw-Hill Education.
55. Cordner Corporation has two production Departments: P1 and P2 and two service departments: S1 and S2.
Direct costs for each department and the proportion of service costs used by the various departments for the
month of July are as follows:
 
Proportion of Services Used
   
by:
Direct
Department S1 S2 P1 P2
costs
S1 $60,000   0.70 0.10 0.20
S2 $100,000 0.20   0.30 0.50
P1 $160,000        
P2 $140,000        

Under the step-method of allocation, the total amount of service costs allocated to producing departments
would be:  
 

A. $118,000.
B. $160,000.
C.  $140,000.
D. $40,000.
 
56. Palace Company has two service departments and two user departments. The number of employees in each
department is:
 
Personnel 10
Cafeteria 25
Producing Department A 265
Producing Department B 250
  550

The fixed costs of the Personnel Department are allocated on a basis of the number of employees. If these
costs are budgeted at $37,125 during a given period, the amount of cost allocated to the Cafeteria under the
step method would be:  
 

A. $0.
B. $1,718.75.
C.  $1,687.50.
D. $1,802.18.
 

11-18
Copyright © 2017 McGraw-Hill Education. All rights reserved. No reproduction or distribution without the prior written consent of
McGraw-Hill Education.
57. There are several methods for allocating service department costs to production departments. The method
which recognizes service provided by one service department to another but does not recognize reciprocal
interdepartmental service is called: (CMA adapted)  
 

A. direct method.
B. variable method.
C.  linear method.
D. step-down method.
 
58. Because this allocation method recognizes that service departments often provide each other with inter-
departmental service, it is theoretically considered to be the most accurate method for allocating service
department costs to production departments. This method is: (CMA adapted) 
 

A. direct method.
B. variable method.
C.  linear method.
D. reciprocal method.
 
59. The following information relates to Osceola Corporation for the past accounting period.
 
  Direct costs
Service Dept A $80,000
Service Dept B 60,000
Producing Dept C 15,000
Producing Dept D 20,000

Proportion of service by A Proportion of service by B


to: to:
B 10% A 30%
C 60% C 20%
D 30% D 50%

Using the simultaneous solution method, Department A's cost allocated to Department C is:  
 

A. $48,000.
B. $58,800.
C.  $60,619.
D. $98,000.
 

11-19
Copyright © 2017 McGraw-Hill Education. All rights reserved. No reproduction or distribution without the prior written consent of
McGraw-Hill Education.
60. The following information relates to Osceola Corporation for the past accounting period.
 
  Direct costs
Service Dept A $80,000
Service Dept B 60,000
Producing Dept C 15,000
Producing Dept D 20,000

Proportion of service by A Proportion of service by B


to: to:
B 10% A 30%
C 60% C 20%
D 30% D 50%

Using the simultaneous solution method, Department B's cost allocated to Department C is:  
 

A. $29,021
B. $14,021
C.  $13,192
D. $7,794
 

11-20
Copyright © 2017 McGraw-Hill Education. All rights reserved. No reproduction or distribution without the prior written consent of
McGraw-Hill Education.
61. The following set up is a system of simultaneous linear equations to allocate costs using the reciprocal
method. Matrix algebra is not required.
The following costs were incurred in three operating departments and three service departments in
Westmoreland Company.
 
Department Direct Costs Label
Subassemblies $550,000 P1
Final assembly 775,000 P2
Marketing 285,000 P3
Building occupancy 85,000 S1
Research&development 120,000 S2
Supervision 45,000 S3

Use of services by other departments is as follows.


 
                       User Department

Service Cost Sub- Building


Final Assembly Marketin R&
Center assemblies Occupancy Supervision
g D
Bldg.occupancy .30 .25 .20 --- .15 .10
R&D .50 .50 --- --- --- ---
Supervision .20 .30 .20 .10 .20 ---

The equation for department P1 (subassemblies) is:  


 

A. P1 = $550,000 + .25P2 + .20P3 + .15S2 + 10S3.


B. P1 = $550,000 + .30S1 + .50S2 + .20S3.
C.  P1 = .30S1 + .50S2 + 20S3.
D. P1 = .30S1 + .50S.
 

11-21
Copyright © 2017 McGraw-Hill Education. All rights reserved. No reproduction or distribution without the prior written consent of
McGraw-Hill Education.
62. The following set up is a system of simultaneous linear equations to allocate costs using the reciprocal
method. Matrix algebra is not required.
The following costs were incurred in three operating departments and three service departments in
Westmoreland Company.
 
Department Direct Costs Label
Subassemblies $550,000 P1
Final assembly 775,000 P2
Marketing 285,000 P3
Building occupancy 85,000 S1
Research&development 120,000 S2
Supervision 45,000 S3

Use of services by other departments is as follows.


 
                       User Department

Service Cost Sub- Building


Final Assembly Marketin R&
Center assemblies Occupancy Supervision
g D
Bldg.occupancy .30 .25 .20 --- .15 .10
R&D .50 .50 --- --- --- ---
Supervision .20 .30 .20 .10 .20 ---

The equation for department P2 (final assembly) is:  


 

A. P2 = .25S1 + .50S2 + .30S3.


B. P2 = $775,000 + .25P2 + .20P3 + .15S2 + .10S3.
C.  P2 = $775,000 + .30S1 + .50S2 + .20S3.
D. P2 = $775,000 + .25S1 + .50S2 + .30S3.
 

11-22
Copyright © 2017 McGraw-Hill Education. All rights reserved. No reproduction or distribution without the prior written consent of
McGraw-Hill Education.
63. The following set up is a system of simultaneous linear equations to allocate costs using the reciprocal
method. Matrix algebra is not required.
The following costs were incurred in three operating departments and three service departments in
Westmoreland Company.
 
Department Direct Costs Label
Subassemblies $550,000 P1
Final assembly 775,000 P2
Marketing 285,000 P3
Building occupancy 85,000 S1
Research&development 120,000 S2
Supervision 45,000 S3

Use of services by other departments is as follows.


 
                       User Department

Service Cost Sub- Building


Final Assembly Marketin R&
Center assemblies Occupancy Supervision
g D
Bldg.occupancy .30 .25 .20 --- .15 .10
R&D .50 .50 --- --- --- ---
Supervision .20 .30 .20 .10 .20 ---

The equation for department P3 (marketing) is:  


 

A. P3 = $285,000 + .20S1 + .20S3.


B. P3 = $285,000 + .20S1 + .60S2 + .20S3.
C.  P3 = $285,000 + .20S1 + .20S2 + .60S3.
D. P3 = $285,000 + .50S1 + .50S3.
 

11-23
Copyright © 2017 McGraw-Hill Education. All rights reserved. No reproduction or distribution without the prior written consent of
McGraw-Hill Education.
64. The following set up is a system of simultaneous linear equations to allocate costs using the reciprocal
method. Matrix algebra is not required.
The following costs were incurred in three operating departments and three service departments in
Westmoreland Company.
 
Department Direct Costs Label
Subassemblies $550,000 P1
Final assembly 775,000 P2
Marketing 285,000 P3
Building occupancy 85,000 S1
Research&development 120,000 S2
Supervision 45,000 S3

Use of services by other departments is as follows.


 
                       User Department

Service Cost Sub- Building


Final Assembly Marketin R&
Center assemblies Occupancy Supervision
g D
Bldg.occupancy .30 .25 .20 --- .15 .10
R&D .50 .50 --- --- --- ---
Supervision .20 .30 .20 .10 .20 ---

The equation for department S1 (building occupancy) is:  


 

A. S1 = .10S3.
B. S1 = $85,000 + 1.00S3.
C.  S1 = $85,000 + .10S3.
D. S1 = $85,000 + .90S2 + .10S3.
 

11-24
Copyright © 2017 McGraw-Hill Education. All rights reserved. No reproduction or distribution without the prior written consent of
McGraw-Hill Education.
65. The following set up is a system of simultaneous linear equations to allocate costs using the reciprocal
method. Matrix algebra is not required.
The following costs were incurred in three operating departments and three service departments in
Westmoreland Company.
 
Department Direct Costs Label
Subassemblies $550,000 P1
Final assembly 775,000 P2
Marketing 285,000 P3
Building occupancy 85,000 S1
Research&development 120,000 S2
Supervision 45,000 S3

Use of services by other departments is as follows.


 
                       User Department

Service Cost Sub- Building


Final Assembly Marketin R&
Center assemblies Occupancy Supervision
g D
Bldg.occupancy .30 .25 .20 --- .15 .10
R&D .50 .50 --- --- --- ---
Supervision .20 .30 .20 .10 .20 ---

The equation for department S2 (research and development) is:  


 

A. S2 = $120,000 + .15S1 + .65S2 + .20S3.


B. S2 = .15S1 + 20S3.
C.  S2 = $120,000 + .15S1 + .20S3.
D. S2 = $120,000 + .40S1 + .60S3.
 

11-25
Copyright © 2017 McGraw-Hill Education. All rights reserved. No reproduction or distribution without the prior written consent of
McGraw-Hill Education.
66. The following set up is a system of simultaneous linear equations to allocate costs using the reciprocal
method. Matrix algebra is not required.
The following costs were incurred in three operating departments and three service departments in
Westmoreland Company.
 
Department Direct Costs Label
Subassemblies $550,000 P1
Final assembly 775,000 P2
Marketing 285,000 P3
Building occupancy 85,000 S1
Research&development 120,000 S2
Supervision 45,000 S3

Use of services by other departments is as follows.


 
                       User Department

Service Cost Sub- Building


Final Assembly Marketin R&
Center assemblies Occupancy Supervision
g D
Bldg.occupancy .30 .25 .20 --- .15 .10
R&D .50 .50 --- --- --- ---
Supervision .20 .30 .20 .10 .20 ---

The equation for department S3 (supervision) is:  


 

A. S3 = $45,000 + .90S1 + .10S2.


B. S3 = $45,000 + .10S1.
C.  S3 = $45,000 + 1.00S1.
D. S3 = .10S1.
 

11-26
Copyright © 2017 McGraw-Hill Education. All rights reserved. No reproduction or distribution without the prior written consent of
McGraw-Hill Education.
67. Advanced Computer Solutions, Inc. has two main services: (1) time on a timeshared computer system, and
(2) proprietary computer programs. Computer time is provided by the operation department (Op) and
programs are written by the programming department (P).

The percentage of each service used by each department for a typical period is:

  Supplied
User Op P
Op --- 40%
P 30% ---
Sold to customers 70% 60%

In a typical period, the operation department (Op) spends $4,500 and the programming department (P)
spends $2,500.

Under the step method (Op first), what is the cost of the computer time and the computer programs for
sale?
 
  Time Programs
A. $4,500 $2,500
B. $3,150 $3,850
C. $1,350 $5,650
D. $2,700 $4,300
 
 

A. Option A
B. Option B
C.  Option C
D. Option D
 

11-27
Copyright © 2017 McGraw-Hill Education. All rights reserved. No reproduction or distribution without the prior written consent of
McGraw-Hill Education.
68. Advanced Computer Solutions, Inc. has two main services: (1) time on a timeshared computer system, and
(2) proprietary computer programs. Computer time is provided by the operation department (Op) and
programs are written by the programming department (P).

The percentage of each service used by each department for a typical period is:

  Supplied
User Op P
Op --- 40%
P 30% ---
Sold to customers 70% 60%

In a typical period, the operation department (Op) spends $4,500 and the programming department (P)
spends $2,500.

Under the reciprocal method what is the algebraic solution to the cost allocation problem?  
 

A. Op = 4,500 + .40 P; P = 2,500 + .30 Op.


B. Op = 4,500 + .70 P; P = 2,500 + .60 Op.
C.  Op = 2,500 + .40 P; P = 4,500 + .30 Op.
D. Op = 2,500 + .70 P; P = 4,500 + .60 Op.
 
69. Which of the following best describes the objective of joint cost allocation?  
 

A. Inventory valuation.
B. Pricing goods for sale.
C.  Making decisions about levels of production.
D. Making decisions about raw materials requirements.
 
70. Allocated joint costs are useful for:  
 

A. setting the selling price of a product.


B. determining whether to continue producing an item.
C.  controlling user department costs.
D. determining inventory cost for accounting purposes.
 
71. For the purposes of allocating joint costs to joint products, the sales price at the point of sale, reduced by
the cost to complete after split-off, is assumed to be equal to the: (CPA adapted) 
 

A. total costs.
B. joint costs.
C.  sales price less a normal profit margin at point of sale.
D. relative sales value at split-off.
 

11-28
Copyright © 2017 McGraw-Hill Education. All rights reserved. No reproduction or distribution without the prior written consent of
McGraw-Hill Education.
72. Which of the following cost items is not allocable as joint costs when a single manufacturing process
produces several main products and several by-products? 
 

A. Direct materials.
B. Variable overhead.
C.  Direct labor.
D. Freight-out.
 
73. Which of the following is not a step needed to maximize the profits from joint products? 
 

A. Forecasting the sales price of each final product.


B. Identifying alternative sets and quantities of final products possible from the joint process.
C.  Determining how to allocate joint costs to the final products.
D. Estimating the costs required to further process joint products into salable products.
 
74. Which of the following statements about maximizing the profit of joint product processes is true? 
 

A. Joint processing costs incurred prior to split-off should be allocated before making those decisions.
B. Only costs caused by management decisions to choose one or another set of products after split-off are
relevant.
C.  Only revenues from selling or processing beyond the split-off point and additional expenditures for
further processing are relevant.
D. Both B and C are true.
 
75. For purposes of allocating joint costs to joint products, the estimated net realizable value at split-off is
equal to: 
 

A. final sales price reduced by cost to complete after split-off.


B. sales price less a normal profit margin at the point of sale.
C.  separable product cost plus a normal profit margin.
D. total sales value less joint costs at point of split-off.
 
76. The method of accounting for joint product costs that will produce the same gross margin percentage for all
products is the: 
 

A. replacement method.
B. physical quantities method.
C.  net realizable value method.
D. units produced method.
 

11-29
Copyright © 2017 McGraw-Hill Education. All rights reserved. No reproduction or distribution without the prior written consent of
McGraw-Hill Education.
77. Which of the following statements is false? 
 

A. The estimated net realizable value for a product is its estimated selling price after processing the product
beyond the split-off point.
B. In general, it is better to use a product's market value at the split-off point than its estimated net
realizable value.
C.  The estimated net realizable value at the split-off point is calculated by taking the sales value after
further processing and deducting the additional processing costs.
D. It is better to use the net realizable value method for allocating joint costs than the estimated net
realizable value method.
 
78. Net realizable value at the split-off point is used to allocate:
 
  Costs After Split-off Point Incurred Joint Costs
A. No No
B. No Yes
C. Yes No
D. Yes Yes
 
 

A. Option A
B. Option B
C.  Option C
D. Option D
 
79. Products X, Y, and Z are produced from the same process at a cost of $5,200. Five thousand pounds of raw
material yields 1,500 X, 2,500 Y, and 1,000 Z. Selling prices are: X $2 per unit, Y $4 per unit, Z valueless.
The ending inventory of X is 50 units. What is the value of the ending inventory if joint costs are allocated
using net realizable value?  
 

A. $21.67.
B. $31.20.
C.  $40.00.
D. $42.00.
 

11-30
Copyright © 2017 McGraw-Hill Education. All rights reserved. No reproduction or distribution without the prior written consent of
McGraw-Hill Education.
80. Bonanza Co. manufactures products X and Y from a joint process that also yields a by-product, Z. Revenue
from sales of Z is treated as a reduction of joint costs. Additional information is as follows:
 
  PRODUCTS  
  X Y Z TOTAL
Units
20,000 20,000 10,000 50,000
produced
Joint costs ? ? ? 262,000
Sales value at
$300,000 $150,000 $10,000 $460,000
split-off

Joint costs were allocated using the net realizable value method at the split-off point. The joint costs
allocated to product X were  
 

A. $75,000.
B. $100,800.
C.  $150,000.
D. $168,000.
 
81. Great Falls Company makes two products, Wool Gloves and Wool Mittens. They are initially processed
from the same raw material and then, after split-off, further processed separately. Additional information is
as follows:
 
Mitten
  Gloves Total
s
Final Sales Price $9,000 $6,000 $15,000
Joint Costs Prior to Split-Off
? ? 6,600
Point
Costs Beyond Split-Off Point $3,000 $3,000 6,000

What are the joint costs allocated to Gloves and Mittens assuming Great Falls uses the estimated net
realizable value approach?
 
  Gloves Mittens
A. $3,300 $3,300
B $3,960 $2,640
C. $4,400 $2,200
D. $4,560 $2,040
 
 

A. Option A
B. Option B
C.  Option C
D. Option D
 

11-31
Copyright © 2017 McGraw-Hill Education. All rights reserved. No reproduction or distribution without the prior written consent of
McGraw-Hill Education.
82. Atkinson, Inc., manufactures products A, B, and C from a common process. Joint costs were $60,000.
Additional information is as follows:
 
    If Processed Further
Units Sales Value Sales Additional
Product
Produced at Split-Off Value Costs
A 6,000 $40,000 $55,000 $4,000
B 4,000 35,000 45,000 6,000
C  2,000     25,000   30,000    8,000
  12,000 $100,000 $130,000 $18,000

Assuming that joint production costs are allocated using the physical quantities method (units produced),
what were the costs allocated to Product A?  
 

A. $27,000.
B. $29,000.
C.  $33,000.
D. $30,000.
 
83. Atkinson, Inc., manufactures products A, B, and C from a common process. Joint costs were $60,000.
Additional information is as follows:
 
    If Processed Further
Units Sales Value Sales Additional
Product
Produced at Split-Off Value Costs
A 6,000 $40,000 $55,000 $4,000
B 4,000 35,000 45,000 6,000
C  2,000     25,000   30,000    8,000
  12,000 $100,000 $130,000 $18,000

Assuming that joint product costs are allocated using the net realizable value method, what were the total
costs assigned to Product B?  
 

A. $26,000.
B. $26,796.
C.  $27,000.
D. $28,286.
 

11-32
Copyright © 2017 McGraw-Hill Education. All rights reserved. No reproduction or distribution without the prior written consent of
McGraw-Hill Education.
84. Tanner Corporation produced 3,660 units, consisting of three separate products, in a joint process for the
year. The market for these products was so unstable that it was not practical to estimate the selling price of
the products. A cost of $425,000 was incurred in the joint process. Product X's production was 80% of
product Y's while product Z's production was 125% of product Y's. What is the amount of the joint cost
allocable to product X assuming Tanner uses the physical quantities method of allocation?  
 

A. $111,475.
B. $114,865.
C.  $139,344.
D. $141,667.
 
85. Cariboo Manufacturing Company incurred a joint cost of $600,000 in the production of X and Y in a joint
process. Presently, 1,800 of X and 1,400 of Y are being produced each month. Management plans to
decrease X's production by 300 units in order to increase the production of Y by 500 units. Additionally,
this change will require minor modifications, which will add $20,000 to the joint cost. This cost is entirely
attributable to product Y. What is the amount of the joint costs allocable to X and Y before changes to
existing production assuming Cariboo allocates their joint costs according to the proportion of Y and X
produced?
 
  Product X Product Y
A. $262,500 $337,500
B. $264,706 $335,294
C. $273,529 $326,471
D. $337,500 $262,500
 
 

A. Option A
B. Option B
C.  Option C
D. Option D
 

11-33
Copyright © 2017 McGraw-Hill Education. All rights reserved. No reproduction or distribution without the prior written consent of
McGraw-Hill Education.
86. Upton Company produces two main products and a by-product out of a joint process. The ratio of output
quantities to input quantities of direct material used in the joint process remains consistent from month to
month. Upton has employed the physical-volume method to allocate joint production costs to the two main
products. The net realizable value of the by-product is used to reduce the joint production costs before the
joint costs are allocated to the main products. Data regarding Upton's operations for the current month are
presented in the chart below. During the month, Upton incurred joint production costs of $2,520,000. The
main products are not marketable at the split-off point and, thus, have to be processed further.
 

First Second
  Main Main
By-
Product Product
product
Monthly output in
90,000 150,000 60,000
pounds
Selling Price per pound $30 $14 $2
Separable process costs $540,000 $660,000  

The amount of joint production cost that Upton would allocate to the Second Main Product by using the
physical quantities method to allocate joint production costs would be:  
 

A. $1,200,000.
B. $1,260,000.
C.  $1,500,000.
D. $1,575,000.
 
87. The Mallak Company produced three joint products at a joint cost of $100,000. Two of these products were
processed further. Production and sales were:
 
  Weight Sales Addt’l. Processing Costs
P 300,000 lbs. $245,000 $200,000
Q 100,000 lbs. 30,000 -0-
R 100,000 lbs. 175,000 100,000

If the estimated net realizable value method is used and product Q is accounted for as a main product, how
much of the joint costs would be allocated to product R?  
 

A. $38,889.
B. $41,667.
C.  $50,000.
D. $62,500.
 

11-34
Copyright © 2017 McGraw-Hill Education. All rights reserved. No reproduction or distribution without the prior written consent of
McGraw-Hill Education.
88. The Mallak Company produced three joint products at a joint cost of $100,000. Two of these products were
processed further. Production and sales were:
 
  Weight Sales Addt’l. Processing Costs
P 300,000 lbs. $245,000 $200,000
Q 100,000 lbs. 30,000 -0-
R 100,000 lbs. 175,000 100,000

Assume Q is a by-product and Mallak uses the cost reduction method of accounting for by-product cost. If
estimated net realizable value is used, how much of the joint costs would be allocated to product R?  
 

A. $38,889.
B. $43,750.
C.  $50,000.
D. $62,500.
 
89. The Mallak Company produced three joint products at a joint cost of $100,000. Two of these products were
processed further. Production and sales were:
 
  Weight Sales Addt’l. Processing Costs
P 300,000 lbs. $245,000 $200,000
Q 100,000 lbs. 30,000 -0-
R 100,000 lbs. 175,000 100,000

If joint costs are allocated based on relative weight of the outputs and all products are main products, how
much of the joint costs would be allocated to product P?  
 

A. $43,750.
B. $50,000.
C.  $60,000.
D. $62,500.
 

11-35
Copyright © 2017 McGraw-Hill Education. All rights reserved. No reproduction or distribution without the prior written consent of
McGraw-Hill Education.
90. The Mallak Company produced three joint products at a joint cost of $100,000. Two of these products were
processed further. Production and sales were:
 
  Weight Sales Addt’l. Processing Costs
P 300,000 lbs. $245,000 $200,000
Q 100,000 lbs. 30,000 -0-
R 100,000 lbs. 175,000 100,000

What is the net income of Mallak Company if the estimated net realizable value method of joint cost
allocation is used?  
 

A. $20,000.
B. $50,000.
C.  $150,000.
D. $350,000.
 
91. Cambridge Company manufactures three main products, L, M, and N, from a joint process. Additional
information for June production activity follows:
 
  L M N Total
Units
50,000 40,000 10,000 100,000
produced
Joint costs ? ? ? $450,000
Sales value at
$420,000 $270,000 $60,000 $750,000
split-off
Additional
costs if
$88,000 $30,000 $12,000 $130,000
process
further
Sale value if
processed $538,000 $320,000 $78,000 $936,000
further

Assuming that the 10,000 units of N were processed further and sold for $78,000, what was Anchorage's
gross profit from this sale? Assume the physical quantities method of allocation is used.  
 

A. $21,000.
B. $28,500.
C.  $30,000.
D. $66,000.
 

11-36
Copyright © 2017 McGraw-Hill Education. All rights reserved. No reproduction or distribution without the prior written consent of
McGraw-Hill Education.
92. Which of the following is not a physical measure that can be used for allocating joint costs using the
physical quantities method?  
 

A. Tons of steel.
B. Ounces of gold.
C.  Dollars of labor.
D. Feet of lumber.
 
93. In joint product costing and analysis, which one of the following costs is relevant when deciding the point
at which a product should be sold in order to maximize profits? (CMA adapted)  
 

A. separable costs after the split-off point.


B. joint costs to the spilt-off point.
C.  sales salaries for the period when the units were produced.
D. purchase costs of the materials required for the joint products.
 
94. Delite Confectionary Company produces various types of candies. Several candies could be sold at the
split-off point or processed further and sold in a different form after further processing. The candies are
produced in a joint processing operation with $500,000 of joint processing costs monthly, which are
allocated based on pounds produced. Information concerning this process for a recent month appears
below:
 

Price
Further Price after
Number per
processing processing
Candy of pound
costs further
type pounds at split-
off
Sweet Mats 50,000 $8 $75,000 $10.00
Chocolate
100,000 $10 $30,000 $10.50
Delight
Minty
25,000 $5 $20,000 $5.50
Wonders

Based on the information presented, which of the products should be processed further?  
 

A. Sweet Meats only.


B. Both Sweet Meets and Chocolate Delight.
C.  Minty Wonders only.
D. Both Sweet Meats and Minty Wonders.
 

11-37
Copyright © 2017 McGraw-Hill Education. All rights reserved. No reproduction or distribution without the prior written consent of
McGraw-Hill Education.
95. Delite Confectionary Company produces various types of candies. Several candies could be sold at the
split-off point or processed further and sold in a different form after further processing. The candies are
produced in a joint processing operation with $500,000 of joint processing costs monthly, which are
allocated based on pounds produced. Information concerning this process for a recent month appears
below:
 

Price
Further Price after
Number per
processing processing
Candy of pound
costs further
type pounds at split-
off
Sweet Mats 50,000 $8 $75,000 $10.00
Chocolate
100,000 $10 $30,000 $10.50
Delight
Minty
25,000 $5 $20,000 $5.50
Wonders

The net advantage (disadvantage) of processing Sweet Meats further is:  


 

A. a $25,000 disadvantage to process further.


B. a $32,143 advantage to process further.
C.  a $25,000 advantage to process further.
D. a $282,143 disadvantage to process further.
 

11-38
Copyright © 2017 McGraw-Hill Education. All rights reserved. No reproduction or distribution without the prior written consent of
McGraw-Hill Education.
96. Delite Confectionary Company produces various types of candies. Several candies could be sold at the
split-off point or processed further and sold in a different form after further processing. The candies are
produced in a joint processing operation with $500,000 of joint processing costs monthly, which are
allocated based on pounds produced. Information concerning this process for a recent month appears
below:
 

Price
Further Price after
Number per
processing processing
Candy of pound
costs further
type pounds at split-
off
Sweet Mats 50,000 $8 $75,000 $10.00
Chocolate
100,000 $10 $30,000 $10.50
Delight
Minty
25,000 $5 $20,000 $5.50
Wonders

The joint processing costs in this operation:  


 

A. should be allocated to products to determine whether they are sold at split-off or processed further.
B. should be ignored in determining whether to sell at split-off or process further.
C.  should be ignored in making all product decisions.
D. are never included in product cost, as they are misleading to all management decisions.
 

11-39
Copyright © 2017 McGraw-Hill Education. All rights reserved. No reproduction or distribution without the prior written consent of
McGraw-Hill Education.
97. Delite Confectionary Company produces various types of candies. Several candies could be sold at the
split-off point or processed further and sold in a different form after further processing. The candies are
produced in a joint processing operation with $500,000 of joint processing costs monthly, which are
allocated based on pounds produced. Information concerning this process for a recent month appears
below:
 

Price
Further Price after
Number per
processing processing
Candy of pound
costs further
type pounds at split-
off
Sweet Mats 50,000 $8 $75,000 $10.00
Chocolate
100,000 $10 $30,000 $10.50
Delight
Minty
25,000 $5 $20,000 $5.50
Wonders

If Chocolate Delight is processed further, the gross profit margin that will appear in a product line income
statement for Chocolate Delight would be:  
 

A. $734,286.
B. $520,000.
C.  $1,020,000.
D. $632,596.
 
98. The Foxmoor Company produces three products, X, Y, and Z from a single raw material input. Product Y
can be sold at the split-off point for total revenues of $50,000 or it can be processed further at a total cost of
$16,000 and then sold for $68,000. Product Y:  
 

A. should be sold at the split-off point, rather than processed further.


B. would increase the company's overall net income by $18,000 if processed further and then sold.
C.  would increase the company's overall net income by $68,000 if processed further and then sold.
D. would increase the company's overall net income by $2,000 if processed further and then sold.
 
99. Product C is one of several joint products that come out of Department M. The joint costs incurred in
Department M total $40,000. Product C can be sold at split-off or processed further and sold as a higher
quality item. The decision to process further should be based on the:  
 

A. assumption that the $40,000 is irrelevant.


B. allocation of the $40,000, using the net realizable value.
C.  allocation of the $40,000, using a physical measures approach.
D. allocation of the $40,000, using the relative sales value at split-off method.
 

11-40
Copyright © 2017 McGraw-Hill Education. All rights reserved. No reproduction or distribution without the prior written consent of
McGraw-Hill Education.
100. The characteristic that is most often used to distinguish a product as either a main product or a by-product
is the amount of: 
 

A. sales value of the products produced during the common production process.
B. direct manufacturing costs (e.g., materials) incurred before the split-off point.
C.  physical measures in the products produced during the common production process.
D. time (i.e., labor) required to produce the products from start to finish.
 
101. If by-product revenue is treated as other revenue instead of deducted from the net-realizable-value of the
main products: 
 

A. overall gross margin of the company will be higher.


B. overall gross margin of the company will be lower.
C.  the answer would depend on how joint product costs were allocated.
D. there is no difference in the overall gross margin of the company.
 
102. Products with a relatively minor sales value are called: 
 

A. scrap.
B. spoilage.
C.  by-products.
D. main products.
 
103. Joint products and by-products are produced simultaneously by a single process or series of processes and:

A. joint products are salable at the split-off point, but by-products are not.
B. by-products are salable at the split-off point, but joint products are not.
C.  the revenue from by-products may be recognized at the time of production.
D. all by-products must be allocated some portion of joint costs.
 
104. Which of the following statements regarding accounting for by-products is true?  
 

A. If all products are sold in the same period as they are produced, treating by-product revenue as other
revenue will result in a higher overall gross margin.
B. If all products are sold in the same period as they are produced, treating by-product net-realizable-value
as a deduction of the cost of the main products will result in a higher overall gross margin.
C.  If all products are sold in the same period as they are produced, total reported revenues will be the same
regardless of how by-product revenue is accounted for.
D. If all products are sold in the same period as they are produced, the reported gross margin will be the
same regardless of how by-product revenue is accounted for.
 
 

11-41
Copyright © 2017 McGraw-Hill Education. All rights reserved. No reproduction or distribution without the prior written consent of
McGraw-Hill Education.
Essay Questions
 

11-42
Copyright © 2017 McGraw-Hill Education. All rights reserved. No reproduction or distribution without the prior written consent of
McGraw-Hill Education.
105. For each of the support service costs listed below, name an appropriate cost allocation base:
 

(1.) Building rental cost                                           


(2.) Payroll department
                                          
salaries
(3.) Company cafeteria cost                                           
(4.) Human resources
                                          
department
(5.) Accounting department                                           
(6.) Computer equipment
                                          
depreciation
(7.) Insurance costs on
                                          
computer equipment
(8.) Depreciation on
                                          
company airplane
(9.) Factory manager                                           
(10.) Cost to clean company
                                          
uniforms
(11.) Costs of corporate
                                          
daycare facility
(12.) Equipment
                                          
maintenance
(13.) Cost of corporate
                                          
workout facility
(14.) Building insurance                                           
(15.) Cost of delivery truck
                                          
for a moving company
 
 

11-43
Copyright © 2017 McGraw-Hill Education. All rights reserved. No reproduction or distribution without the prior written consent of
McGraw-Hill Education.
106. Wimbledon Corporation has two production Departments: Assembly and Machining and two service
departments: Personnel and Cafeteria. Direct costs for each department and the proportion of service costs
used by the various departments for the month of July, 2016 are as follows:
 
    Proportion of Services Used by:
Departme Direct Personn Cafeter Machini Assemb
nt costs el ia ng ly
$30,00
Personnel   0.40 0.30 0.30
0
$50,00
Cafeteria 0.20   0.50 0.30
0
Machinin $80,00
       
g 0
$70,00
Assembly        
0

Required:

Compute the allocation of service department costs to producing departments for July, 2016 using the
direct-method.  
 

11-44
Copyright © 2017 McGraw-Hill Education. All rights reserved. No reproduction or distribution without the prior written consent of
McGraw-Hill Education.
107. Prestige Financial Credit Company produces two styles of credit reports: Individual and Corporate. The
difference between the two is the amount of background information and data collection required. The
Corporate report uses more skilled personnel because additional checking and data are required. The
relevant figures for the year just completed follow. Total support service costs to be allocated are
$3,200,000.
 
Allocation base Individual Corporate
Data purchased $40,000 $80,000
Research hours 24,000 30,000
Interview hours 1,000 10,000
Number of reports 16,000 3,000

Required:

For each of the four potential allocation bases, determine the amount of support-service cost allocated to
each type of report. Round all percentages to two decimal places.  
 

11-45
Copyright © 2017 McGraw-Hill Education. All rights reserved. No reproduction or distribution without the prior written consent of
McGraw-Hill Education.
108. Data Master is a computer software consulting company. Its three major functional areas are computer
programming, information systems consulting, and software training. Cynthia Moore, a pricing analyst in
the Accounting Department, has been asked to develop total costs for the functional areas. These costs will
be used as a guide in pricing a new contract. In computing these costs, Moore is considering three different
methods of allocating overhead costs-the direct method, the step method, and the reciprocal method.
Moore assembled the following data on overhead from its two service departments, the Information
Systems Department and the Facilities Department.

Service
  User Departments  
Departments
Compu
Info
ter
  Syste Facili Consu Trai
Progra Total
ms ties lt ning
m
Budget
ed $50,00 $25,00 $110,0 $85,0 $345,00
$75,000
Overhe 0 0 00 00 0
ad
Info
System   400 1,100 600 900 3,000
s (hrs)
Faciliti
200,00 600,00 800,0 2,000,0
es (Sq   400,000
0 0 00 00
ft)

Information systems is allocated on the basis of hours of computer usage; facilities are allocated on the
basis of floor space.

Required:

Allocate the service department costs to the user departments using the direct method. (Round to the
nearest dollar and provide total user department costs)  
 

11-46
Copyright © 2017 McGraw-Hill Education. All rights reserved. No reproduction or distribution without the prior written consent of
McGraw-Hill Education.
 

11-47
Copyright © 2017 McGraw-Hill Education. All rights reserved. No reproduction or distribution without the prior written consent of
McGraw-Hill Education.
109. Data Master is a computer software consulting company. Its three major functional areas are computer
programming, information systems consulting, and software training. Cynthia Moore, a pricing analyst in
the Accounting Department, has been asked to develop total costs for the functional areas. These costs will
be used as a guide in pricing a new contract. In computing these costs, Moore is considering three different
methods of allocating overhead costs-the direct method, the step method, and the reciprocal method.
Moore assembled the following data on overhead from its two service departments, the Information
Systems Department and the Facilities Department.
 
Service
  User Departments  
Departments
Compu
Info
ter
  Syste Facili Consu Trai
Progra Total
ms ties lt ning
m
Budget
ed $50,00 $25,00 $110,0 $85,0 $345,00
$75,000
Overhe 0 0 00 00 0
ad
Info
System   400 1,100 600 900 3,000
s (hrs)
Faciliti
200,00 600,00 800,0 2,000,0
es (Sq   400,000
0 0 00 00
ft)

Information systems is allocated on the basis of hours of computer usage; facilities are allocated on the
basis of floor space.

Required:

Allocate the service department costs to the user departments using the step method. Allocate Information
Systems first and round to the nearest dollar. Provide total user department costs.  
 

11-48
Copyright © 2017 McGraw-Hill Education. All rights reserved. No reproduction or distribution without the prior written consent of
McGraw-Hill Education.
 

 
110. Yellville Regional Hospital is a small hospital with two service departments and three revenue areas:
 
Direct Laundry
Service Dept Costs Sq Ft Pounds
   Housekeeping $80,000 - 16,000
   Laundry $132,000 500  
Revenue Areas      
   Surgery $400,000 1,500 48,000
   Semiprivate  rooms $200,000 2,000 24,000
   Maternity $150,000 1,000 12,000

The hospital wants to allocate the service department costs to the revenue areas. Housekeeping is allocated
based on square footage; Laundry is allocated based on pounds of laundry. The normal capacity for
Surgery is 200 hours per month; normal capacity for semiprivate rooms is 600 patient days; and normal
capacity for maternity is 200 patient days.

Required:

Determine the overhead rate for the three revenue areas. Allocate the service department costs to the
revenue areas using the direct method.  
 

11-49
Copyright © 2017 McGraw-Hill Education. All rights reserved. No reproduction or distribution without the prior written consent of
McGraw-Hill Education.
111. Yellville Regional Hospital is a small hospital with two service departments and three revenue areas:
 
Direct Laundry
Service Dept Costs Sq Ft Pounds
   Housekeeping $80,000 - 16,000
   Laundry $132,000 500  
Revenue Areas      
   Surgery $400,000 1,500 48,000
   Semiprivate Rooms $200,000 2,000 24,000
   Maternity $150,000 1,000 12,000

The hospital wants to allocate the service department costs to the revenue areas. Housekeeping is allocated
based on square footage; Laundry is allocated based on pounds of laundry. The normal capacity for
Surgery is 200 hours per month; normal capacity for semiprivate rooms is 600 patient days; and normal
capacity for maternity is 200 patient days.

Required:

Determine the overhead rate for the three revenue areas. Allocate the service department costs to the
revenue areas using the step method. Allocate the service department with the largest dollar value first.  
 

11-50
Copyright © 2017 McGraw-Hill Education. All rights reserved. No reproduction or distribution without the prior written consent of
McGraw-Hill Education.
112. Jack Donaldson owns and operates Jack's Abstracting Service. Jack's two revenue generating operations
(Abstracting Services and Closing Services) are supported by two service departments: Clerical and
Custodial. Costs in the service departments are allocated in the following order using the designated
allocation bases.
Clerical: number of transactions processed.
Custodial: square footage of space occupied.
Average and expected activity levels for next month are as follows:
 
Numbers of Square Expected
 
Transactions Footage     Costs
Abstract services 50 1,800  
Closing services 25 2,200  
Clerical   1,600 $40,000
Custodial 5   10,000

Required:

Use the direct method to allocate the service department costs to the revenue generating departments.
Provide the total costs for the revenue departments.  
 

11-51
Copyright © 2017 McGraw-Hill Education. All rights reserved. No reproduction or distribution without the prior written consent of
McGraw-Hill Education.
113. Jack Donaldson owns and operates Jack's Abstracting Service. Jack's two revenue generating operations
Abstracting Services and Closing Services are supported by two service departments: Clerical and
Custodial. Costs in the service departments are allocated in the following order using the designated
allocation bases.
Clerical: number of transactions processed.
Custodial: square footage of space occupied.
Average and expected activity levels for next month are as follows:
 
Number of Square Expected
 
Transactions Footage     Costs
Abstract services 50 1,800  
Closing services 25 2,200  
Clerical   1,600 $40,000
Custodial 5   10,000

Required:

a. Use the step method to allocate the service department costs to the revenue generating departments.
Assume Clerical costs are allocated before Custodial costs and round all calculations to the nearest whole
dollar. Provide the total costs for the revenue departments.
b. Use the step method to allocate the service department costs to the revenue generating departments but
now assume Custodial costs are allocated before Clerical costs. Provide the total costs for the revenue
departments.  
 

11-52
Copyright © 2017 McGraw-Hill Education. All rights reserved. No reproduction or distribution without the prior written consent of
McGraw-Hill Education.
114. Ardvark Industries has two production departments, Assembly and Finishing and three Service
Departments, Personnel, Maintenance, and Cafeteria. Data relevant to Ardvark are:
 
Direct Person Mainten Cafet Assem Finis
Dept
Cost nel ance eria bly hing
Personne $500,
  0.10   0.70 0.20
l 000
Mainten 420,0
      0.80 0.20
ance 00
200,0
Cafeteria 0.20 0.20   0.30 0.30
00
Finishin 150,0
         
g 00
Assembl 380,0
         
y 00

Required:

Allocate the service department costs of Ardvark Industries using the step-method of cost allocation.  
 

11-53
Copyright © 2017 McGraw-Hill Education. All rights reserved. No reproduction or distribution without the prior written consent of
McGraw-Hill Education.
115. Mena Corporation has two production departments, Assembly and Finishing and three service departments,
Personnel, Maintenance, and Cafeteria. Data relevant to Mena are:
 
Direct Person Mainten Cafet Assem Finis
Dept
cost nel ance eria bly hing
Personne $500,
  0.10   0.70 0.20
l 000
Mainten 420,0
      0.80 0.20
ance 00
200,0
Cafeteria 0.20 0.20   0.30 0.30
00
Finishin 150,0
         
g 00
Assembl 380,0
         
y 00

Assembly and Finishing work on two jobs during the month: Job 100 and 101. Costs are allocated to jobs
based on machine hours in assembly and labor hours in Finishing. The machine and labor hours worked in
each department are as follows:
 
    Assembly Finishing
Job 100 Labor Hours 200 800
  Machine Hours 1,000 200
Job 101 Labor Hours 100 900
  Machine Hours 500 100

Required:

Determine the amount of service department costs to be allocated to Jobs 100 and 101. Mena allocates
service department costs to production departments using the direct-method of allocation.  
 

11-54
Copyright © 2017 McGraw-Hill Education. All rights reserved. No reproduction or distribution without the prior written consent of
McGraw-Hill Education.
116. Boston Corporation has two production Departments: Assembly and Machining and two service
departments: Personnel and Cafeteria. Direct costs for each department and the proportion of service costs
used by the various departments for the month of July, 2016 are as follows:
 
    Proportion of Services Used by:
Departme Direct Personn Cafeter Machini Assemb
nt costs el ia ng ly
$30,00
Personnel   0.40 0.30 0.30
0
$50,00
Cafeteria 0.20   0.50 0.30
0
Machinin $80,00
       
g 0
$70,00
Assembly        
0

Required:

Compute the allocation of service department costs to producing departments for July, 2016 using the
step-method.  
 

11-55
Copyright © 2017 McGraw-Hill Education. All rights reserved. No reproduction or distribution without the prior written consent of
McGraw-Hill Education.
117. Data Master is a computer software consulting company. Its three major functional areas are computer
programming, information systems consulting, and software training. Cynthia Moore, a pricing analyst in
the Accounting Department, has been asked to develop total costs for the functional areas. These costs will
be used as a guide in pricing a new contract. In computing these costs, Moore is considering three different
methods of allocating overhead costs-the direct method, the step method, and the reciprocal method.
Moore assembled the following data on overhead from its two service departments, the Information
Systems Department and the Facilities Department.
 
Service
  User Departments  
Departments
Compu
Info
ter
  Syste Facili Consu Trai
Progra Total
ms ties lt ning
m
Budget
ed $50,00 $25,00 $110,0 $85,0 $345,00
$75,000
Overhe 0 0 00 00 0
ad
Info
System   400 1,100 600 900 3,000
s (hrs)
Faciliti
200,00 600,00 800,0 2,000,0
es (Sq   400,000
0 0 00 00
ft)

Information systems is allocated on the basis of hours of computer usage; facilities are allocated on the
basis of floor space.

Required:

Allocate the service department costs to the user departments using the reciprocal method. Round to the
nearest dollar.  
 

11-56
Copyright © 2017 McGraw-Hill Education. All rights reserved. No reproduction or distribution without the prior written consent of
McGraw-Hill Education.
118. Jack Donaldson owns and operates Jack's Abstracting Service. Jack's two revenue generating operations
(Abstracting Services and Closing Services) are supported by two service departments: Clerical and
Custodial. Costs in the service departments are allocated in the following order using the designated
allocation bases.
Clerical: number of transactions processed.
Custodial: square footage of space occupied.
Average and expected activity levels for next month are as follows:

Number of Square Expected


 
Transactions  Footage   Costs
Abstract services 50 1,800  
Closing services 25 2,200  
Clerical   1,600 $40,000
Custodial 5   10,000

Required:

Use the reciprocal method to allocate the service department costs to the revenue generating departments.
Provide the total costs for the revenue departments.  
 

11-57
Copyright © 2017 McGraw-Hill Education. All rights reserved. No reproduction or distribution without the prior written consent of
McGraw-Hill Education.
119. Yellville Regional Hospital is a small hospital with two service departments and three revenue areas:
 
Direct Laundry
Service Dept Costs Sq Ft Pounds
   Housekeeping $80,000 - 16,000
   Laundry $132,000 500  
Revenue Areas      
   Surgery $400,000 1,500 48,000
   Semiprivate Rooms $200,000 2,000 24,000
   Maternity $150,000 1,000 12,000

The hospital wants to allocate the service department costs to the revenue areas. Housekeeping is allocated
based on square footage; Laundry is allocated based on pounds of laundry. The normal capacity for
Surgery is 200 hours per month; normal capacity for semiprivate rooms is 600 patient days; and normal
capacity for maternity is 200 patient days.

Required:

Determine the overhead rate for the three revenue areas. Allocate the service department costs to the
revenue areas using the reciprocal method.  
 

11-58
Copyright © 2017 McGraw-Hill Education. All rights reserved. No reproduction or distribution without the prior written consent of
McGraw-Hill Education.
120. Franklin Corporation has three operating departments (Fabricating, Assembly, and Finishing) and two
service departments (Custodial and Administrative). The following information has been provided:
 
Custodi Fabrica Assem Finish
  Admin
al ting bly ing
$250,00 $400,0
Dept Costs -- -- --
0 00
# employees 10 -- 80 100 60
Sq ft -- 15,000 30,000 35,000 20,000
Allocations are based on the
     
following:
Square
    Custodial:        
feet
Number
    Administra of
       
tive: employe
es

Required:

Franklin has been approached by Sparkle Cleaning to outsource the custodial service. Assuming all costs
are variable, what is the relevant cost of the custodial department to compare with the Sparkle Cleaning
bid?  
 

11-59
Copyright © 2017 McGraw-Hill Education. All rights reserved. No reproduction or distribution without the prior written consent of
McGraw-Hill Education.
121. The Joplin Company conducts a simple chemical process in Department #1, which produces three separate
items: A, K, and H. A is processed further in Department #2. K is processed further in Department #3.
Product H is a by-product, to be accounted for by the cost reduction method. The following information
relates to September:

Department #1's costs $420,000.


Department #2's costs $150,000.
Department #3's costs $60,000.

A: 25,000 pounds completed; 23,500 pounds sold for $12 per pound.
K: 75,000 pounds completed; 70,000 pounds sold for $7.50 per pound.
H: 10,000 pounds completed; 10,000 pounds sold for $1.50 per pound. (There are shipping costs of $0.30
per pound.)

There were no September 1st inventories.

Required:

Prepare a schedule to show the computation for the unit costs per pound for Products A, K, and H
assuming Joplin uses the estimated net realizable value method to allocate joint costs to the main products.

11-60
Copyright © 2017 McGraw-Hill Education. All rights reserved. No reproduction or distribution without the prior written consent of
McGraw-Hill Education.
122. Simpson Manufacturing Enterprises uses a joint production process that produces three products at the
split-off point. Joint production costs during April were $720,000. The company uses the net realizable
value method for allocating joint costs. Product information for April was as follows:
 
  Product
  R S T
Gallons produced 2,500 5,000 7,500
Sales prices per gallon:      
   At the split-off $100 $80 $20
   After further
$150 $115 $30
processing
Costs to process after
$150,000 $150,000 $100,000
split-off

Required:

a. Assume that all three products are main products and that they can be sold at the split-off point or
processed further, whichever is economically beneficial to Simpson. Allocate the joint costs to the three
products.
b. Assume that Simpson uses the physical quantities method to allocate the joint costs. How much would
be allocated to each of the three products?  
 

11-61
Copyright © 2017 McGraw-Hill Education. All rights reserved. No reproduction or distribution without the prior written consent of
McGraw-Hill Education.
123. Clean-Burn, Inc. is a small petroleum company that acquires high-grade crude oil from low-volume
production wells owned by individuals and small partnerships. The crude oil is processed in a single
refinery into Two Oil, Six Oil, and impure distillates. Clean-Burn does not have the technology or capacity
to process these products further and sells most of its output each month to major refineries. There were no
inventories on November 1.

Crude oil acquired and placed into


$5,000,000
production
Direct labor and related costs 2,000,000
Refinery overhead 3,000,000

Production and sales


Two Oil, 300,000 barrels produced; 280,000 barrels sold at $20 each.
Six Oil, 240,000 barrels produced; 220,000 barrels sold at $30 each.
Distillates, 120,000 barrels produced and sold at $15 per barrel.

Required:

a. Allocate the joint costs to the products using the physical quantities method.
b. Allocate the joint costs to the products using the net realizable value method.  
 

11-62
Copyright © 2017 McGraw-Hill Education. All rights reserved. No reproduction or distribution without the prior written consent of
McGraw-Hill Education.
124. Smokey Enterprises buys Liquid Charcoal for $0.80 a gallon. At the end of processing in department 1, the
liquid charcoal splits off into Products U, V, and W. Product U is sold at the split-off point, with no further
processing. Products V and W require further processing before they can be sold; Product V is processed in
Department 2, and Product W is processed in Department 3. Following is a summary of costs and other
related data for the most recent accounting period:
 
  Department
  1 2 3
Cost of liquid charcoal $104,000    
Direct labor 16,000 45,000 65,000
Manufacturing overhead 10,000 27,000 49,000
  Products
  U V W
Gallons sold 20,000 30,000 50,000
Gallons on hand end of
15,000 0 15,000
period
Sales in dollars $30,000 $96,000 $142,000

There were no beginning inventories and there was no liquid charcoal on hand at the end of the period. All
gallons on hand in ending inventory were complete as to processing. Smokey uses the estimated net
realizable value method of allocating joint costs.

Required:

a. Determine the product cost for U, V, and W, assuming the physical quantity method is used to allocate
joint costs.
b. Determine the product cost for U, V, and W, assuming the net realizable value method is used to
allocate joint costs.  
 

11-63
Copyright © 2017 McGraw-Hill Education. All rights reserved. No reproduction or distribution without the prior written consent of
McGraw-Hill Education.
125. The Wang Company conducts a simple chemical process in Department #1, which produces three separate
items: A, B, and C. A is processed further in Department #2. B is processed further in Department #3.
Product C can be sold immediately. The following information relates to October:

Department #1's costs $540,000.


Department #2's costs $120,000.
Department #3's costs $300,000.

A: 25,000 pounds completed; 23,500 pounds sold for $12 per pound.
B: 75,000 pounds completed; 70,000 pounds sold for $7.50 per pound.
C: 50,000 pounds completed; 46,000 pounds sold for $5.00 per pound.

There were no October 1st inventories.

Required:

a. Allocate the joint process costs to Products A, B, and C assuming the estimated net realizable value
method is used.
b. Allocate the joint process costs to products A, B, and C assuming the physical quantities method is
used.  
 

11-64
Copyright © 2017 McGraw-Hill Education. All rights reserved. No reproduction or distribution without the prior written consent of
McGraw-Hill Education.
126. The Marketplace Corporation produces two consumer products and a by-product. Zylon is ready for sale
after split-off, while Qytol must be further processed. The by-product is a heavy residue in the bottom of
the vat. The net realizable value of the by-product is credited against the $565,000 joint cost of the Heating
Department. Volume and cost data for February is as follows:

Gallons Selling Additional


 
Produced Price Processing
Zylon 200,000 $2.00 0
Qytol 400,000 1.10 $40,000
By-Product 5,000 0.50 0

Required:

a. Allocate the Heating Department cost to the products using the physical quantities method.
b. Allocate the Heating Department cost to the products using the workback method.  
 

11-65
Copyright © 2017 McGraw-Hill Education. All rights reserved. No reproduction or distribution without the prior written consent of
McGraw-Hill Education.
127. The Delicious Canning Company processes tomatoes into ketchup, tomato juice, and canned tomatoes.
During the summer, the joint costs of processing the tomatoes were $420,000. There was no beginning or
ending inventories for the summer. Production and sales value information for the summer were as
follows:
 
Product Cases Additional Costs Selling Price
Ketchup 100,000 $3.00 per case $28 per case
Juice 150,000 5.00 per case $25 per case
Canned 200,000 2.50 per case 10 per case

Required:

a. Determine the amount allocated to each product if the estimated net realizable value method is used.
b. Determine the amount allocated to each product if the physical units method is used.  
 

11-66
Copyright © 2017 McGraw-Hill Education. All rights reserved. No reproduction or distribution without the prior written consent of
McGraw-Hill Education.
128. The Joplin Company conducts a simple chemical process in Department #1, which produces three separate
items: A, K, and H. A is processed further in Department #2. K is processed further in Department #3.
Product H is a by-product, to be accounted for by the other revenue method. The following information
relates to September:

Department #1's costs $420,000.


Department #2's costs $150,000.
Department #3's costs $60,000.

A: 25,000 pounds completed; 23,500 pounds sold for $12 per pound.
K: 75,000 pounds completed; 70,000 pounds sold for $7.50 per pound.
H: 10,000 pounds completed; 10,000 pounds sold for $1.50 per pound. (There are shipping costs of $0.30
per pound.)

There were no September 1st inventories.

Required:

Prepare a schedule to show the computation for the unit costs per pound for Products A, K, and H
assuming Joplin uses the physical quantities method to allocate joint costs to the main products.  
 

11-67
Copyright © 2017 McGraw-Hill Education. All rights reserved. No reproduction or distribution without the prior written consent of
McGraw-Hill Education.
129. Highlands, Inc. operates a sawmill facility. The company accounts for the sawdust that results from the
primary sawing operation as a by-product. The sawdust is sold to another company at a price of $1.00 per
hundred cubic feet. Normally, sales revenue from the sawdust is $21,200 per month. The sawdust is
charged to inventory at $2.20 per hundred cubic feet, although there is no direct cost to process it.
As an alternative, Highlands can rent equipment that will process the dust into imitation logs for fireplaces.
These logs sell for $25.00 per hundred to wholesalers, who package and add scent to them. 75 logs can be
produced from 100 cubic feet of sawdust.
Cost of the equipment to produce these logs and the additional personnel required to operate the equipment
are $360,000 per month, regardless of the output.

Required:

A) Should Highlands sell the sawdust for $1.00 per hundred cubic feet or process it into imitation logs?
Support your answer with the appropriate calculations.  
 

11-68
Copyright © 2017 McGraw-Hill Education. All rights reserved. No reproduction or distribution without the prior written consent of
McGraw-Hill Education.
130. Voorhees Manufacturing Corporation produces three products in a joint process.
Additional information is as follows:
 
  O P Q Total
Units produced 42,000 50,000 8,000 100,000
Sales value at
$250,000 $50,000 $20,000 $320,000
split off
Additional
costs if
$18,000 $30,000 $10,000 $58,000
processed
further
Sales value if
processed $290,000 $70,000 $25,000 $385,000
further
Joint costs       $300,000
Product
weights in 84,000 150,000 8,000 242,000
pounds

Required:

(a) Determine which products should be sold at split-off and which should be processed further.
(b) Assuming Voorhees makes decisions that are in its best interests for overall profitability, what would
be the company's gross margin?  
 

11-69
Copyright © 2017 McGraw-Hill Education. All rights reserved. No reproduction or distribution without the prior written consent of
McGraw-Hill Education.
 

11-70
Copyright © 2017 McGraw-Hill Education. All rights reserved. No reproduction or distribution without the prior written consent of
McGraw-Hill Education.
131. Ridgeline Enterprises produces three products in a joint process. Products A and B were processed further.
Additional information is as follows:

  A B C Total
Units produced 42,000 50,000 8,000 100,000
Sales value at
$250,000 $30,000 $20,000 $300,000
split-off
Additional
costs if
$18,000 $30,000 $0 $48,000
processed
further
Sales value if
processed $290,000 $70,000 $0 $360,000
further
Joint Costs       $200,000
Product Weight
168,000 300,000 32,000 500,000
in pounds

Required:

(a) Allocate the joint costs, assuming that all products are joint products and joint-costs are allocated using
the physical-measures method.
(b) Allocate the joint costs using the physical-measures method, assuming that product C is considered a
by-product, whose sales value is deducted from the total joint costs.  
 

11-71
Copyright © 2017 McGraw-Hill Education. All rights reserved. No reproduction or distribution without the prior written consent of
McGraw-Hill Education.
132. Geneva Powder Company produces body powders in batches. Each type of powder can be sold in its
current condition or processed further and specialized for high priced department stores. Data concerning
the various products appear below. Joint processing costs are $200,000.

Price
per
Further
Numbe Pound Price after
Processing
Type of r of at Processing
Costs
Powder Pounds Split- Further
Off
Cosmetic
200,000 $10 $150,000 $11.50
Powder
Medicated
400,000 $8 $60,000 $8.40
Powder
Baby
50,000 $5 $80,000 $5.50
Powder

Required:

(a) Determine which products should be sold at split-off and which should be processed further.
(b) Geneva Powder is approached by the Regis Department Store chain. Regis would like Geneva Powder
to process regular powder into a special powder for its cosmetics department. At what price per pound
would Geneva Powder be economically indifferent between selling the powder at the split-off point and
processing it further for Regis?  
 

11-72
Copyright © 2017 McGraw-Hill Education. All rights reserved. No reproduction or distribution without the prior written consent of
McGraw-Hill Education.
133. Indicate whether the following costs would be treated as joint-product costs or costs incurred after the split-
off point. Use J for joint product costs and S for costs incurred after the split-off point.
 
Cost of planting, growing and picking pineapples in a pineapple
(a)                     
factory
(b) Costs of processing apples at a cider mill                      
(c) Costs of processing pineapples into juice and sliced pineapples                      
(d) Depreciation on oil rigs for an oil producer                      
Costs of running a fishing boat used to catch varieties of fish,
(e)                      
lobsters, etc.
(f) Labor costs, of “shucking” clams to produce clam chowder                      
Costs of chopping onions to be used in spaghetti sauce and soup in a
(g)                      
food manufacturer
Cost of processing rejected meat parts into hot dogs in a meat
(h)                      
processing plant
Cost of processing wood and sawdust into particle board in a
(i)                      
sawmill
(j) Ingredients and packaging added to batches of spaghetti in (g) above __________
(k) Costs of refining gasoline in (d) above                      
(l) Processing of pulp into paperboard in a paper manufacturer                      
(m) Utility costs of processing timber for a lumber manufacturer                      
 
 

11-73
Copyright © 2017 McGraw-Hill Education. All rights reserved. No reproduction or distribution without the prior written consent of
McGraw-Hill Education.
134. The Macon Industries started the production of K1 (its main product) and S2 (its by-product) on January 2,
2016. During 2016, 7,500 units of K1 and 1,500 units of S2 were produced. In 2016, 6,000 units of K1 and
1,000 units of S2 were sold at $57.00 and $1.10 per unit, respectively. Production was halted at the end of
2016 and the inventory was sold in 2017 at the normal selling prices. The joint production costs were
$240,000 and are entirely avoidable. The separable costs to produce K1 were $2.60 per unit and to produce
S2 were $0.45 per unit. Operating expenses were $60,000 in 2016 and $12,000 in 2017.

Required:

a. Prepare an income statement for 2016 and 2017 assuming the "other revenue" method of accounting for
by-products is used.
b. Prepare an income statement for 2016 and 2017, assuming the "cost reduction" method of accounting
for by-products is used.  
 

 
135. What is the difference between an intermediate cost center and a final cost center?  
 

 
136. Describe the difference between the direct method of service department allocation, the step method, and
the reciprocal method. 
 

11-74
Copyright © 2017 McGraw-Hill Education. All rights reserved. No reproduction or distribution without the prior written consent of
McGraw-Hill Education.
137. Why does the sequence in which service departments are allocated make a difference when using the step
method but not when using the reciprocal method? 
 

 
138. Which of the three service department allocation methods should be used for decision making? Explain
your reasoning. 
 

 
139. What are some of the reasons that joint costs are allocated? 
 

11-75
Copyright © 2017 McGraw-Hill Education. All rights reserved. No reproduction or distribution without the prior written consent of
McGraw-Hill Education.
140. Explain the difference between the net realizable value method for joint cost allocation and the netback (or
workback) method. 
 

 
141. In a sell-or-process-further decision (a) what are the relevant data to be considered and (b) what is the
decision process associated with the split-off point?  
 

 
142. Describe two methods of accounting for by-products. What effects do these methods have on the allocation
of the joint cost to the main products?  
 

11-76
Copyright © 2017 McGraw-Hill Education. All rights reserved. No reproduction or distribution without the prior written consent of
McGraw-Hill Education.
143. Boswell Consulting has two service departments: S1 and S2 and three production departments: P1, P2, and
P3. Data for a recent month follow:
 
  S1 S2 P1 P2 P3
Service
Provide   0.10 0.20 0.40 0.30
d to:
  0.30   0.20 0.40 0.10
$200,00 $100,00 $600,00 $800,00 $1,000,00
Costs
0 0 0 0 0

Required:

(a) Determine the allocations to the production departments when the reciprocal method is used.
(b) Briefly describe why the reciprocal method is theoretically preferable to other methods of allocation.  
 

11-77
Copyright © 2017 McGraw-Hill Education. All rights reserved. No reproduction or distribution without the prior written consent of
McGraw-Hill Education.
144. Morgan and Regis Consultants is a large, international consulting organization. The company provides
consulting services in the computer and Internet areas. The company also has several divisions that provide
manufacturing of various computer parts. The company has five divisions which are all profit centers. Each
division includes allocated corporate costs in its annual budget. The budget for the coming year for the
Building and Grounds Service department is $6,000,000. Included in this budget is the maintenance of all
corporate buildings, depreciation, cleaning, insurance, and all other facility-related maintenance costs. The
company uses a weighted method of allocating facility costs based on the type of space maintained by each
division. Space ranges from manufacturing warehouses, which are least expensive to maintain, to computer
mainframe space, which requires specialized temperature controls, air conditioning, and maintenance. The
company has decided to use a weighting system assigning the following relative weights to each type of
space: 1 for warehouse, 3 for office, and 5 for computer space. Below, find data relating to the five
divisions and the square footage of each type of space. Currently, Division 5, the Internet consulting
division, is the largest in sales volume and profits for the company, which has been growing at the rate of
20% per year, while divisions 3 and 4 have been struggling due to declining margins on technology
products.
 
    Type of space  
  Office Computer Warehouse Total
Weighting 3 5 1  
Division 1 10000 0 0 10000
Division 2 31000 10000 5000 46000
Division 3 15000 12000 32000 59000
Division 4 15000 10000 50000 75000
Division 5 30000 30000 0 60000
Total 101000 62000 87000 250000

Required:

(a) As director of corporate budgeting, you are required to send to each division its facility allocation for
the coming year. Prepare a schedule showing how the budget of $6,000,000 will be allocated to each
division.
(b) Describe potential motivational problems brought on by these allocations. (Based on an actual
company)  
 

11-78
Copyright © 2017 McGraw-Hill Education. All rights reserved. No reproduction or distribution without the prior written consent of
McGraw-Hill Education.
145. Castro Corporation has one service department and three producing departments. The budget for the
following year allocates the service department costs to the producing departments based on the number of
employees in each department. Currently, the budget for the service department is $2,400,000 and the
number of employees in each department is as follows:

Department 1: 100
Department 2: 50
Department 3: 150

During the year, due to sudden expanded growth, Department 2 has to add 50 new employees; however
the service department costs have not increased due to budget constraints.

Required:

(a) What were the expected service department allocations at the beginning of the year to each production
department?
(b) What will be the actual allocations based on the number of employees each department has at year
end?
(c) Comment on the reasonableness of the situation. What are the potential causes of any problems created
by this allocation method?  
 

11-79
Copyright © 2017 McGraw-Hill Education. All rights reserved. No reproduction or distribution without the prior written consent of
McGraw-Hill Education.
146. Liberty Credit Checks produces two styles of credit reports: personal and corporate. The difference
between the two is the amount of background information and data collection required. The corporate
report uses more skilled personnel because additional checking and data are required. The relevant figures
for the year just completed follow: Total support service costs to be allocated are $3,200,000.
 
Allocation base Individual Corporate
Data purchased $40,000 $80,000
Research hours 24,000 30,000
Interview hours 1,000 10,000
Number of reports 16,000 3,000

Required:

(a) Which method would be preferred by each manager? Which method would be least preferred?
(b) Provide arguments that each manager would make for his/her preferred method. How would each
manager argue against his/her least preferred method?  
 

11-80
Copyright © 2017 McGraw-Hill Education. All rights reserved. No reproduction or distribution without the prior written consent of
McGraw-Hill Education.
147. Portofino Manufacturing Corporation manufactures three products in a joint process. Additional
information is as follows:
 
  J K L Total
Units
16,000 4,000 2,000 22,000
produced
Sales value at
$300,000 $100,000 $20,000 $420,000
split-off
Addition costs
if
  Processed $48,000 $20,000 $6,000 $74,000
further
Sales value if
processed
$340,000 $160,000 $40,000 $540,000
   Further
Joint costs       $120,000

Required:

(a) Allocate the joint costs to the three products using the net realizable value method.
(b) Determine which products should be sold at split-off and which products should be processed further.  
 

11-81
Copyright © 2017 McGraw-Hill Education. All rights reserved. No reproduction or distribution without the prior written consent of
McGraw-Hill Education.
148. Dawson Corporation produces a product called Blocker, which gives rise to a by-product called Spotter.
The only costs associated with Spotter are additional processing costs of $4 for each unit. Dawson accounts
for Spotter's sales first by deducting its separable costs from its sales and then by deducting this net amount
from the cost of sales of Blocker. This year, 9,600 units of Spotter were produced. They were all sold for
$8 each. Company operating expenses were $250,000 for the year. Sales revenue and cost of goods sold for
Blocker were $1,600,000 and $800,000 respectively. (CPA adapted)

Required:

(a) Calculate the company's gross margin under the current accounting method.
(b) Assume the company changes its accounting method and accounts for the by-product's net realizable
value as "other revenue." Calculate the gross margin under the new method.
(c) Under what circumstances would method a or b be preferred?  
 

11-82
Copyright © 2017 McGraw-Hill Education. All rights reserved. No reproduction or distribution without the prior written consent of
McGraw-Hill Education.
149. Bartoff Foods produces three supplemental food products simultaneously through a refining process
costing $186,000. The joint products, Bulkup and Bodybuilder, have a final selling price of $8 per pound
and $10 per pound, respectively, after additional processing costs of $2 per pound for each product
incurred after the split-off point. Quicksnack, a by-product, is sold at the split-off point for $6 per pound.
The production of Bulkup results in 20,000 pounds with a caloric value of 6,000 calories per pound. The
production of Bodybuilder, which is very high in carbohydrates, has a caloric value of 12,500 calories per
pound. 10,000 pounds of Bodybuilder are produced. Quicksnack has a caloric value of 2,000 calories a
pound and 2,000 pounds are produced. (CMA adapted)

Required:

(a) Allocate the joint product costs using the net-realizable-value method, assuming that Quicksnack is
accounted for as a by-product, with its net realizable value deducted from the cost of the main products.
(b) Allocate the joint product costs using the physical measures method, assuming that Quicksnack is
accounted for as a by-product, with its sales revenue accounted for as "other revenue."
(c) Compute Bartoff Food's gross margin under requirements a and b.  
 

11-83
Copyright © 2017 McGraw-Hill Education. All rights reserved. No reproduction or distribution without the prior written consent of
McGraw-Hill Education.
150. Timberland Corporation produces three products from a joint process: One-X, Two-Y, and Three-Z. Each
product can be processed further and sold for more. Data on the processes are as follows:

Product One-X Two-Y Three-Z Total


Units produced 16,000 8,000 4,000 28,000
Joint costs $60,000 (a) (b) $120,000
Sales value at
(c) (d) $30,000 $200,000
split-off
Additional
Processing $14,000 $10,000 $6,000 $30,000
Costs
Sales value if
Processed $140,000 $60,000 $40,000 $240,000
Further

The amount of joint costs for One-X is the amount that has been allocated.

Required:

Determine the values for the lettered spaces. (CPA adapted)  


 

11-84
Copyright © 2017 McGraw-Hill Education. All rights reserved. No reproduction or distribution without the prior written consent of
McGraw-Hill Education.
151. Penny's Pineapples is a pineapple grower. After cultivating, fertilizing, growing, and picking pineapples,
the company sells whole pineapples to food processors. The company is considering adding a processing
line where sliced pineapples and pineapple juice, along with a "mash" used for animal feed will be the final
products. Projected information about the costs follows:
 
Units Separable Final
Product
produced costs selling price
  Per unit    
Sliced $3.00 per
900,000 cans $600,000
pineapple can
Pineapple 400,000 $1.75 per
$150,000
juice bottles bottle
500,000 $.50 per
Mash $120,000
pounds pound

Joint product costs of cultivating, fertilizing and picking pineapples total $1,000,000.

Required:

(a) Determine the amount of separable costs allocated to each product using the net realizable value
method.
(b) Determine the final cost per unit for each product.
(c) Determine the gross margin for each product.
(d) A fertilizer manufacturer approaches Penny Martin, the President of the company, and asks to buy the
rinds and other excess materials currently used to produce Mash. He would be willing to pay $0.30 per
pound for these materials. What advice would you give Penny?  
 

11-85
Copyright © 2017 McGraw-Hill Education. All rights reserved. No reproduction or distribution without the prior written consent of
McGraw-Hill Education.
152. Fantasy Manufacturing produces three products in a joint operation. Information regarding the products
appears below:

  Item 1 Item 2 Item 3 Total


Units
20,000 25,000 10,000 55,000
Produced
Sales Value at
$150,000 $50,000 $20,000 $220,000
Split-off
Additional
costs if
$10,000 $30,000 $5,000 $45,000
Processed
further
Sales Value if
Processed $170,000 $90,000 $28,000 $288,000
Further
Joint Costs       $100,000

Required:

(a) Allocate the joint costs using the relative sales value at split-off method.
(b) Allocate the joint costs using the constant gross margin percentage method.  
 

11-86
Copyright © 2017 McGraw-Hill Education. All rights reserved. No reproduction or distribution without the prior written consent of
McGraw-Hill Education.
Chapter 11 Service Department and Joint Cost Allocation Answer Key

True / False Questions


 

1. The human resource department in a manufacturing company would be considered a service


department. 
 
TRUE

Human resources would be considered a support department for the manufacturing activities of the firm,
and therefore a service department.

 
AACSB: Analytical Thinking
AICPA: FN Measurement
Accessibility: Keyboard Navigation
Blooms: Remember
Difficulty: 1 Easy
Gradable: automatic
Learning Objective: 11-01 Explain why service costs are allocated.
Topic: Service Department Cost Allocation
 
2. One reason to allocate service department costs to user departments is to encourage the user
departments to monitor their use of the service department costs. 
 
TRUE

Allocating service department costs makes manufacturing managers aware of the support costs required
to complete their manufacturing efforts as well as provide better product costing.

 
AACSB: Analytical Thinking
AICPA: FN Measurement
Accessibility: Keyboard Navigation
Blooms: Remember
Difficulty: 2 Medium
Gradable: automatic
Learning Objective: 11-01 Explain why service costs are allocated.
Topic: Service Department Cost Allocation
 

11-87
Copyright © 2017 McGraw-Hill Education. All rights reserved. No reproduction or distribution without the prior written consent of
McGraw-Hill Education.
3. The direct method makes no cost allocations between or among service departments. 
 
TRUE

All costs are allocated directly to the producing departments, as if service departments were not a user
of services themselves.

 
AACSB: Analytical Thinking
AICPA: FN Measurement
Accessibility: Keyboard Navigation
Blooms: Remember
Difficulty: 1 Easy
Gradable: automatic
Learning Objective: 11-02 Allocate service department costs using the direct method.
Topic: Methods of Allocating Service Department Costs
 
4. The selection of an allocation base in the direct method is easier than the selection of an allocation base
in the step method. 
 
FALSE

The allocation base would remain constant across the three allocation methods.

 
AACSB: Analytical Thinking
AICPA: FN Measurement
Accessibility: Keyboard Navigation
Blooms: Remember
Difficulty: 2 Medium
Gradable: automatic
Learning Objective: 11-02 Allocate service department costs using the direct method.
Topic: Methods of Allocating Service Department Costs
 
5. The step method allocates some, but not all, service department costs to other service departments. 
 
TRUE

In allocating service department costs, the majority of the services provided are usually allocated to the
production departments. Once the costs of a service department have been allocated, it is considered
"closed" and cannot be the recipient in future allocations.

 
AACSB: Analytical Thinking
AICPA: FN Measurement
Accessibility: Keyboard Navigation
Blooms: Remember
Difficulty: 2 Medium
Gradable: automatic
Learning Objective: 11-03 Allocate service department costs using the step method.
Topic: Methods of Allocating Service Department Costs
 

11-88
Copyright © 2017 McGraw-Hill Education. All rights reserved. No reproduction or distribution without the prior written consent of
McGraw-Hill Education.
6. One advantage of the step method is that all reciprocal services are recognized between service
departments. 
 
FALSE

In the step method, once the costs of a service department have been allocated, it is considered "closed"
and cannot be the recipient in future allocations.

 
AACSB: Analytical Thinking
AICPA: FN Measurement
Accessibility: Keyboard Navigation
Blooms: Remember
Difficulty: 1 Easy
Gradable: automatic
Learning Objective: 11-03 Allocate service department costs using the step method.
Topic: Methods of Allocating Service Department Costs
 
7. With the reciprocal method, the total service department costs less the direct costs of the service
department equals the cost allocated to the service department. 
 
TRUE

Direct costs + costs allocated from other services = total cost.

 
AACSB: Analytical Thinking
AICPA: FN Measurement
Accessibility: Keyboard Navigation
Blooms: Apply
Difficulty: 2 Medium
Gradable: automatic
Learning Objective: 11-04 Allocate service department costs using the reciprocal method.
Topic: Methods of Allocating Service Department Costs
 
8. One potential disadvantage of the reciprocal method is it could overstate the cost of running the
organization's service departments. 
 
FALSE

The reciprocal method is the most accurate method.

 
AACSB: Analytical Thinking
AICPA: FN Measurement
Accessibility: Keyboard Navigation
Blooms: Apply
Difficulty: 2 Medium
Gradable: automatic
Learning Objective: 11-04 Allocate service department costs using the reciprocal method.
Topic: Methods of Allocating Service Department Costs
 

11-89
Copyright © 2017 McGraw-Hill Education. All rights reserved. No reproduction or distribution without the prior written consent of
McGraw-Hill Education.
9. In deciding whether to outsource a service department or not, the cost of the service department should
be estimated using the step method of allocation. 
 
FALSE

The reciprocal method should be used as it is the most accurate method.

 
AACSB: Analytical Thinking
AICPA: FN Decision Making
Accessibility: Keyboard Navigation
Blooms: Apply
Difficulty: 2 Medium
Gradable: automatic
Learning Objective: 11-05 Use the reciprocal method approach for outsourcing decisions.
Topic: Methods of Allocating Service Department Costs
 
10. Joint products are outputs from common inputs and a common production process. 
 
TRUE

This is the definition of joint products.

 
AACSB: Analytical Thinking
AICPA: FN Measurement
Accessibility: Keyboard Navigation
Blooms: Remember
Difficulty: 1 Easy
Gradable: automatic
Learning Objective: 11-06 Explain why joint costs are allocated.
Topic: Allocation of Joint Costs
 
11. Joint costs are processing costs incurred after the split-off point in a common production process. 
 
FALSE

Joint costs are the costs that are incurred prior to reaching the split-off point.

 
AACSB: Analytical Thinking
AICPA: FN Measurement
Accessibility: Keyboard Navigation
Blooms: Remember
Difficulty: 1 Easy
Gradable: automatic
Learning Objective: 11-06 Explain why joint costs are allocated.
Topic: Allocation of Joint Costs
 

11-90
Copyright © 2017 McGraw-Hill Education. All rights reserved. No reproduction or distribution without the prior written consent of
McGraw-Hill Education.
12. The estimated net realizable value for a product is its estimated selling price after processing the product
beyond the split-off point. 
 
FALSE

If the products require further processing before they are marketable, it could be necessary to estimate
the net realizable value at the split-off point.

 
AACSB: Analytical Thinking
AICPA: FN Measurement
Accessibility: Keyboard Navigation
Blooms: Remember
Difficulty: 2 Medium
Gradable: automatic
Learning Objective: 11-07 Allocate joint costs using the net realizable value method.
Topic: Joint Cost Allocation Methods
 
13. In general, it is better to use a product's market value at the split-off point than its estimated net
realizable value in allocating joint costs. 
 
TRUE

Actual market value is better than an estimate.

 
AACSB: Analytical Thinking
AICPA: FN Measurement
Accessibility: Keyboard Navigation
Blooms: Apply
Difficulty: 2 Medium
Gradable: automatic
Learning Objective: 11-07 Allocate joint costs using the net realizable value method.
Topic: Joint Cost Allocation Methods
 
14. The estimated net realizable value at the split-off point is calculated by taking the sales value after
further processing and deducting the additional processing costs. 
 
TRUE

This is the definition of estimated net realizable value.

 
AACSB: Analytical Thinking
AICPA: FN Measurement
Accessibility: Keyboard Navigation
Blooms: Remember
Difficulty: 1 Easy
Gradable: automatic
Learning Objective: 11-07 Allocate joint costs using the net realizable value method.
Topic: Joint Cost Allocation Methods
 

11-91
Copyright © 2017 McGraw-Hill Education. All rights reserved. No reproduction or distribution without the prior written consent of
McGraw-Hill Education.
15. If a company's two joint products can be sold at the split-off point, there is no reason for allocating the
joint costs to the products. 
 
FALSE

Just because they can be sold, doesn't mean that they are all sold. Inventory valuation would still be
necessary.

 
AACSB: Analytical Thinking
AICPA: FN Measurement
Accessibility: Keyboard Navigation
Blooms: Remember
Difficulty: 2 Medium
Gradable: automatic
Learning Objective: 11-07 Allocate joint costs using the net realizable value method.
Topic: Joint Cost Allocation Methods
 
16. The physical quantities method of allocating joint costs is often used when the output sales prices are
highly volatile. 
 
TRUE

In this instance, there would not be reliable sales values; physical values would be preferred.

 
AACSB: Analytical Thinking
AICPA: FN Measurement
Accessibility: Keyboard Navigation
Blooms: Apply
Difficulty: 1 Easy
Gradable: automatic
Learning Objective: 11-08 Allocate joint costs using the physical quantities method.
Topic: Joint Cost Allocation Methods
 
17. The physical quantities method allocates joint costs so that each joint product has the same gross margin
as a percentage of sales. 
 
FALSE

Each product has the same cost per pound (or foot, or gallon, etc).

 
AACSB: Analytical Thinking
AICPA: FN Measurement
Accessibility: Keyboard Navigation
Blooms: Remember
Difficulty: 2 Medium
Gradable: automatic
Learning Objective: 11-08 Allocate joint costs using the physical quantities method.
Topic: Joint Cost Allocation Methods
 

11-92
Copyright © 2017 McGraw-Hill Education. All rights reserved. No reproduction or distribution without the prior written consent of
McGraw-Hill Education.
18. In a sell-or-process-further decision, the additional costs incurred after the split-off point are irrelevant. 
 
FALSE

These additional costs are incremental and would be relevant.

 
AACSB: Analytical Thinking
AICPA: FN Decision Making
Accessibility: Keyboard Navigation
Blooms: Remember
Difficulty: 1 Easy
Gradable: automatic
Learning Objective: 11-09 Explain how cost data are used in the sell-or-process-further decision.
Topic: Deciding Whether to Sell Goods Now or Process Them Further
 
19. In a sell-or-process-further decision, the common costs incurred prior to the spilt-off point are
irrelevant. 
 
TRUE

Common costs will be there whatever the end decision is.

 
AACSB: Analytical Thinking
AICPA: FN Decision Making
Accessibility: Keyboard Navigation
Blooms: Remember
Difficulty: 2 Medium
Gradable: automatic
Learning Objective: 11-09 Explain how cost data are used in the sell-or-process-further decision.
Topic: Deciding Whether to Sell Goods Now or Process Them Further
 
20. Since by-products have minor sales value, alternative methods of accounting for them will not have a
material effect on the financial statements. 
 
TRUE

The sales value is immaterial, so the alternate results would be immaterial.

 
AACSB: Analytical Thinking
AICPA: FN Measurement
Accessibility: Keyboard Navigation
Blooms: Remember
Difficulty: 1 Easy
Gradable: automatic
Learning Objective: 11-10 Account for by-products.
Topic: Deciding What to Do with By-Products
 
 

Multiple Choice Questions


 

11-93
Copyright © 2017 McGraw-Hill Education. All rights reserved. No reproduction or distribution without the prior written consent of
McGraw-Hill Education.
21. Allocation of factory service department costs to the production departments is necessary to: 
 

A.  measure use of plant capacity.


B.  make sure that machines are operating efficiently.
C.  calculate cost per unit for purposes of external financial reporting.
D.  control costs.

Accurate total costs cannot be determined unless factory service department cost is allocated in an
appropriate manner.

 
AACSB: Reflective Thinking
AICPA: FN Measurement
Accessibility: Keyboard Navigation
Blooms: Remember
Difficulty: 1 Easy
Gradable: automatic
Learning Objective: 11-01 Explain why service costs are allocated.
Topic: Service Department Cost Allocation
 
22. Which of the following statements is false regarding the use of multiple cost pools? 
 

A.  Using more pools generally results in better cost allocations.


B.  Designing and maintaining a system with multiple pools can be complex and costly.
C.  The use of Activity-Based-Accounting will almost always require multiple cost pools.
D.  All support service costs are generally allocated using a single cost pool.

Each service department has a unique allocation base to be used for expense allocation.

 
AACSB: Reflective Thinking
AICPA: FN Measurement
Accessibility: Keyboard Navigation
Blooms: Remember
Difficulty: 2 Medium
Gradable: automatic
Learning Objective: 11-01 Explain why service costs are allocated.
Topic: Service Department Cost Allocation
 

11-94
Copyright © 2017 McGraw-Hill Education. All rights reserved. No reproduction or distribution without the prior written consent of
McGraw-Hill Education.
23. Which of the following statements is true regarding the use of multiple cost-pools? 
 

A.  Organizations might implement multiple cost pools when the uses of the service resources have both
facility-level and unit-level components.
B.  Designing and maintaining multiple cost pools is a relatively simple undertaking.
C.  The benefits of using multiple cost pools always outweigh the costs of setting them up.
D.  Both resources supplied and resources used are allocated with multiple cost pools.

This is a basic concept underlying the use of multiple cost-pools for lower level costs in the cost
hierarchy.

 
AACSB: Reflective Thinking
AICPA: FN Measurement
Accessibility: Keyboard Navigation
Blooms: Remember
Difficulty: 2 Medium
Gradable: automatic
Learning Objective: 11-01 Explain why service costs are allocated.
Topic: Service Department Cost Allocation
 
24. Which of the following would be an appropriate cost-allocation base for allocating the cost of the
company cafeteria? 
 

A.  Square footage occupied by departments.


B.  Number of hours of use.
C.  Number of meals served.
D.  Salaries of personnel purchasing meals.

Meals served is the most appropriate measure of allocation for the cafeteria.

 
AACSB: Reflective Thinking
AICPA: FN Measurement
Accessibility: Keyboard Navigation
Blooms: Remember
Difficulty: 2 Medium
Gradable: automatic
Learning Objective: 11-01 Explain why service costs are allocated.
Topic: Service Department Cost Allocation
 

11-95
Copyright © 2017 McGraw-Hill Education. All rights reserved. No reproduction or distribution without the prior written consent of
McGraw-Hill Education.
25. Which of the following is the least practical reason for allocating service department costs to user
departments? 
 

A.  To ascertain profitability of user departments.


B.  To evaluate performance of managers and divisions.
C.  To make user departments aware that services are costly.
D.  To provide the best possible service to users.

Allocation of services does not provide incentives for service levels.

 
AACSB: Analytical Thinking
AICPA: FN Measurement
Accessibility: Keyboard Navigation
Blooms: Remember
Difficulty: 1 Easy
Gradable: automatic
Learning Objective: 11-01 Explain why service costs are allocated.
Topic: Service Department Cost Allocation
 
26. Service department costs are: 
 

A.  generally treated as period costs rather than product costs.


B.  reported as selling and administrative expenses on the income statement.
C.  eventually applied by the user departments to the units produced.
D.  seldom found in manufacturing organizations.

Service department costs are considered product costs since they are part of the production process.

 
AACSB: Analytical Thinking
AICPA: FN Measurement
Accessibility: Keyboard Navigation
Blooms: Remember
Difficulty: 2 Medium
Gradable: automatic
Learning Objective: 11-01 Explain why service costs are allocated.
Topic: Service Department Cost Allocation
 

11-96
Copyright © 2017 McGraw-Hill Education. All rights reserved. No reproduction or distribution without the prior written consent of
McGraw-Hill Education.
27. A management purpose for allocating joint costs of a processing center to the various products produced
is to:  
 

A.  establish inventory values for unsold units.


B.  record accurate cost of sales by-product line.
C.  compute total processing cost variances by-product.
D.  report correct standard product costs for comparative analysis.

Service department costs are considered product costs since they are part of the production process.

 
AACSB: Analytical Thinking
AICPA: FN Measurement
Accessibility: Keyboard Navigation
Blooms: Remember
Difficulty: 1 Easy
Gradable: automatic
Learning Objective: 11-01 Explain why service costs are allocated.
Topic: Service Department Cost Allocation
 
28. Which of the following service departments could logically use space occupied (square footage) to
allocate its costs to user departments?  
 

A.  Material Handling.


B.  Cafeteria.
C.  Custodial Services.
D.  Cost Accounting.

Custodial services cleans space, so space occupied would be logical.

 
AACSB: Analytical Thinking
AICPA: FN Measurement
Accessibility: Keyboard Navigation
Blooms: Apply
Difficulty: 2 Medium
Gradable: automatic
Learning Objective: 11-01 Explain why service costs are allocated.
Topic: Methods of Allocating Service Department Costs
 

11-97
Copyright © 2017 McGraw-Hill Education. All rights reserved. No reproduction or distribution without the prior written consent of
McGraw-Hill Education.
29. Which of the following departments is not a service department in a typical manufacturing company? 
 

A.  Assembly.
B.  Accounting.
C.  Human resources.
D.  Information processing.

Assembly is a producing department, not a service department.

 
AACSB: Analytical Thinking
AICPA: FN Measurement
Accessibility: Keyboard Navigation
Blooms: Remember
Difficulty: 2 Medium
Gradable: automatic
Learning Objective: 11-01 Explain why service costs are allocated.
Topic: Service Department Cost Allocation
 
30. Criteria for selecting allocation bases for service department allocations should not include:  
 

A.  direct, traceable benefits from the service.


B.  the extent of facilities provided.
C.  the ease of making an allocation.
D.  sales dollars generated during the period.

Revenues are never a good base for allocating service costs.

 
AACSB: Analytical Thinking
AICPA: FN Measurement
Accessibility: Keyboard Navigation
Blooms: Remember
Difficulty: 2 Medium
Gradable: automatic
Learning Objective: 11-01 Explain why service costs are allocated.
Topic: Methods of Allocating Service Department Costs
 

11-98
Copyright © 2017 McGraw-Hill Education. All rights reserved. No reproduction or distribution without the prior written consent of
McGraw-Hill Education.
31. Dreamland University has 20 departments. Two of its best departments are the (1) College of
Innovation (COI) and (2) Testing Services. The College of Innovation (COI) attempts to teach students
the difficult, but useful, skill of innovation. Testing Services grades examinations for professors. How
would these two departments be classified?
 
College of Testing
 
Innovation Services
A. Service Service
B. User Service
C. User User
D. Service User
 
 

A.  Option A
B.  Option B
C.  Option C
D.  Option D

COI interacts directly with the consumer, while testing serves producing departments, not consumers.

 
AACSB: Analytical Thinking
AICPA: FN Measurement
Accessibility: Keyboard Navigation
Blooms: Remember
Difficulty: 1 Easy
Gradable: automatic
Learning Objective: 11-01 Explain why service costs are allocated.
Topic: Service Department Cost Allocation
 
32. Which of the following is not a reason to justify the allocation of support services? 
 

A.  Tax reporting requirements.


B.  Influencing behavior of employees.
C.  To trace costs to the activity that created the costs.
D.  Cost based contracts.

The purpose for selecting the appropriate allocation method is because of the strong relationship
between cause and effect.

 
AACSB: Reflective Thinking
AICPA: FN Decision Making
Accessibility: Keyboard Navigation
Blooms: Remember
Difficulty: 1 Easy
Gradable: automatic
Learning Objective: 11-01 Explain why service costs are allocated.
Topic: Service Department Cost Allocation
 

11-99
Copyright © 2017 McGraw-Hill Education. All rights reserved. No reproduction or distribution without the prior written consent of
McGraw-Hill Education.
33. Which of the following statements is(are) false regarding the direct method of allocating service
department costs?
(A) The selection of an allocation base in the direct method is easier than the selection of an allocation
base in the step method.
(B) Once an allocation is made from a service department using the direct method, no further
allocations are made back to that department.  
 

A.  Only A is false.


B.  Only B is false.
C.  Neither A nor B is false.
D.  Both A and B are false.

The selection of a base is no easier for the direct method than it is for the step method.

 
AACSB: Analytical Thinking
AICPA: FN Measurement
Accessibility: Keyboard Navigation
Blooms: Remember
Difficulty: 1 Easy
Gradable: automatic
Learning Objective: 11-02 Allocate service department costs using the direct method.
Topic: Methods of Allocating Service Department Costs
 
34. Jamison Company has two service departments and two producing departments. Square footage of
space occupied by each department follows:
 
Custodial services 1,000 ft
General administration 3,000 ft
Producing department A 8,000 ft
Producing department B  8,000 ft
  20,000 ft

The department costs of Custodial Services are allocated on a basis of square footage of space. If
Custodial Services costs are budgeted at $38,000, the amount of cost allocated to General
Administration under the direct method would be:  
 

A.  $0.
B.  $7,125.
C.  $6,000.
D.  $5,700.

$0. There are no allocations between service departments when using the direct method.

 
AACSB: Analytical Thinking
AICPA: FN Measurement
Accessibility: Keyboard Navigation
Blooms: Apply

11-100
Copyright © 2017 McGraw-Hill Education. All rights reserved. No reproduction or distribution without the prior written consent of
McGraw-Hill Education.
Difficulty: 2 Medium
Gradable: automatic
Learning Objective: 11-02 Allocate service department costs using the direct method.
Topic: Methods of Allocating Service Department Costs
 
35. Veneer Company has two service departments and two producing departments. The number of
employees in each department is:
 
Personnel 10
Cafeteria 25
Producing Department A 265
Producing Department B 250
  550

The department costs of the Personnel Department are allocated on a basis of the number of employees.
If these costs are budgeted at $37,125 during a given period, the amount of cost allocated to Department
B under the direct method would be:  
 

A.  $0.
B.  $17,187.50.
C.  $16,875.00.
D.  $18,021.84.

[250/(265 + 250)] × $37,125 = $18,021.84

 
AACSB: Analytical Thinking
AICPA: FN Measurement
Accessibility: Keyboard Navigation
Blooms: Apply
Difficulty: 1 Easy
Gradable: automatic
Learning Objective: 11-02 Allocate service department costs using the direct method.
Topic: Methods of Allocating Service Department Costs
 
36. Which of the following is not a benefit of cost allocation? 
 

A.  Instilling responsibility for all costs of the company in division managers.
B.  Constructing performance measures that may be more meaningful than contribution margins.
C.  Relating indirect costs to contracts, jobs and products.
D.  Additional bookkeeping costs incurred to provide cost allocation information.

The additional cost of the appropriate allocation of costs to objects is exceeded by the benefits derived.

 
AACSB: Reflective Thinking
AICPA: FN Measurement
Accessibility: Keyboard Navigation
Blooms: Remember
Difficulty: 1 Easy
Gradable: automatic

11-101
Copyright © 2017 McGraw-Hill Education. All rights reserved. No reproduction or distribution without the prior written consent of
McGraw-Hill Education.
Learning Objective: 11-02 Allocate service department costs using the direct method.
Topic: Methods of Allocating Service Department Costs
 
37. Tenet Engineering, Inc. operates two user divisions as separate cost objects. To determine the costs of
each division, the company allocates common costs to the divisions. During the past month, the
following common costs were incurred:
 
Computer services (85% fixed) $260,000
Building occupancy 600,000
Personnel costs  110,000
Total common costs $970,000

The following information is available concerning various activity measures and service usages by each
of the divisions:
 
Division
  Division B
A
Area occupied (square feet) 20,000 40,000
Payroll $380,000 $180,000
Computer time (hours) 200 220
Computer storage (megabytes) 4,050 -0-
Equipment value $200,000 $250,000
Operating profit (pre-
$555,000 $495,000
allocations)

If common computer service costs are allocated using computer time as the allocation basis, what is the
computer cost allocated to Division B?  
 

A.  $136,190.
B.  $137,647.
C.  $144,444.
D.  $173,333.

[$260,000 × (220/420)] = $136,190

 
AACSB: Analytical Thinking
AICPA: FN Measurement
Accessibility: Keyboard Navigation
Blooms: Apply
Difficulty: 1 Easy
Gradable: automatic
Learning Objective: 11-02 Allocate service department costs using the direct method.
Topic: Methods of Allocating Service Department Costs
 

11-102
Copyright © 2017 McGraw-Hill Education. All rights reserved. No reproduction or distribution without the prior written consent of
McGraw-Hill Education.
38. Tenet Engineering, Inc. operates two user divisions as separate cost objects. To determine the costs of
each division, the company allocates common costs to the divisions. During the past month, the
following common costs were incurred:
 
Computer services (85% fixed) $260,000
Building occupancy 600,000
Personnel costs  110,000
Total common costs $970,000

The following information is available concerning various activity measures and service usages by each
of the divisions:
 
Division
  Division B
A
Area occupied (square feet) 20,000 40,000
Payroll $380,000 $180,000
Computer time (hours) 200 220
Computer storage (megabytes) 4,050 -0-
Equipment value $200,000 $250,000
Operating profit (pre-
$555,000 $495,000
allocations)

Using the most appropriate allocation basis, what is the personnel cost allocated to Division A?  
 

A.  $58,143.
B.  $74,643.
C.  $76,463.
D.  $110,000.

Payroll Expense. [$110,000 × ($380,000/560,000)] = $74,643

 
AACSB: Analytical Thinking
AICPA: FN Measurement
Accessibility: Keyboard Navigation
Blooms: Apply
Difficulty: 1 Easy
Gradable: automatic
Learning Objective: 11-02 Allocate service department costs using the direct method.
Topic: Methods of Allocating Service Department Costs
 

11-103
Copyright © 2017 McGraw-Hill Education. All rights reserved. No reproduction or distribution without the prior written consent of
McGraw-Hill Education.
39. Tenet Engineering, Inc. operates two user divisions as separate cost objects. To determine the costs of
each division, the company allocates common costs to the divisions. During the past month, the
following common costs were incurred:
 
Computer services (85% fixed) $260,000
Building occupancy 600,000
Personnel costs  110,000
Total common costs $970,000

The following information is available concerning various activity measures and service usages by each
of the divisions:
 
Division
  Division B
A
Area occupied (square feet) 20,000 40,000
Payroll $380,000 $180,000
Computer time (hours) 200 220
Computer storage (megabytes) 4,050 -0-
Equipment value $200,000 $250,000
Operating profit (pre-
$555,000 $495,000
allocations)

If all common costs are allocated using operating profit as the allocation basis, what is the total cost
allocated to Division B?  
 

A.  $457,286.
B.  $512,714.
C.  $555,000.
D.  $1,087,576.

$970,000 × ($495,000/1,050,000) = $457,286

 
AACSB: Analytical Thinking
AICPA: FN Measurement
Accessibility: Keyboard Navigation
Blooms: Apply
Difficulty: 2 Medium
Gradable: automatic
Learning Objective: 11-02 Allocate service department costs using the direct method.
Topic: Methods of Allocating Service Department Costs
 

11-104
Copyright © 2017 McGraw-Hill Education. All rights reserved. No reproduction or distribution without the prior written consent of
McGraw-Hill Education.
40. Which of the following methods provides no data for service departments to monitor each other's
costs? 
 

A.  Direct method.


B.  Reciprocal method.
C.  Step method.
D.  All three methods, Direct, Reciprocal, and Step, provide data for monitoring costs.

No reciprocal services are recognized.

 
AACSB: Analytical Thinking
AICPA: FN Measurement
Accessibility: Keyboard Navigation
Blooms: Apply
Difficulty: 1 Easy
Gradable: automatic
Learning Objective: 11-02 Allocate service department costs using the direct method.
Topic: Methods of Allocating Service Department Costs
 
41. The Maryville Construction Company occupies 85,000 square feet for construction of mobile homes.
There are two manufacturing departments, finishing and assembly, and four service departments labeled
S1, S2, S3, and S4. Information relevant to Maryville is as follows:
 
Allocatio
Area n
Dept S1 S2 S3 S4 Assembly
used Finishing
S1 17,000 --- .10 .20 --- .20 .50
S2 4,250 --- --- .30 .30 --- .40
S3 8,500 .20 .20 --- .30 .20 .10
S4 4,250 .30 .10 .30 --- .20 .10
Fin. 21,250 --- --- --- --- --- ---
Asm. 29,750 --- --- --- --- --- ---

Rent paid for the area used is $720,000.

How much rent is allocable to the assembly department using the direct method of allocation?  
 

A.  $420,000.
B.  $332,500.
C.  $300,000.
D.  $252,000.

[29,750/(21,250 + 29,750)] × $720,000 = $420,000

 
AACSB: Analytical Thinking
AICPA: FN Measurement
Accessibility: Keyboard Navigation

11-105
Copyright © 2017 McGraw-Hill Education. All rights reserved. No reproduction or distribution without the prior written consent of
McGraw-Hill Education.
Blooms: Apply
Difficulty: 2 Medium
Gradable: automatic
Learning Objective: 11-02 Allocate service department costs using the direct method.
Topic: Methods of Allocating Service Department Costs
 
42. The Maryville Construction Company occupies 85,000 square feet for construction of mobile homes.
There are two manufacturing departments, finishing and assembly, and four service departments labeled
S1, S2, S3, and S4. Information relevant to Maryville is as follows:
 
Allocatio
Area n
Dept S1 S2 S3 S4 Assembly
used Finishing
S1 17,000 --- .10 .20 --- .20 .50
S2 4,250 --- --- .30 .30 --- .40
S3 8,500 .20 .20 --- .30 .20 .10
S4 4,250 .30 .10 .30 --- .20 .10
Fin. 21,250 --- --- --- --- --- ---
Asm. 29,750 --- --- --- --- --- ---

Rent paid for the area used is $720,000.

How much rent would be charged to S4 using the step method of allocation and a S3-S4-S1-S2
sequence for the allocations?  
 

A.  $36,000.
B.  $40,000.
C.  $54,000.
D.  $90,000.

(4,250/76,500) × $720,000 = $40,000

 
AACSB: Analytical Thinking
AICPA: FN Measurement
Accessibility: Keyboard Navigation
Blooms: Apply
Difficulty: 2 Medium
Gradable: automatic
Learning Objective: 11-03 Allocate service department costs using the step method.
Topic: Methods of Allocating Service Department Costs
 

11-106
Copyright © 2017 McGraw-Hill Education. All rights reserved. No reproduction or distribution without the prior written consent of
McGraw-Hill Education.
43. If two service departments service the same number of departments, which service department's costs
should be allocated first when using the step method? 
 

A.  The service department that provides the most service to other service departments.
B.  The service department that provides the most service to the user departments.
C.  The service department with the least cost.
D.  The service department that provides the least service to other service departments.

This will minimize the percentage of services ignored in the allocation process.

 
AACSB: Analytical Thinking
AICPA: FN Measurement
Accessibility: Keyboard Navigation
Blooms: Apply
Difficulty: 2 Medium
Gradable: automatic
Learning Objective: 11-03 Allocate service department costs using the step method.
Topic: Methods of Allocating Service Department Costs
 
44. Which of the following is a weakness of the step method of service cost allocations?  
 

A.  Computations are more complex than the reciprocal method.


B.  All interdepartmental services are ignored.
C.  All intradepartmental services are ignored.
D.  The order of service department allocation has to be determined.

Computations are easier than the reciprocal method and some services are ignored.

 
AACSB: Analytical Thinking
AICPA: FN Measurement
Accessibility: Keyboard Navigation
Blooms: Remember
Difficulty: 2 Medium
Gradable: automatic
Learning Objective: 11-03 Allocate service department costs using the step method.
Topic: Methods of Allocating Service Department Costs
 

11-107
Copyright © 2017 McGraw-Hill Education. All rights reserved. No reproduction or distribution without the prior written consent of
McGraw-Hill Education.
45. The Hsu Manufacturing Company has two service departments: Maintenance and Accounting. The
Maintenance Department's costs of $300,000 are allocated on the basis of machine hours. The
Accounting Department's costs of $120,000 are allocated on the basis of the number of employees
within a specific department. The direct departmental costs for A and B are $300,000 and $500,000,
respectively.
 
  Maint Acctg A B
Machine hours 480 20 2,300 200
Number of employees 2 2 8 4

What is the Maintenance Department's cost allocated to Department A using the direct method?  
 

A.  $92,000.
B.  $230,000.
C.  $276,000.
D.  $386,400.

[$300,000/(2,300 + 200)] × 2,300 = $276,000

 
AACSB: Analytical Thinking
AICPA: FN Measurement
Accessibility: Keyboard Navigation
Blooms: Apply
Difficulty: 2 Medium
Gradable: automatic
Learning Objective: 11-02 Allocate service department costs using the direct method.
Topic: Methods of Allocating Service Department Costs
 

11-108
Copyright © 2017 McGraw-Hill Education. All rights reserved. No reproduction or distribution without the prior written consent of
McGraw-Hill Education.
46. The Hsu Manufacturing Company has two service departments: Maintenance and Accounting. The
Maintenance Department's costs of $300,000 are allocated on the basis of machine hours. The
Accounting Department's costs of $120,000 are allocated on the basis of the number of employees
within a specific department. The direct departmental costs for A and B are $300,000 and $500,000,
respectively.
 
  Maint Acctg A B
Machine hours 480 20 2,300 200
Number of employees 2 2 8 4

What is the Accounting Department's cost allocated to Department B using the direct method?  
 

A.  $40,000.
B.  $80,000.
C.  $20,000.
D.  $10,000.

[$120,000/(8 + 4)] × 4 = $40,000

 
AACSB: Analytical Thinking
AICPA: FN Measurement
Accessibility: Keyboard Navigation
Blooms: Apply
Difficulty: 2 Medium
Gradable: automatic
Learning Objective: 11-02 Allocate service department costs using the direct method.
Topic: Methods of Allocating Service Department Costs
 

11-109
Copyright © 2017 McGraw-Hill Education. All rights reserved. No reproduction or distribution without the prior written consent of
McGraw-Hill Education.
47. The Hsu Manufacturing Company has two service departments: Maintenance and Accounting. The
Maintenance Department's costs of $300,000 are allocated on the basis of machine hours. The
Accounting Department's costs of $120,000 are allocated on the basis of the number of employees
within a specific department. The direct departmental costs for A and B are $300,000 and $500,000,
respectively.
 
  Maint Acctg A B
Machine hours 480 20 2,300 200
Number of employees 2 2 8 4

What is the Maintenance Department's cost allocated to Department B using the step method and
assuming the Maintenance Department's costs are allocated first?  
 

A.  $276,000.
B.  $230,000.
C.  $322,000.
D.  $23,810.

[200/(20 + 2,300 + 200)] × $300,000 = $23,809.52, round to $23,810.

 
AACSB: Analytical Thinking
AICPA: FN Measurement
Accessibility: Keyboard Navigation
Blooms: Apply
Difficulty: 3 Hard
Gradable: automatic
Learning Objective: 11-03 Allocate service department costs using the step method.
Topic: Methods of Allocating Service Department Costs
 

11-110
Copyright © 2017 McGraw-Hill Education. All rights reserved. No reproduction or distribution without the prior written consent of
McGraw-Hill Education.
48. The Hsu Manufacturing Company has two service departments: Maintenance and Accounting. The
Maintenance Department's costs of $300,000 are allocated on the basis of machine hours. The
Accounting Department's costs of $120,000 are allocated on the basis of the number of employees
within a specific department. The direct departmental costs for A and B are $300,000 and $500,000,
respectively.
 
  Maint Acctg A B
Machine hours 480 20 2,300 200
Number of employees 2 2 8 4

What is the cost of the Accounting Department's cost allocated to Department A using the step method
and assuming the Maintenance Department's costs are allocated first?  
 

A.  $81,333.
B.  $81,587.
C.  $80,000.
D.  $68,571.

($300,000/2,520) × 20 = $2,381; [($120,000 + 2,381)/(8 + 4)] × 8 = $81,587

 
AACSB: Analytical Thinking
AICPA: FN Measurement
Accessibility: Keyboard Navigation
Blooms: Apply
Difficulty: 2 Medium
Gradable: automatic
Learning Objective: 11-03 Allocate service department costs using the step method.
Topic: Methods of Allocating Service Department Costs
 

11-111
Copyright © 2017 McGraw-Hill Education. All rights reserved. No reproduction or distribution without the prior written consent of
McGraw-Hill Education.
49. Steven Parker owns and operates Steven's Septic Service and Legal Advice. Steven's two revenue
generating (production) operations are supported by two service departments: Clerical and Janitorial.
Costs in the service departments are allocated in the following order using the designated allocation
bases:

Clerical:

Variable cost: expected number of work orders processed


Fixed cost: long-run average number of work orders processed

Janitorial:

Variable cost: labor hours


Fixed cost: square footage of space occupied
Average and expected activity levels for next month (June) are as follows:
 
No. of Work
Labor Square
  Orders
Average Hours Footage
Expected
Septic
50 80 560 1,800
Service
Legal advice 25 20 840 2,200
Clerical 20 20 400 1,600
Janitorial 5 20 200 1,000

Expected costs in the service departments for June are as follows:


 
  Clerical Janitorial
Variable costs $12,000 $4,200
Fixed costs $8,400 $800

Under the step method of allocation, how much Clerical service cost should be allocated to the Septic
Service operation for June? (Assume Clerical costs are allocated before Janitorial costs and round all
calculations to the nearest whole dollar.)  
 

A.  $12,689.
B.  $13,100.
C.  $13,620.
D.  $15,596.

[(50/80) × $12,000] + [(80/120) × $8,400] = $13,100

 
AACSB: Analytical Thinking
AICPA: FN Measurement
Accessibility: Keyboard Navigation
Blooms: Apply
Difficulty: 3 Hard

11-112
Copyright © 2017 McGraw-Hill Education. All rights reserved. No reproduction or distribution without the prior written consent of
McGraw-Hill Education.
Gradable: automatic
Learning Objective: 11-03 Allocate service department costs using the step method.
Topic: Methods of Allocating Service Department Costs
 
50. Steven Parker owns and operates Steven's Septic Service and Legal Advice. Steven's two revenue
generating (production) operations are supported by two service departments: Clerical and Janitorial.
Costs in the service departments are allocated in the following order using the designated allocation
bases:

Clerical:

Variable cost: expected number of work orders processed


Fixed cost: long-run average number of work orders processed

Janitorial:

Variable cost: labor hours


Fixed cost: square footage of space occupied
Average and expected activity levels for next month (June) are as follows:
 
No. of Work
Labor Square
  Orders
Average Hours Footage
Expected
Septic
50 80 560 1,800
Service
Legal advice 25 20 840 2,200
Clerical 20 20 400 1,600
Janitorial 5 20 200 1,000

Expected costs in the service departments for June are as follows:


 
  Clerical Janitorial
Variable costs $12,000 $4,200
Fixed costs $8,400 $800

Under the direct method of allocation, what is the total amount of service cost allocated to the Legal
Advice operation for June? (Round all calculations to the nearest whole dollar.)  
 

A.  $6,231.
B.  $7,720.
C.  $8,640.
D.  $9,330.

[(25/75) × $12,000] + [(20/100) × $8,400] + [(840/1400) × $4,200] + [(2200/4000) × $800] = $8,640

 
AACSB: Analytical Thinking
AICPA: FN Measurement
Accessibility: Keyboard Navigation

11-113
Copyright © 2017 McGraw-Hill Education. All rights reserved. No reproduction or distribution without the prior written consent of
McGraw-Hill Education.
Blooms: Apply
Difficulty: 2 Medium
Gradable: automatic
Learning Objective: 11-02 Allocate service department costs using the direct method.
Topic: Methods of Allocating Service Department Costs
 
51. Cordner Corporation has two production Departments: P1 and P2 and two service departments: S1 and
S2. Direct costs for each department and the proportion of service costs used by the various departments
for the month of July are as follows:
 
Proportion of Services Used
   
by:
Direct
Department S1 S2 P1 P2
costs
S1 $60,000   0.70 0.10 0.20
S2 $100,000 0.20   0.30 0.50
P1 $160,000        
P2 $140,000        

Under the direct-method of cost allocation, the amount of S1 costs allocated to the S2 would be:  
 

A.  $42,000.
B.  $20,000.
C.  $0.
D.  $6,000.

Under the direct method, service department costs are not allocated to other service departments, only
production departments.

 
AACSB: Reflective Thinking
AICPA: FN Measurement
Accessibility: Keyboard Navigation
Blooms: Apply
Difficulty: 2 Medium
Gradable: automatic
Learning Objective: 11-02 Allocate service department costs using the direct method.
Topic: Methods of Allocating Service Department Costs
 

11-114
Copyright © 2017 McGraw-Hill Education. All rights reserved. No reproduction or distribution without the prior written consent of
McGraw-Hill Education.
52. Cordner Corporation has two production Departments: P1 and P2 and two service departments: S1 and
S2. Direct costs for each department and the proportion of service costs used by the various departments
for the month of July are as follows:
 
Proportion of Services Used
   
by:
Direct
Department S1 S2 P1 P2
costs
S1 $60,000   0.70 0.10 0.20
S2 $100,000 0.20   0.30 0.50
P1 $160,000        
P2 $140,000        

Under the direct-method of cost allocation, the amount of S1 costs allocated to P1 would be:  
 

A.  $20,000.
B.  $6,000.
C.  $30,000.
D.  $62,500.

.10/.30 × $60,000 = $20,000

 
AACSB: Reflective Thinking
AICPA: FN Measurement
Accessibility: Keyboard Navigation
Blooms: Apply
Difficulty: 2 Medium
Gradable: automatic
Learning Objective: 11-02 Allocate service department costs using the direct method.
Topic: Methods of Allocating Service Department Costs
 

11-115
Copyright © 2017 McGraw-Hill Education. All rights reserved. No reproduction or distribution without the prior written consent of
McGraw-Hill Education.
53. Cordner Corporation has two production Departments: P1 and P2 and two service departments: S1 and
S2. Direct costs for each department and the proportion of service costs used by the various departments
for the month of July are as follows:
 
Proportion of Services Used
   
by:
Direct
Department S1 S2 P1 P2
costs
S1 $60,000   0.70 0.10 0.20
S2 $100,000 0.20   0.30 0.50
P1 $160,000        
P2 $140,000        

Under the step-method of cost allocation, the amount of S2 costs allocated to S1 would be:  
 

A.  $40,000.
B.  $20,000.
C.  $0.
D.  $42,000.

The correct order of allocation is S1 to S2 and then to P1/P2. S2 to S1 is not the correct process.

 
AACSB: Reflective Thinking
AICPA: FN Measurement
Accessibility: Keyboard Navigation
Blooms: Apply
Difficulty: 2 Medium
Gradable: automatic
Learning Objective: 11-03 Allocate service department costs using the step method.
Topic: Methods of Allocating Service Department Costs
 

11-116
Copyright © 2017 McGraw-Hill Education. All rights reserved. No reproduction or distribution without the prior written consent of
McGraw-Hill Education.
54. Cordner Corporation has two production Departments: P1 and P2 and two service departments: S1 and
S2. Direct costs for each department and the proportion of service costs used by the various departments
for the month of July are as follows:
 
Proportion of Services Used
   
by:
Direct
Department S1 S2 P1 P2
costs
S1 $60,000   0.70 0.10 0.20
S2 $100,000 0.20   0.30 0.50
P1 $160,000        
P2 $140,000        

Under the step-method of cost allocation, the amount of costs allocated from S2 to P2 would be:  
 

A.  $88,750.
B.  $50,000.
C.  $62,500.
D.  $53,250.

See calculation below.

S1’s $60,000 is allocated 70% to S2 or $42,000


S2’s total is $100,000 + $42,000 = $142,000
S2’s $142,000 is allocated 62.5% to P2 or $88,750

 
AACSB: Reflective Thinking
AICPA: FN Measurement
Accessibility: Keyboard Navigation
Blooms: Apply
Difficulty: 2 Medium
Gradable: automatic
Learning Objective: 11-03 Allocate service department costs using the step method.
Topic: Methods of Allocating Service Department Costs
 

11-117
Copyright © 2017 McGraw-Hill Education. All rights reserved. No reproduction or distribution without the prior written consent of
McGraw-Hill Education.
55. Cordner Corporation has two production Departments: P1 and P2 and two service departments: S1 and
S2. Direct costs for each department and the proportion of service costs used by the various departments
for the month of July are as follows:
 
Proportion of Services Used
   
by:
Direct
Department S1 S2 P1 P2
costs
S1 $60,000   0.70 0.10 0.20
S2 $100,000 0.20   0.30 0.50
P1 $160,000        
P2 $140,000        

Under the step-method of allocation, the total amount of service costs allocated to producing
departments would be:  
 

A.  $118,000.
B.  $160,000.
C.  $140,000.
D.  $40,000.

The total costs of $60,000 and $100,000 ($160,000) will be allocated to the producing departments.

 
AACSB: Reflective Thinking
AICPA: FN Measurement
Accessibility: Keyboard Navigation
Blooms: Apply
Difficulty: 1 Easy
Gradable: automatic
Learning Objective: 11-03 Allocate service department costs using the step method.
Topic: Methods of Allocating Service Department Costs
 

11-118
Copyright © 2017 McGraw-Hill Education. All rights reserved. No reproduction or distribution without the prior written consent of
McGraw-Hill Education.
56. Palace Company has two service departments and two user departments. The number of employees in
each department is:
 
Personnel 10
Cafeteria 25
Producing Department A 265
Producing Department B 250
  550

The fixed costs of the Personnel Department are allocated on a basis of the number of employees. If
these costs are budgeted at $37,125 during a given period, the amount of cost allocated to the Cafeteria
under the step method would be:  
 

A.  $0.
B.  $1,718.75.
C.  $1,687.50.
D.  $1,802.18.

[25/(25 + 265 + 250)] × $37,125 = $1,718.75

 
AACSB: Analytical Thinking
AICPA: FN Measurement
Accessibility: Keyboard Navigation
Blooms: Apply
Difficulty: 2 Medium
Gradable: automatic
Learning Objective: 11-03 Allocate service department costs using the step method.
Topic: Methods of Allocating Service Department Costs
 
57. There are several methods for allocating service department costs to production departments. The
method which recognizes service provided by one service department to another but does not recognize
reciprocal interdepartmental service is called: (CMA adapted)  
 

A.  direct method.


B.  variable method.
C.  linear method.
D.  step-down method.

Step-down is another term for step method.

 
AACSB: Analytical Thinking
AICPA: FN Measurement
Accessibility: Keyboard Navigation
Blooms: Remember
Difficulty: 1 Easy
Gradable: automatic
Learning Objective: 11-03 Allocate service department costs using the step method.
Topic: Methods of Allocating Service Department Costs

11-119
Copyright © 2017 McGraw-Hill Education. All rights reserved. No reproduction or distribution without the prior written consent of
McGraw-Hill Education.
 
58. Because this allocation method recognizes that service departments often provide each other with inter-
departmental service, it is theoretically considered to be the most accurate method for allocating service
department costs to production departments. This method is: (CMA adapted) 
 

A.  direct method.


B.  variable method.
C.  linear method.
D.  reciprocal method.

Reciprocal is considered most accurate because it includes all reciprocal services.

 
AACSB: Analytical Thinking
AICPA: FN Measurement
Accessibility: Keyboard Navigation
Blooms: Remember
Difficulty: 1 Easy
Gradable: automatic
Learning Objective: 11-04 Allocate service department costs using the reciprocal method.
Topic: Methods of Allocating Service Department Costs
 

11-120
Copyright © 2017 McGraw-Hill Education. All rights reserved. No reproduction or distribution without the prior written consent of
McGraw-Hill Education.
59. The following information relates to Osceola Corporation for the past accounting period.
 
  Direct costs
Service Dept A $80,000
Service Dept B 60,000
Producing Dept C 15,000
Producing Dept D 20,000

Proportion of service by Proportion of service by B


A to: to:
B 10% A 30%
C 60% C 20%
D 30% D 50%

Using the simultaneous solution method, Department A's cost allocated to Department C is:  
 

A.  $48,000.
B.  $58,800.
C.  $60,619.
D.  $98,000.

See calculation below.

A = $80,000 + .30B; B = $60,000 + .10A


A = $101,031; B = $70,103
C's portion of A = .60 × $101,031 = $60,619

 
AACSB: Analytical Thinking
AICPA: FN Measurement
Accessibility: Keyboard Navigation
Blooms: Apply
Difficulty: 3 Hard
Gradable: automatic
Learning Objective: 11-04 Allocate service department costs using the reciprocal method.
Topic: Methods of Allocating Service Department Costs
 

11-121
Copyright © 2017 McGraw-Hill Education. All rights reserved. No reproduction or distribution without the prior written consent of
McGraw-Hill Education.
60. The following information relates to Osceola Corporation for the past accounting period.
 
  Direct costs
Service Dept A $80,000
Service Dept B 60,000
Producing Dept C 15,000
Producing Dept D 20,000

Proportion of service by Proportion of service by B


A to: to:
B 10% A 30%
C 60% C 20%
D 30% D 50%

Using the simultaneous solution method, Department B's cost allocated to Department C is:  
 

A.  $29,021
B.  $14,021
C.  $13,192
D.  $7,794

See calculation below.

A = $80,000 + .30B; B = $60,000 + .10A


A = $101,031; B = $70,103
C's portion of B = .20 × $70,103 = $14,021

 
AACSB: Analytical Thinking
AICPA: FN Measurement
Accessibility: Keyboard Navigation
Blooms: Apply
Difficulty: 3 Hard
Gradable: automatic
Learning Objective: 11-04 Allocate service department costs using the reciprocal method.
Topic: Methods of Allocating Service Department Costs
 

11-122
Copyright © 2017 McGraw-Hill Education. All rights reserved. No reproduction or distribution without the prior written consent of
McGraw-Hill Education.
61. The following set up is a system of simultaneous linear equations to allocate costs using the reciprocal
method. Matrix algebra is not required.
The following costs were incurred in three operating departments and three service departments in
Westmoreland Company.
 
Department Direct Costs Label
Subassemblies $550,000 P1
Final assembly 775,000 P2
Marketing 285,000 P3
Building occupancy 85,000 S1
Research&development 120,000 S2
Supervision 45,000 S3

Use of services by other departments is as follows.


 
                       User Department

Service Cost Sub- Building


Final Assembly Marketin R&
Center assemblies Occupancy Supervision
g D
Bldg.occupancy .30 .25 .20 --- .15 .10
R&D .50 .50 --- --- --- ---
Supervision .20 .30 .20 .10 .20 ---

The equation for department P1 (subassemblies) is:  


 

A.  P1 = $550,000 + .25P2 + .20P3 + .15S2 + 10S3.


B.  P1 = $550,000 + .30S1 + .50S2 + .20S3.
C.  P1 = .30S1 + .50S2 + 20S3.
D.  P1 = .30S1 + .50S.

The direct cost for subassemblies is $550,000 and it uses 50% of R&D.

 
AACSB: Analytical Thinking
AICPA: FN Measurement
Accessibility: Keyboard Navigation
Blooms: Apply
Difficulty: 2 Medium
Gradable: automatic
Learning Objective: 11-04 Allocate service department costs using the reciprocal method.
Topic: Methods of Allocating Service Department Costs
 

11-123
Copyright © 2017 McGraw-Hill Education. All rights reserved. No reproduction or distribution without the prior written consent of
McGraw-Hill Education.
62. The following set up is a system of simultaneous linear equations to allocate costs using the reciprocal
method. Matrix algebra is not required.
The following costs were incurred in three operating departments and three service departments in
Westmoreland Company.
 
Department Direct Costs Label
Subassemblies $550,000 P1
Final assembly 775,000 P2
Marketing 285,000 P3
Building occupancy 85,000 S1
Research&development 120,000 S2
Supervision 45,000 S3

Use of services by other departments is as follows.


 
                       User Department

Service Cost Sub- Building


Final Assembly Marketin R&
Center assemblies Occupancy Supervision
g D
Bldg.occupancy .30 .25 .20 --- .15 .10
R&D .50 .50 --- --- --- ---
Supervision .20 .30 .20 .10 .20 ---

The equation for department P2 (final assembly) is:  


 

A.  P2 = .25S1 + .50S2 + .30S3.


B.  P2 = $775,000 + .25P2 + .20P3 + .15S2 + .10S3.
C.  P2 = $775,000 + .30S1 + .50S2 + .20S3.
D.  P2 = $775,000 + .25S1 + .50S2 + .30S3.

The direct cost for final assembly is $775,000 and it uses 25% of bldg occupancy and 50% of R&D and
3% of supervision.

 
AACSB: Analytical Thinking
AICPA: FN Measurement
Accessibility: Keyboard Navigation
Blooms: Apply
Difficulty: 2 Medium
Gradable: automatic
Learning Objective: 11-04 Allocate service department costs using the reciprocal method.
Topic: Methods of Allocating Service Department Costs
 

11-124
Copyright © 2017 McGraw-Hill Education. All rights reserved. No reproduction or distribution without the prior written consent of
McGraw-Hill Education.
63. The following set up is a system of simultaneous linear equations to allocate costs using the reciprocal
method. Matrix algebra is not required.
The following costs were incurred in three operating departments and three service departments in
Westmoreland Company.
 
Department Direct Costs Label
Subassemblies $550,000 P1
Final assembly 775,000 P2
Marketing 285,000 P3
Building occupancy 85,000 S1
Research&development 120,000 S2
Supervision 45,000 S3

Use of services by other departments is as follows.


 
                       User Department

Service Cost Sub- Building


Final Assembly Marketin R&
Center assemblies Occupancy Supervision
g D
Bldg.occupancy .30 .25 .20 --- .15 .10
R&D .50 .50 --- --- --- ---
Supervision .20 .30 .20 .10 .20 ---

The equation for department P3 (marketing) is:  


 

A.  P3 = $285,000 + .20S1 + .20S3.


B.  P3 = $285,000 + .20S1 + .60S2 + .20S3.
C.  P3 = $285,000 + .20S1 + .20S2 + .60S3.
D.  P3 = $285,000 + .50S1 + .50S3.

There is no R&D usage by marketing; 20% of bldg occupancy is used.

 
AACSB: Analytical Thinking
AICPA: FN Decision Making
Accessibility: Keyboard Navigation
Blooms: Apply
Difficulty: 2 Medium
Gradable: automatic
Learning Objective: 11-04 Allocate service department costs using the reciprocal method.
Topic: Methods of Allocating Service Department Costs
 

11-125
Copyright © 2017 McGraw-Hill Education. All rights reserved. No reproduction or distribution without the prior written consent of
McGraw-Hill Education.
64. The following set up is a system of simultaneous linear equations to allocate costs using the reciprocal
method. Matrix algebra is not required.
The following costs were incurred in three operating departments and three service departments in
Westmoreland Company.
 
Department Direct Costs Label
Subassemblies $550,000 P1
Final assembly 775,000 P2
Marketing 285,000 P3
Building occupancy 85,000 S1
Research&development 120,000 S2
Supervision 45,000 S3

Use of services by other departments is as follows.


 
                       User Department

Service Cost Sub- Building


Final Assembly Marketin R&
Center assemblies Occupancy Supervision
g D
Bldg.occupancy .30 .25 .20 --- .15 .10
R&D .50 .50 --- --- --- ---
Supervision .20 .30 .20 .10 .20 ---

The equation for department S1 (building occupancy) is:  


 

A.  S1 = .10S3.
B.  S1 = $85,000 + 1.00S3.
C.  S1 = $85,000 + .10S3.
D.  S1 = $85,000 + .90S2 + .10S3.

Bldg occupancy has direct costs of $85,000 and only uses supervision.

 
AACSB: Analytical Thinking
AICPA: FN Measurement
Accessibility: Keyboard Navigation
Blooms: Apply
Difficulty: 2 Medium
Gradable: automatic
Learning Objective: 11-04 Allocate service department costs using the reciprocal method.
Topic: Methods of Allocating Service Department Costs
 

11-126
Copyright © 2017 McGraw-Hill Education. All rights reserved. No reproduction or distribution without the prior written consent of
McGraw-Hill Education.
65. The following set up is a system of simultaneous linear equations to allocate costs using the reciprocal
method. Matrix algebra is not required.
The following costs were incurred in three operating departments and three service departments in
Westmoreland Company.
 
Department Direct Costs Label
Subassemblies $550,000 P1
Final assembly 775,000 P2
Marketing 285,000 P3
Building occupancy 85,000 S1
Research&development 120,000 S2
Supervision 45,000 S3

Use of services by other departments is as follows.


 
                       User Department

Service Cost Sub- Building


Final Assembly Marketin R&
Center assemblies Occupancy Supervision
g D
Bldg.occupancy .30 .25 .20 --- .15 .10
R&D .50 .50 --- --- --- ---
Supervision .20 .30 .20 .10 .20 ---

The equation for department S2 (research and development) is:  


 

A.  S2 = $120,000 + .15S1 + .65S2 + .20S3.


B.  S2 = .15S1 + 20S3.
C.  S2 = $120,000 + .15S1 + .20S3.
D.  S2 = $120,000 + .40S1 + .60S3.

R&D has direct costs of $120,000 and uses 15% of bldg occupancy and 20% of supervision.

 
AACSB: Analytical Thinking
AICPA: FN Measurement
Accessibility: Keyboard Navigation
Blooms: Apply
Difficulty: 2 Medium
Gradable: automatic
Learning Objective: 11-04 Allocate service department costs using the reciprocal method.
Topic: Methods of Allocating Service Department Costs
 

11-127
Copyright © 2017 McGraw-Hill Education. All rights reserved. No reproduction or distribution without the prior written consent of
McGraw-Hill Education.
66. The following set up is a system of simultaneous linear equations to allocate costs using the reciprocal
method. Matrix algebra is not required.
The following costs were incurred in three operating departments and three service departments in
Westmoreland Company.
 
Department Direct Costs Label
Subassemblies $550,000 P1
Final assembly 775,000 P2
Marketing 285,000 P3
Building occupancy 85,000 S1
Research&development 120,000 S2
Supervision 45,000 S3

Use of services by other departments is as follows.


 
                       User Department

Service Cost Sub- Building


Final Assembly Marketin R&
Center assemblies Occupancy Supervision
g D
Bldg.occupancy .30 .25 .20 --- .15 .10
R&D .50 .50 --- --- --- ---
Supervision .20 .30 .20 .10 .20 ---

The equation for department S3 (supervision) is:  


 

A.  S3 = $45,000 + .90S1 + .10S2.


B.  S3 = $45,000 + .10S1.
C.  S3 = $45,000 + 1.00S1.
D.  S3 = .10S1.

Supervision has $45,000 in direct costs and 10% of bldg occupancy.

 
AACSB: Analytical Thinking
AICPA: FN Measurement
Accessibility: Keyboard Navigation
Blooms: Apply
Difficulty: 2 Medium
Gradable: automatic
Learning Objective: 11-04 Allocate service department costs using the reciprocal method.
Topic: Methods of Allocating Service Department Costs
 

11-128
Copyright © 2017 McGraw-Hill Education. All rights reserved. No reproduction or distribution without the prior written consent of
McGraw-Hill Education.
67. Advanced Computer Solutions, Inc. has two main services: (1) time on a timeshared computer system,
and (2) proprietary computer programs. Computer time is provided by the operation department (Op)
and programs are written by the programming department (P).

The percentage of each service used by each department for a typical period is:

  Supplied
User Op P
Op --- 40%
P 30% ---
Sold to customers 70% 60%

In a typical period, the operation department (Op) spends $4,500 and the programming department (P)
spends $2,500.

Under the step method (Op first), what is the cost of the computer time and the computer programs for
sale?
 
  Time Programs
A. $4,500 $2,500
B. $3,150 $3,850
C. $1,350 $5,650
D. $2,700 $4,300
 
 

A.  Option A
B.  Option B
C.  Option C
D.  Option D

Time = .70($4,500) = $3,150; Programs = $2,500 + .30($4,500) = $3,850

 
AACSB: Analytical Thinking
AICPA: FN Measurement
Accessibility: Keyboard Navigation
Blooms: Apply
Difficulty: 3 Hard
Gradable: automatic
Learning Objective: 11-03 Allocate service department costs using the step method.
Topic: Methods of Allocating Service Department Costs
 

11-129
Copyright © 2017 McGraw-Hill Education. All rights reserved. No reproduction or distribution without the prior written consent of
McGraw-Hill Education.
68. Advanced Computer Solutions, Inc. has two main services: (1) time on a timeshared computer system,
and (2) proprietary computer programs. Computer time is provided by the operation department (Op)
and programs are written by the programming department (P).

The percentage of each service used by each department for a typical period is:

  Supplied
User Op P
Op --- 40%
P 30% ---
Sold to customers 70% 60%

In a typical period, the operation department (Op) spends $4,500 and the programming department (P)
spends $2,500.

Under the reciprocal method what is the algebraic solution to the cost allocation problem?  
 

A.  Op = 4,500 + .40 P; P = 2,500 + .30 Op.


B.  Op = 4,500 + .70 P; P = 2,500 + .60 Op.
C.  Op = 2,500 + .40 P; P = 4,500 + .30 Op.
D.  Op = 2,500 + .70 P; P = 4,500 + .60 Op.

Op direct cost is $4,500 and uses 40% of P.

 
AACSB: Analytical Thinking
AICPA: FN Measurement
Accessibility: Keyboard Navigation
Blooms: Apply
Difficulty: 2 Medium
Gradable: automatic
Learning Objective: 11-04 Allocate service department costs using the reciprocal method.
Topic: Methods of Allocating Service Department Costs
 
69. Which of the following best describes the objective of joint cost allocation?  
 

A.  Inventory valuation.


B.  Pricing goods for sale.
C.  Making decisions about levels of production.
D.  Making decisions about raw materials requirements.

Joint cost allocations generally do not provide good decision making support, it is used primarily to
account for cost allocation between inventory and cost of goods sold.

 
AACSB: Analytical Thinking
AICPA: FN Measurement
Accessibility: Keyboard Navigation
Blooms: Remember

11-130
Copyright © 2017 McGraw-Hill Education. All rights reserved. No reproduction or distribution without the prior written consent of
McGraw-Hill Education.
Difficulty: 2 Medium
Gradable: automatic
Learning Objective: 11-06 Explain why joint costs are allocated.
Topic: Allocation of Joint Costs
 
70. Allocated joint costs are useful for:  
 

A.  setting the selling price of a product.


B.  determining whether to continue producing an item.
C.  controlling user department costs.
D.  determining inventory cost for accounting purposes.

Joint cost allocations generally do not provide good decision making support, it is used primarily to
account for cost allocation between inventory and cost of goods sold.

 
AACSB: Analytical Thinking
AICPA: FN Measurement
Accessibility: Keyboard Navigation
Blooms: Remember
Difficulty: 2 Medium
Gradable: automatic
Learning Objective: 11-06 Explain why joint costs are allocated.
Topic: Allocation of Joint Costs
 
71. For the purposes of allocating joint costs to joint products, the sales price at the point of sale, reduced by
the cost to complete after split-off, is assumed to be equal to the: (CPA adapted) 
 

A.  total costs.


B.  joint costs.
C.  sales price less a normal profit margin at point of sale.
D.  relative sales value at split-off.

This is an estimate of the sales value at split-off.

 
AACSB: Analytical Thinking
AICPA: FN Measurement
Accessibility: Keyboard Navigation
Blooms: Apply
Difficulty: 1 Easy
Gradable: automatic
Learning Objective: 11-06 Explain why joint costs are allocated.
Topic: Joint Cost Allocation Methods
 

11-131
Copyright © 2017 McGraw-Hill Education. All rights reserved. No reproduction or distribution without the prior written consent of
McGraw-Hill Education.
72. Which of the following cost items is not allocable as joint costs when a single manufacturing process
produces several main products and several by-products? 
 

A.  Direct materials.


B.  Variable overhead.
C.  Direct labor.
D.  Freight-out.

Freight-out would occur after the split-off point and would be a separable cost, not a joint cost.

 
AACSB: Analytical Thinking
AICPA: FN Measurement
Accessibility: Keyboard Navigation
Blooms: Remember
Difficulty: 1 Easy
Gradable: automatic
Learning Objective: 11-06 Explain why joint costs are allocated.
Topic: Allocation of Joint Costs
 
73. Which of the following is not a step needed to maximize the profits from joint products? 
 

A.  Forecasting the sales price of each final product.


B.  Identifying alternative sets and quantities of final products possible from the joint process.
C.  Determining how to allocate joint costs to the final products.
D.  Estimating the costs required to further process joint products into salable products.

Allocating profits under joint costing changes based on method but cannot be said to be maximized as it
is a closed set.

 
AACSB: Reflective Thinking
AICPA: FN Measurement
Accessibility: Keyboard Navigation
Blooms: Remember
Difficulty: 2 Medium
Gradable: automatic
Learning Objective: 11-07 Allocate joint costs using the net realizable value method.
Topic: Joint Cost Allocation Methods
 

11-132
Copyright © 2017 McGraw-Hill Education. All rights reserved. No reproduction or distribution without the prior written consent of
McGraw-Hill Education.
74. Which of the following statements about maximizing the profit of joint product processes is true? 
 

A.  Joint processing costs incurred prior to split-off should be allocated before making those decisions.
B.  Only costs caused by management decisions to choose one or another set of products after split-off
are relevant.
C.  Only revenues from selling or processing beyond the split-off point and additional expenditures for
further processing are relevant.
D.  Both B and C are true.

Only revenues and costs incurred after the split-off point are relevant.

 
AACSB: Reflective Thinking
AICPA: FN Measurement
Accessibility: Keyboard Navigation
Blooms: Remember
Difficulty: 2 Medium
Gradable: automatic
Learning Objective: 11-07 Allocate joint costs using the net realizable value method.
Topic: Joint Cost Allocation Methods
 
75. For purposes of allocating joint costs to joint products, the estimated net realizable value at split-off is
equal to: 
 

A.  final sales price reduced by cost to complete after split-off.


B.  sales price less a normal profit margin at the point of sale.
C.  separable product cost plus a normal profit margin.
D.  total sales value less joint costs at point of split-off.

This is the definition of estimated net realizable value.

 
AACSB: Analytical Thinking
AICPA: FN Measurement
Accessibility: Keyboard Navigation
Blooms: Remember
Difficulty: 1 Easy
Gradable: automatic
Learning Objective: 11-07 Allocate joint costs using the net realizable value method.
Topic: Joint Cost Allocation Methods
 

11-133
Copyright © 2017 McGraw-Hill Education. All rights reserved. No reproduction or distribution without the prior written consent of
McGraw-Hill Education.
76. The method of accounting for joint product costs that will produce the same gross margin percentage for
all products is the: 
 

A.  replacement method.


B.  physical quantities method.
C.  net realizable value method.
D.  units produced method.

Under net realizable value, the cost allocated is a constant percentage of the sales value so each product
will have the same gross margin percentage.

 
AACSB: Analytical Thinking
AICPA: FN Measurement
Accessibility: Keyboard Navigation
Blooms: Remember
Difficulty: 1 Easy
Gradable: automatic
Learning Objective: 11-07 Allocate joint costs using the net realizable value method.
Topic: Joint Cost Allocation Methods
 
77. Which of the following statements is false? 
 

A.  The estimated net realizable value for a product is its estimated selling price after processing the
product beyond the split-off point.
B.  In general, it is better to use a product's market value at the split-off point than its estimated net
realizable value.
C.  The estimated net realizable value at the split-off point is calculated by taking the sales value after
further processing and deducting the additional processing costs.
D.  It is better to use the net realizable value method for allocating joint costs than the estimated net
realizable value method.

It is the final selling price minus the additional processing costs.

 
AACSB: Analytical Thinking
AICPA: FN Measurement
Accessibility: Keyboard Navigation
Blooms: Apply
Difficulty: 2 Medium
Gradable: automatic
Learning Objective: 11-07 Allocate joint costs using the net realizable value method.
Topic: Joint Cost Allocation Methods
 

11-134
Copyright © 2017 McGraw-Hill Education. All rights reserved. No reproduction or distribution without the prior written consent of
McGraw-Hill Education.
78. Net realizable value at the split-off point is used to allocate:
 
  Costs After Split-off Point Incurred Joint Costs
A. No No
B. No Yes
C. Yes No
D. Yes Yes
 
 

A.  Option A
B.  Option B
C.  Option C
D.  Option D

The net realizable value method is used to allocate joint costs.

 
AACSB: Analytical Thinking
AICPA: FN Measurement
Accessibility: Keyboard Navigation
Blooms: Remember
Difficulty: 2 Medium
Gradable: automatic
Learning Objective: 11-07 Allocate joint costs using the net realizable value method.
Topic: Joint Cost Allocation Methods
 
79. Products X, Y, and Z are produced from the same process at a cost of $5,200. Five thousand pounds of
raw material yields 1,500 X, 2,500 Y, and 1,000 Z. Selling prices are: X $2 per unit, Y $4 per unit, Z
valueless. The ending inventory of X is 50 units. What is the value of the ending inventory if joint costs
are allocated using net realizable value?  
 

A.  $21.67.
B.  $31.20.
C.  $40.00.
D.  $42.00.

[(1,500 × $2)/($3,000 + 10,000)] × $5,200 = $1,200; ($1,200/1,500) × 50 = $40.00

 
AACSB: Analytical Thinking
AICPA: FN Measurement
Accessibility: Keyboard Navigation
Blooms: Apply
Difficulty: 2 Medium
Gradable: automatic
Learning Objective: 11-07 Allocate joint costs using the net realizable value method.
Topic: Joint Cost Allocation Methods
 

11-135
Copyright © 2017 McGraw-Hill Education. All rights reserved. No reproduction or distribution without the prior written consent of
McGraw-Hill Education.
80. Bonanza Co. manufactures products X and Y from a joint process that also yields a by-product, Z.
Revenue from sales of Z is treated as a reduction of joint costs. Additional information is as follows:
 
  PRODUCTS  
  X Y Z TOTAL
Units
20,000 20,000 10,000 50,000
produced
Joint costs ? ? ? 262,000
Sales value
$300,000 $150,000 $10,000 $460,000
at split-off

Joint costs were allocated using the net realizable value method at the split-off point. The joint costs
allocated to product X were  
 

A.  $75,000.
B.  $100,800.
C.  $150,000.
D.  $168,000.

($300,000/450,000) × ($262,000 - 10,000) = $168,000

 
AACSB: Analytical Thinking
AICPA: FN Measurement
Accessibility: Keyboard Navigation
Blooms: Apply
Difficulty: 2 Medium
Gradable: automatic
Learning Objective: 11-07 Allocate joint costs using the net realizable value method.
Topic: Joint Cost Allocation Methods
 

11-136
Copyright © 2017 McGraw-Hill Education. All rights reserved. No reproduction or distribution without the prior written consent of
McGraw-Hill Education.
81. Great Falls Company makes two products, Wool Gloves and Wool Mittens. They are initially processed
from the same raw material and then, after split-off, further processed separately. Additional
information is as follows:
 
Mitten
  Gloves Total
s
Final Sales Price $9,000 $6,000 $15,000
Joint Costs Prior to Split-Off
? ? 6,600
Point
Costs Beyond Split-Off
$3,000 $3,000 6,000
Point

What are the joint costs allocated to Gloves and Mittens assuming Great Falls uses the estimated net
realizable value approach?
 
  Gloves Mittens
A. $3,300 $3,300
B $3,960 $2,640
C. $4,400 $2,200
D. $4,560 $2,040
 
 

A.  Option A
B.  Option B
C.  Option C
D.  Option D

See calculation below.

G: [($9,000 - 3,000)/($6,000 + 3,000)] × $6,600 = $4,400


H: [($6,000 - 3,000)/($6,000 + 3,000)] × $6,600 = $2,200

 
AACSB: Analytical Thinking
AICPA: FN Measurement
Accessibility: Keyboard Navigation
Blooms: Apply
Difficulty: 2 Medium
Gradable: automatic
Learning Objective: 11-07 Allocate joint costs using the net realizable value method.
Topic: Joint Cost Allocation Methods
 

11-137
Copyright © 2017 McGraw-Hill Education. All rights reserved. No reproduction or distribution without the prior written consent of
McGraw-Hill Education.
82. Atkinson, Inc., manufactures products A, B, and C from a common process. Joint costs were $60,000.
Additional information is as follows:
 
    If Processed Further
Sales
Units Sales Additional
Product Value at
Produced Value Costs
Split-Off
A 6,000 $40,000 $55,000 $4,000
B 4,000 35,000 45,000 6,000
C  2,000     25,000   30,000    8,000
  12,000 $100,000 $130,000 $18,000

Assuming that joint production costs are allocated using the physical quantities method (units
produced), what were the costs allocated to Product A?  
 

A.  $27,000.
B.  $29,000.
C.  $33,000.
D.  $30,000.

(6,000/12,000) × $60,000 = $30,000

 
AACSB: Analytical Thinking
AICPA: FN Measurement
Accessibility: Keyboard Navigation
Blooms: Apply
Difficulty: 2 Medium
Gradable: automatic
Learning Objective: 11-08 Allocate joint costs using the physical quantities method.
Topic: Joint Cost Allocation Methods
 

11-138
Copyright © 2017 McGraw-Hill Education. All rights reserved. No reproduction or distribution without the prior written consent of
McGraw-Hill Education.
83. Atkinson, Inc., manufactures products A, B, and C from a common process. Joint costs were $60,000.
Additional information is as follows:
 
    If Processed Further
Sales
Units Sales Additional
Product Value at
Produced Value Costs
Split-Off
A 6,000 $40,000 $55,000 $4,000
B 4,000 35,000 45,000 6,000
C  2,000     25,000   30,000    8,000
  12,000 $100,000 $130,000 $18,000

Assuming that joint product costs are allocated using the net realizable value method, what were the
total costs assigned to Product B?  
 

A.  $26,000.
B.  $26,796.
C.  $27,000.
D.  $28,286.

$6,000 + [($35,000/100,000) × $60,000] = $27,000

 
AACSB: Analytical Thinking
AICPA: FN Measurement
Accessibility: Keyboard Navigation
Blooms: Apply
Difficulty: 2 Medium
Gradable: automatic
Learning Objective: 11-07 Allocate joint costs using the net realizable value method.
Topic: Joint Cost Allocation Methods
 
84. Tanner Corporation produced 3,660 units, consisting of three separate products, in a joint process for
the year. The market for these products was so unstable that it was not practical to estimate the selling
price of the products. A cost of $425,000 was incurred in the joint process. Product X's production was
80% of product Y's while product Z's production was 125% of product Y's. What is the amount of the
joint cost allocable to product X assuming Tanner uses the physical quantities method of allocation?  
 

A.  $111,475.
B.  $114,865.
C.  $139,344.
D.  $141,667.

X = .8Y; Z = 1.25Y; Y + .8Y + 1.25Y = 3,660 units; Y = 1,200, X = 960, Z = 1,500; 960/3,660 ×
$425,000 = $111,475

 
AACSB: Analytical Thinking
AICPA: FN Measurement

11-139
Copyright © 2017 McGraw-Hill Education. All rights reserved. No reproduction or distribution without the prior written consent of
McGraw-Hill Education.
Accessibility: Keyboard Navigation
Blooms: Apply
Difficulty: 2 Medium
Gradable: automatic
Learning Objective: 11-08 Allocate joint costs using the physical quantities method.
Topic: Joint Cost Allocation Methods
 
85. Cariboo Manufacturing Company incurred a joint cost of $600,000 in the production of X and Y in a
joint process. Presently, 1,800 of X and 1,400 of Y are being produced each month. Management plans
to decrease X's production by 300 units in order to increase the production of Y by 500 units.
Additionally, this change will require minor modifications, which will add $20,000 to the joint cost.
This cost is entirely attributable to product Y. What is the amount of the joint costs allocable to X and Y
before changes to existing production assuming Cariboo allocates their joint costs according to the
proportion of Y and X produced?
 
  Product X Product Y
A. $262,500 $337,500
B. $264,706 $335,294
C. $273,529 $326,471
D. $337,500 $262,500
 
 

A.  Option A
B.  Option B
C.  Option C
D.  Option D

X: (1,800/3,200) × $600,000 = $337,500; Y: (1,400/3,200) × $600,000 = $262,500

 
AACSB: Analytical Thinking
AICPA: FN Measurement
Accessibility: Keyboard Navigation
Blooms: Apply
Difficulty: 2 Medium
Gradable: automatic
Learning Objective: 11-08 Allocate joint costs using the physical quantities method.
Topic: Joint Cost Allocation Methods
 

11-140
Copyright © 2017 McGraw-Hill Education. All rights reserved. No reproduction or distribution without the prior written consent of
McGraw-Hill Education.
86. Upton Company produces two main products and a by-product out of a joint process. The ratio of
output quantities to input quantities of direct material used in the joint process remains consistent from
month to month. Upton has employed the physical-volume method to allocate joint production costs to
the two main products. The net realizable value of the by-product is used to reduce the joint production
costs before the joint costs are allocated to the main products. Data regarding Upton's operations for the
current month are presented in the chart below. During the month, Upton incurred joint production costs
of $2,520,000. The main products are not marketable at the split-off point and, thus, have to be
processed further.
 

First Second
  Main Main
By-
Product Product
product
Monthly output in
90,000 150,000 60,000
pounds
Selling Price per
$30 $14 $2
pound
Separable process
$540,000 $660,000  
costs

The amount of joint production cost that Upton would allocate to the Second Main Product by using the
physical quantities method to allocate joint production costs would be:  
 

A.  $1,200,000.
B.  $1,260,000.
C.  $1,500,000.
D.  $1,575,000.

$2,520,000 - ($2 × 60,000) = $2,400,000; (150,000/240,000) × $2,400,000 = $1,500,000

 
AACSB: Analytical Thinking
AICPA: FN Measurement
Accessibility: Keyboard Navigation
Blooms: Apply
Difficulty: 2 Medium
Gradable: automatic
Learning Objective: 11-08 Allocate joint costs using the physical quantities method.
Topic: Joint Cost Allocation Methods
 

11-141
Copyright © 2017 McGraw-Hill Education. All rights reserved. No reproduction or distribution without the prior written consent of
McGraw-Hill Education.
87. The Mallak Company produced three joint products at a joint cost of $100,000. Two of these products
were processed further. Production and sales were:
 
  Weight Sales Addt’l. Processing Costs
P 300,000 lbs. $245,000 $200,000
Q 100,000 lbs. 30,000 -0-
R 100,000 lbs. 175,000 100,000

If the estimated net realizable value method is used and product Q is accounted for as a main product,
how much of the joint costs would be allocated to product R?  
 

A.  $38,889.
B.  $41,667.
C.  $50,000.
D.  $62,500.

[($175,000 - 100,000)/($75,000 + 30,000 + 45,000)] × $100,000 = $50,000

 
AACSB: Analytical Thinking
AICPA: FN Measurement
Accessibility: Keyboard Navigation
Blooms: Apply
Difficulty: 3 Hard
Gradable: automatic
Learning Objective: 11-07 Allocate joint costs using the net realizable value method.
Topic: Joint Cost Allocation Methods
 
88. The Mallak Company produced three joint products at a joint cost of $100,000. Two of these products
were processed further. Production and sales were:
 
  Weight Sales Addt’l. Processing Costs
P 300,000 lbs. $245,000 $200,000
Q 100,000 lbs. 30,000 -0-
R 100,000 lbs. 175,000 100,000

Assume Q is a by-product and Mallak uses the cost reduction method of accounting for by-product cost.
If estimated net realizable value is used, how much of the joint costs would be allocated to product R?  
 

A.  $38,889.
B.  $43,750.
C.  $50,000.
D.  $62,500.

[($175,000 - 100,000)/($75,000 + 45,000)] × ($100,000 - 30,000) = $43,750

 
AACSB: Analytical Thinking

11-142
Copyright © 2017 McGraw-Hill Education. All rights reserved. No reproduction or distribution without the prior written consent of
McGraw-Hill Education.
AICPA: FN Measurement
Accessibility: Keyboard Navigation
Blooms: Apply
Difficulty: 3 Hard
Gradable: automatic
Learning Objective: 11-10 Account for by-products.
Topic: Deciding What to Do with By-Products
 
89. The Mallak Company produced three joint products at a joint cost of $100,000. Two of these products
were processed further. Production and sales were:
 
  Weight Sales Addt’l. Processing Costs
P 300,000 lbs. $245,000 $200,000
Q 100,000 lbs. 30,000 -0-
R 100,000 lbs. 175,000 100,000

If joint costs are allocated based on relative weight of the outputs and all products are main products,
how much of the joint costs would be allocated to product P?  
 

A.  $43,750.
B.  $50,000.
C.  $60,000.
D.  $62,500.

[300,000/(300,000 + 100,000 + 100,000)] × $100,000 = $60,000

 
AACSB: Analytical Thinking
AICPA: FN Measurement
Accessibility: Keyboard Navigation
Blooms: Apply
Difficulty: 1 Easy
Gradable: automatic
Learning Objective: 11-07 Allocate joint costs using the net realizable value method.
Topic: Joint Cost Allocation Methods
 

11-143
Copyright © 2017 McGraw-Hill Education. All rights reserved. No reproduction or distribution without the prior written consent of
McGraw-Hill Education.
90. The Mallak Company produced three joint products at a joint cost of $100,000. Two of these products
were processed further. Production and sales were:
 
  Weight Sales Addt’l. Processing Costs
P 300,000 lbs. $245,000 $200,000
Q 100,000 lbs. 30,000 -0-
R 100,000 lbs. 175,000 100,000

What is the net income of Mallak Company if the estimated net realizable value method of joint cost
allocation is used?  
 

A.  $20,000.
B.  $50,000.
C.  $150,000.
D.  $350,000.

$450,000 - $300,000 - $100,000 = $50,000

 
AACSB: Analytical Thinking
AICPA: FN Measurement
Accessibility: Keyboard Navigation
Blooms: Apply
Difficulty: 2 Medium
Gradable: automatic
Learning Objective: 11-07 Allocate joint costs using the net realizable value method.
Topic: Joint Cost Allocation Methods
 

11-144
Copyright © 2017 McGraw-Hill Education. All rights reserved. No reproduction or distribution without the prior written consent of
McGraw-Hill Education.
91. Cambridge Company manufactures three main products, L, M, and N, from a joint process. Additional
information for June production activity follows:
 
  L M N Total
Units
50,000 40,000 10,000 100,000
produced
Joint costs ? ? ? $450,000
Sales value
$420,000 $270,000 $60,000 $750,000
at split-off
Additional
costs if
$88,000 $30,000 $12,000 $130,000
process
further
Sale value if
processed $538,000 $320,000 $78,000 $936,000
further

Assuming that the 10,000 units of N were processed further and sold for $78,000, what was
Anchorage's gross profit from this sale? Assume the physical quantities method of allocation is used.  
 

A.  $21,000.
B.  $28,500.
C.  $30,000.
D.  $66,000.

[10,000/(50,000 + 40,000 + 10,000)] × $450,000 = $45,000; $78,000 - ($45,000 + 12,000) = $21,000

 
AACSB: Analytical Thinking
AICPA: FN Decision Making
Accessibility: Keyboard Navigation
Blooms: Apply
Difficulty: 2 Medium
Gradable: automatic
Learning Objective: 11-08 Allocate joint costs using the physical quantities method.
Topic: Joint Cost Allocation Methods
 
92. Which of the following is not a physical measure that can be used for allocating joint costs using the
physical quantities method?  
 

A.  Tons of steel.


B.  Ounces of gold.
C.  Dollars of labor.
D.  Feet of lumber.

Dollars of labor is not a physical measurement, it is a value measurement.

 
AACSB: Analytical Thinking

11-145
Copyright © 2017 McGraw-Hill Education. All rights reserved. No reproduction or distribution without the prior written consent of
McGraw-Hill Education.
AICPA: FN Measurement
Accessibility: Keyboard Navigation
Blooms: Apply
Difficulty: 1 Easy
Gradable: automatic
Learning Objective: 11-08 Allocate joint costs using the physical quantities method.
Topic: Joint Cost Allocation Methods
 
93. In joint product costing and analysis, which one of the following costs is relevant when deciding the
point at which a product should be sold in order to maximize profits? (CMA adapted)  
 

A.  separable costs after the split-off point.


B.  joint costs to the spilt-off point.
C.  sales salaries for the period when the units were produced.
D.  purchase costs of the materials required for the joint products.

Only the separable costs after the split-off point are relevant when deciding the point at which a product
should be sold to maximize profits.

 
AACSB: Analytical Thinking
AICPA: FN Decision Making
Accessibility: Keyboard Navigation
Blooms: Apply
Difficulty: 1 Easy
Gradable: automatic
Learning Objective: 11-09 Explain how cost data are used in the sell-or-process-further decision.
Topic: Deciding Whether to Sell Goods Now or Process Them Further
 

11-146
Copyright © 2017 McGraw-Hill Education. All rights reserved. No reproduction or distribution without the prior written consent of
McGraw-Hill Education.
94. Delite Confectionary Company produces various types of candies. Several candies could be sold at the
split-off point or processed further and sold in a different form after further processing. The candies are
produced in a joint processing operation with $500,000 of joint processing costs monthly, which are
allocated based on pounds produced. Information concerning this process for a recent month appears
below:
 

Price
per Further Price after
Numbe
pound processing processing
Candy r of
at costs further
type pounds
split-
off
Sweet Mats 50,000 $8 $75,000 $10.00
Chocolate
100,000 $10 $30,000 $10.50
Delight
Minty
25,000 $5 $20,000 $5.50
Wonders

Based on the information presented, which of the products should be processed further?  
 

A.  Sweet Meats only.


B.  Both Sweet Meets and Chocolate Delight.
C.  Minty Wonders only.
D.  Both Sweet Meats and Minty Wonders.

See calculation below.

Sweet Meats: ($10 - $8) = $2 × 50,000 units = $100,000 - costs of $75,000 = $25,000 profit
Chocolate Delight ($10.50 - $10.00) = $0.50 × 100,000 units = $50,000 - costs of $30,000 = $20,000
profit
Minty Wonders ($5.50 - $5.00) = $0.50 × 25,000 units = $12,500 - costs of $20,000 = $7,500 loss

 
AACSB: Analytical Thinking
AICPA: FN Measurement
Accessibility: Keyboard Navigation
Blooms: Apply
Difficulty: 2 Medium
Gradable: automatic
Learning Objective: 11-09 Explain how cost data are used in the sell-or-process-further decision.
Topic: Deciding Whether to Sell Goods Now or Process Them Further
 

11-147
Copyright © 2017 McGraw-Hill Education. All rights reserved. No reproduction or distribution without the prior written consent of
McGraw-Hill Education.
95. Delite Confectionary Company produces various types of candies. Several candies could be sold at the
split-off point or processed further and sold in a different form after further processing. The candies are
produced in a joint processing operation with $500,000 of joint processing costs monthly, which are
allocated based on pounds produced. Information concerning this process for a recent month appears
below:
 

Price
per Further Price after
Numbe
pound processing processing
Candy r of
at costs further
type pounds
split-
off
Sweet Mats 50,000 $8 $75,000 $10.00
Chocolate
100,000 $10 $30,000 $10.50
Delight
Minty
25,000 $5 $20,000 $5.50
Wonders

The net advantage (disadvantage) of processing Sweet Meats further is:  


 

A.  a $25,000 disadvantage to process further.


B.  a $32,143 advantage to process further.
C.  a $25,000 advantage to process further.
D.  a $282,143 disadvantage to process further.

Sweet Meats: ($10 - $8) = $2 × 50,000 units = $100,000 - costs of $75,000 = $25,000 profit

 
AACSB: Analytical Thinking
AICPA: FN Measurement
Accessibility: Keyboard Navigation
Blooms: Apply
Difficulty: 2 Medium
Gradable: automatic
Learning Objective: 11-09 Explain how cost data are used in the sell-or-process-further decision.
Topic: Deciding Whether to Sell Goods Now or Process Them Further
 

11-148
Copyright © 2017 McGraw-Hill Education. All rights reserved. No reproduction or distribution without the prior written consent of
McGraw-Hill Education.
96. Delite Confectionary Company produces various types of candies. Several candies could be sold at the
split-off point or processed further and sold in a different form after further processing. The candies are
produced in a joint processing operation with $500,000 of joint processing costs monthly, which are
allocated based on pounds produced. Information concerning this process for a recent month appears
below:
 

Price
per Further Price after
Numbe
pound processing processing
Candy r of
at costs further
type pounds
split-
off
Sweet Mats 50,000 $8 $75,000 $10.00
Chocolate
100,000 $10 $30,000 $10.50
Delight
Minty
25,000 $5 $20,000 $5.50
Wonders

The joint processing costs in this operation:  


 

A.  should be allocated to products to determine whether they are sold at split-off or processed further.
B.  should be ignored in determining whether to sell at split-off or process further.
C.  should be ignored in making all product decisions.
D.  are never included in product cost, as they are misleading to all management decisions.

Joint costs are sunk given the decision to process further.

 
AACSB: Analytical Thinking
AICPA: FN Measurement
Accessibility: Keyboard Navigation
Blooms: Apply
Difficulty: 1 Easy
Gradable: automatic
Learning Objective: 11-09 Explain how cost data are used in the sell-or-process-further decision.
Topic: Deciding Whether to Sell Goods Now or Process Them Further
 

11-149
Copyright © 2017 McGraw-Hill Education. All rights reserved. No reproduction or distribution without the prior written consent of
McGraw-Hill Education.
97. Delite Confectionary Company produces various types of candies. Several candies could be sold at the
split-off point or processed further and sold in a different form after further processing. The candies are
produced in a joint processing operation with $500,000 of joint processing costs monthly, which are
allocated based on pounds produced. Information concerning this process for a recent month appears
below:
 

Price
per Further Price after
Numbe
pound processing processing
Candy r of
at costs further
type pounds
split-
off
Sweet Mats 50,000 $8 $75,000 $10.00
Chocolate
100,000 $10 $30,000 $10.50
Delight
Minty
25,000 $5 $20,000 $5.50
Wonders

If Chocolate Delight is processed further, the gross profit margin that will appear in a product line
income statement for Chocolate Delight would be:  
 

A.  $734,286.
B.  $520,000.
C.  $1,020,000.
D.  $632,596.

(100,000 × $10.50) - $30,000 = $1,020,000; $1,020,000 - ($500,000 × 100/175) = $734,286

 
AACSB: Analytical Thinking
AICPA: FN Measurement
Accessibility: Keyboard Navigation
Blooms: Apply
Difficulty: 2 Medium
Gradable: automatic
Learning Objective: 11-09 Explain how cost data are used in the sell-or-process-further decision.
Topic: Deciding Whether to Sell Goods Now or Process Them Further
 

11-150
Copyright © 2017 McGraw-Hill Education. All rights reserved. No reproduction or distribution without the prior written consent of
McGraw-Hill Education.
98. The Foxmoor Company produces three products, X, Y, and Z from a single raw material input. Product
Y can be sold at the split-off point for total revenues of $50,000 or it can be processed further at a total
cost of $16,000 and then sold for $68,000. Product Y:  
 

A.  should be sold at the split-off point, rather than processed further.
B.  would increase the company's overall net income by $18,000 if processed further and then sold.
C.  would increase the company's overall net income by $68,000 if processed further and then sold.
D.  would increase the company's overall net income by $2,000 if processed further and then sold.

Process further: $68,000 - 16,000 = $52,000 vs. $50,000 sell now

 
AACSB: Analytical Thinking
AICPA: FN Decision Making
Accessibility: Keyboard Navigation
Blooms: Analyze
Difficulty: 2 Medium
Gradable: automatic
Learning Objective: 11-09 Explain how cost data are used in the sell-or-process-further decision.
Topic: Deciding Whether to Sell Goods Now or Process Them Further
 
99. Product C is one of several joint products that come out of Department M. The joint costs incurred in
Department M total $40,000. Product C can be sold at split-off or processed further and sold as a higher
quality item. The decision to process further should be based on the:  
 

A.  assumption that the $40,000 is irrelevant.


B.  allocation of the $40,000, using the net realizable value.
C.  allocation of the $40,000, using a physical measures approach.
D.  allocation of the $40,000, using the relative sales value at split-off method.

The decision should not depend upon allocations. The joint cost is irrelevant.

 
AACSB: Analytical Thinking
AICPA: FN Decision Making
Accessibility: Keyboard Navigation
Blooms: Apply
Difficulty: 2 Medium
Gradable: automatic
Learning Objective: 11-09 Explain how cost data are used in the sell-or-process-further decision.
Topic: Deciding Whether to Sell Goods Now or Process Them Further
 

11-151
Copyright © 2017 McGraw-Hill Education. All rights reserved. No reproduction or distribution without the prior written consent of
McGraw-Hill Education.
100. The characteristic that is most often used to distinguish a product as either a main product or a by-
product is the amount of: 
 

A.  sales value of the products produced during the common production process.
B.  direct manufacturing costs (e.g., materials) incurred before the split-off point.
C.  physical measures in the products produced during the common production process.
D.  time (i.e., labor) required to produce the products from start to finish.

A by-product has minimal value relative to the main products.

 
AACSB: Analytical Thinking
AICPA: FN Measurement
Accessibility: Keyboard Navigation
Blooms: Apply
Difficulty: 1 Easy
Gradable: automatic
Learning Objective: 11-10 Account for by-products.
Topic: Deciding What to Do with By-Products
 
101. If by-product revenue is treated as other revenue instead of deducted from the net-realizable-value of the
main products: 
 

A.  overall gross margin of the company will be higher.


B.  overall gross margin of the company will be lower.
C.  the answer would depend on how joint product costs were allocated.
D.  there is no difference in the overall gross margin of the company.

Costs are allocated to joint products or joint products and by-products, but costs do not change;
therefore gross margin is the same under all alternatives.

 
AACSB: Reflective Thinking
AICPA: FN Measurement
Accessibility: Keyboard Navigation
Blooms: Remember
Difficulty: 2 Medium
Gradable: automatic
Learning Objective: 11-10 Account for by-products.
Topic: Deciding What to Do with By-Products
 

11-152
Copyright © 2017 McGraw-Hill Education. All rights reserved. No reproduction or distribution without the prior written consent of
McGraw-Hill Education.
102. Products with a relatively minor sales value are called: 
 

A.  scrap.
B.  spoilage.
C.  by-products.
D.  main products.

This is a definition of a by-product.

 
AACSB: Analytical Thinking
AICPA: FN Measurement
Accessibility: Keyboard Navigation
Blooms: Remember
Difficulty: 1 Easy
Gradable: automatic
Learning Objective: 11-10 Account for by-products.
Topic: Deciding What to Do with By-Products
 
103. Joint products and by-products are produced simultaneously by a single process or series of processes
and:  
 

A.  joint products are salable at the split-off point, but by-products are not.
B.  by-products are salable at the split-off point, but joint products are not.
C.  the revenue from by-products may be recognized at the time of production.
D.  all by-products must be allocated some portion of joint costs.

By-products can be recognized either at time of production or time of sale.

 
AACSB: Analytical Thinking
AICPA: FN Measurement
Accessibility: Keyboard Navigation
Blooms: Remember
Difficulty: 2 Medium
Gradable: automatic
Learning Objective: 11-10 Account for by-products.
Topic: Deciding What to Do with By-Products
 

11-153
Copyright © 2017 McGraw-Hill Education. All rights reserved. No reproduction or distribution without the prior written consent of
McGraw-Hill Education.
104. Which of the following statements regarding accounting for by-products is true?  
 

A.  If all products are sold in the same period as they are produced, treating by-product revenue as other
revenue will result in a higher overall gross margin.
B.  If all products are sold in the same period as they are produced, treating by-product net-realizable-
value as a deduction of the cost of the main products will result in a higher overall gross margin.
C.  If all products are sold in the same period as they are produced, total reported revenues will be the
same regardless of how by-product revenue is accounted for.
D.  If all products are sold in the same period as they are produced, the reported gross margin will be
the same regardless of how by-product revenue is accounted for.

This is the result of using the Net Realizable Value Method.

 
AACSB: Reflective Thinking
AICPA: FN Measurement
Accessibility: Keyboard Navigation
Blooms: Remember
Difficulty: 2 Medium
Gradable: automatic
Learning Objective: 11-10 Account for by-products.
Topic: Deciding What to Do with By-Products
 
 

Essay Questions
 

11-154
Copyright © 2017 McGraw-Hill Education. All rights reserved. No reproduction or distribution without the prior written consent of
McGraw-Hill Education.
105. For each of the support service costs listed below, name an appropriate cost allocation base:
 
(1.) Building rental cost                                           
(2.) Payroll department
                                          
salaries
(3.) Company cafeteria
                                          
cost
(4.) Human resources
                                          
department
(5.) Accounting
                                          
department
(6.) Computer equipment
                                          
depreciation
(7.) Insurance costs on
                                          
computer equipment
(8.) Depreciation on
                                          
company airplane
(9.) Factory manager                                           
(10.) Cost to clean
                                          
company uniforms
(11.) Costs of corporate
                                          
daycare facility
(12.) Equipment
                                          
maintenance
(13.) Cost of corporate
                                          
workout facility
(14.) Building insurance                                           
(15.) Cost of delivery
truck for a moving                                           
company
 
 

(Answers may vary)

(1) Square footage; space occupied


(2) Number of employees
(3) Number of meals served
(4) Number of employees
(5) Number of documents
(6) Hours of use
(7) Hours of use
(8) Hours of use; miles traveled
(9) Number of employees
(10) Number of uniforms; weight of laundry processed
(11) Number of employees
(12) Number of machines; hours of use

11-155
Copyright © 2017 McGraw-Hill Education. All rights reserved. No reproduction or distribution without the prior written consent of
McGraw-Hill Education.
(13) Number of employees
(14) Square footage; space occupied
(15) Miles driven

 
AACSB: Reflective Thinking
AICPA: FN Decision Making
Blooms: Remember
Difficulty: 1 Easy
Gradable: manual
Learning Objective: 11-01 Explain why service costs are allocated.
Topic: Methods of Allocating Service Department Costs
 

11-156
Copyright © 2017 McGraw-Hill Education. All rights reserved. No reproduction or distribution without the prior written consent of
McGraw-Hill Education.
11-157
Copyright © 2017 McGraw-Hill Education. All rights reserved. No reproduction or distribution without the prior written consent of
McGraw-Hill Education.
106. Wimbledon Corporation has two production Departments: Assembly and Machining and two service
departments: Personnel and Cafeteria. Direct costs for each department and the proportion of service
costs used by the various departments for the month of July, 2016 are as follows:
 
    Proportion of Services Used by:
Departm Direct Personn Cafeter Machini Assemb
ent costs el ia ng ly
$30,00
Personnel   0.40 0.30 0.30
0
$50,00
Cafeteria 0.20   0.50 0.30
0
Machinin $80,00
       
g 0
$70,00
Assembly        
0

Required:

Compute the allocation of service department costs to producing departments for July, 2016 using the
direct-method.  
 

    Proportion of Services Used by:


Departm Direct Personn Cafeter Machini Assemb
ent costs el ia ng ly
$30,00
Personnel   0.40 0.30 0.30
0
$50,00
Cafeteria 0.20   0.50 0.30
0
Machinin $80,00
       
g 0
$70,00
Assembly        
0

Personn Cafete Machini Assemb Total


 
el ria ng ly costs
Direct
departme          
nt
$50,00 $230,00
Costs $30,000 $80,000 $70,000
0 0
Allocatio
n of          
Personnel

11-158
Copyright © 2017 McGraw-Hill Education. All rights reserved. No reproduction or distribution without the prior written consent of
McGraw-Hill Education.
.30/.60 .
 -30,000   15,000 15,000  
30/.60
Allocatio
n of          
Cafeteria
.50/.80 .
              -50,000    31,250   18,750              
30/.80
         $ $126,25 $103,75 $230,00
  $0
0 0 0 0
 
AACSB: Analytical Thinking
AICPA: FN Measurement
Blooms: Analyze
Difficulty: 2 Medium
Gradable: manual
Learning Objective: 11-02 Allocate service department costs using the direct method.
Topic: Methods of Allocating Service Department Costs
 

11-159
Copyright © 2017 McGraw-Hill Education. All rights reserved. No reproduction or distribution without the prior written consent of
McGraw-Hill Education.
107. Prestige Financial Credit Company produces two styles of credit reports: Individual and Corporate. The
difference between the two is the amount of background information and data collection required. The
Corporate report uses more skilled personnel because additional checking and data are required. The
relevant figures for the year just completed follow. Total support service costs to be allocated are
$3,200,000.
 
Allocation base Individual Corporate
Data purchased $40,000 $80,000
Research hours 24,000 30,000
Interview hours 1,000 10,000
Number of reports 16,000 3,000

Required:

For each of the four potential allocation bases, determine the amount of support-service cost allocated
to each type of report. Round all percentages to two decimal places.  
 

Allocatio Individu
Percent Corporate Percent
n base al
Data
$40,000 33.33% $80,000 66.67%
purchased
Research
24,000 44.44% 30,000 55.56%
hours
Interview
1,000 9.09% 10,000 90.91%
hours
Number
16,000 84.21% 3,000 15.79%
of reports
    Costs allocated  
Allocatio Total
  Individual Corporate
n base costs
Data $1,066,66 $2,133,33 $3,200,00
 
purchased 7 3 0
Research $1,422,08 $1,777,92 $3,200,00
 
hours 0 0 0
Interview $2,909,12 $3,200,00
  $290,880
hours 0 0
Number $2,694,72 $3,200,00
  $505,280
of reports 0 0
 
AACSB: Analytical Thinking
AICPA: FN Measurement
Blooms: Analyze
Difficulty: 1 Easy
Gradable: manual

11-160
Copyright © 2017 McGraw-Hill Education. All rights reserved. No reproduction or distribution without the prior written consent of
McGraw-Hill Education.
Learning Objective: 11-01 Explain why service costs are allocated.
Topic: Service Department Cost Allocation
 

11-161
Copyright © 2017 McGraw-Hill Education. All rights reserved. No reproduction or distribution without the prior written consent of
McGraw-Hill Education.
11-162
Copyright © 2017 McGraw-Hill Education. All rights reserved. No reproduction or distribution without the prior written consent of
McGraw-Hill Education.
108. Data Master is a computer software consulting company. Its three major functional areas are computer
programming, information systems consulting, and software training. Cynthia Moore, a pricing analyst
in the Accounting Department, has been asked to develop total costs for the functional areas. These
costs will be used as a guide in pricing a new contract. In computing these costs, Moore is considering
three different methods of allocating overhead costs-the direct method, the step method, and the
reciprocal method. Moore assembled the following data on overhead from its two service departments,
the Information Systems Department and the Facilities Department.

Service
  User Departments  
Departments
Comp
Info
uter
  Syste Facili Consu Trai
Progra Total
ms ties lt ning
m
Budget
ed $50,0 $25,00 $75,00 $110,0 $85,0 $345,0
Overhe 00 0 0 00 00 00
ad
Info
System   400 1,100 600 900 3,000
s (hrs)
Faciliti
200,0 400,00 600,00 800,0 2,000,0
es (Sq  
00 0 0 00 00
ft)

Information systems is allocated on the basis of hours of computer usage; facilities are allocated on the
basis of floor space.

Required:

Allocate the service department costs to the user departments using the direct method. (Round to the
nearest dollar and provide total user department costs)  
 

Computer programming $101,710; Consulting: $129,871; Training: $113,419


 

    IS Fac CP Cons Train


De Allocatio $50,00 $25,0 $75,00 $110,0 $85,00
pt n 0 00 0 00 0
IS
1,100/2,6 -
to   21,154    
00 21,154
CP
IS
to 600/2,60 -
    11,538  
Con 0 11,538
s

11-163
Copyright © 2017 McGraw-Hill Education. All rights reserved. No reproduction or distribution without the prior written consent of
McGraw-Hill Education.
IS
to 900/2,60 -
      17,308
Trai 0 17,308
n
Fac
400/1,80
to   -5,556 5,556    
0
CP
Fac
to 600/1,80
  -8,333   8,333  
Con 0
s
Fac
-
to 800/1,80                                            11,11
11,11
Trai 0        1
1
n
                       $101,7 $129,8 $113,4
   
0 0 10 71 19
 
AACSB: Analytical Thinking
AICPA: FN Measurement
Blooms: Apply
Difficulty: 1 Easy
Gradable: manual
Learning Objective: 11-02 Allocate service department costs using the direct method.
Topic: Methods of Allocating Service Department Costs
 

11-164
Copyright © 2017 McGraw-Hill Education. All rights reserved. No reproduction or distribution without the prior written consent of
McGraw-Hill Education.
11-165
Copyright © 2017 McGraw-Hill Education. All rights reserved. No reproduction or distribution without the prior written consent of
McGraw-Hill Education.
109. Data Master is a computer software consulting company. Its three major functional areas are computer
programming, information systems consulting, and software training. Cynthia Moore, a pricing analyst
in the Accounting Department, has been asked to develop total costs for the functional areas. These
costs will be used as a guide in pricing a new contract. In computing these costs, Moore is considering
three different methods of allocating overhead costs-the direct method, the step method, and the
reciprocal method. Moore assembled the following data on overhead from its two service departments,
the Information Systems Department and the Facilities Department.
 
Service
  User Departments  
Departments
Comp
Info
uter
  Syste Facili Consu Trai
Progra Total
ms ties lt ning
m
Budget
ed $50,0 $25,00 $75,00 $110,0 $85,0 $345,0
Overhe 00 0 0 00 00 00
ad
Info
System   400 1,100 600 900 3,000
s (hrs)
Faciliti
200,0 400,00 600,00 800,0 2,000,0
es (Sq  
00 0 0 00 00
ft)

Information systems is allocated on the basis of hours of computer usage; facilities are allocated on the
basis of floor space.

Required:

Allocate the service department costs to the user departments using the step method. Allocate
Information Systems first and round to the nearest dollar. Provide total user department costs.  
 

Computer Programming: $100,370; Consulting: $130,556; Training: $114,074


 

    IS Fac CP Cons Train


De Allocatio $50,00 $25,0 $110,00 $85,00
$75,000
pt n 0 00 0 0
IS
400/3,00
to -6,667 6,667      
0
Fac
IS
1,100/3,0 -
to   18,333    
00 18,333
CP
IS 600/3,00 -     10,000  

11-166
Copyright © 2017 McGraw-Hill Education. All rights reserved. No reproduction or distribution without the prior written consent of
McGraw-Hill Education.
to
Con 0 10,000
s
IS
to 900/3,00 -
      15,000
Trai 0 15,000
n
Fac
400/1,80
to   -7,037 7,037    
0
CP
Fac
-
to 600/1,80
  10,55   10,556  
Con 0
6
s
Fac
-
to 800/1,80                                              14,07
14,07
Trai 0          4
4
n
$100,37 $130,55 $114,0
             0         0
0 6 74
 
AACSB: Analytical Thinking
AICPA: FN Measurement
Blooms: Apply
Difficulty: 2 Medium
Gradable: manual
Learning Objective: 11-03 Allocate service department costs using the step method.
Topic: Methods of Allocating Service Department Costs
 

11-167
Copyright © 2017 McGraw-Hill Education. All rights reserved. No reproduction or distribution without the prior written consent of
McGraw-Hill Education.
11-168
Copyright © 2017 McGraw-Hill Education. All rights reserved. No reproduction or distribution without the prior written consent of
McGraw-Hill Education.
110. Yellville Regional Hospital is a small hospital with two service departments and three revenue areas:
 
Direct Laundry
Service Dept Costs Sq Ft Pounds
   Housekeeping $80,000 - 16,000
   Laundry $132,000 500  
Revenue Areas      
   Surgery $400,000 1,500 48,000
   Semiprivate  rooms $200,000 2,000 24,000
   Maternity $150,000 1,000 12,000

The hospital wants to allocate the service department costs to the revenue areas. Housekeeping is
allocated based on square footage; Laundry is allocated based on pounds of laundry. The normal
capacity for Surgery is 200 hours per month; normal capacity for semiprivate rooms is 600 patient days;
and normal capacity for maternity is 200 patient days.

Required:

Determine the overhead rate for the three revenue areas. Allocate the service department costs to the
revenue areas using the direct method.  
 

Surgery: $2,510.48/hr; SemiPrivate rooms: $455.45/day; Maternity: $933.18/day


 

Hous
    Laund Surg SemiP Mat
ek
Allocati $80,0 $132,0 $400,0 $200,0 $150,0
Dept
on 00 00 00 00 00
HK -
1,500/4,
to 26,66   26,667    
500
Surg 7
HK
-
to 2,000/4,
35,55     35,555  
Semi 500
5
P
HK -
1,000/4,
to 17,77       17,778
500
Mat 8
Laun
-
d to 48/84   75,429    
75,429
Surg
Laun
d to -
24/84     37,714  
Semi 37,714
P

11-169
Copyright © 2017 McGraw-Hill Education. All rights reserved. No reproduction or distribution without the prior written consent of
McGraw-Hill Education.
Laun
            -                               18,85
d to 12/84
   18,857         7
Mat
          $502,0 $273,2 $186,6
             0
0 96 69 35

Surgery: $502,096/200 hrs = $2,510.48/hr


SemiPrivate Rooms: $273,269/600 patient days = $455.45/day
Maternity: $186,635/200 patient days = $933.18/day
 
AACSB: Analytical Thinking
AICPA: FN Measurement
Blooms: Analyze
Difficulty: 2 Medium
Gradable: manual
Learning Objective: 11-02 Allocate service department costs using the direct method.
Topic: Methods of Allocating Service Department Costs
 

11-170
Copyright © 2017 McGraw-Hill Education. All rights reserved. No reproduction or distribution without the prior written consent of
McGraw-Hill Education.
11-171
Copyright © 2017 McGraw-Hill Education. All rights reserved. No reproduction or distribution without the prior written consent of
McGraw-Hill Education.
111. Yellville Regional Hospital is a small hospital with two service departments and three revenue areas:
 
Direct Laundry
Service Dept Costs Sq Ft Pounds
   Housekeeping $80,000 - 16,000
   Laundry $132,000 500  
Revenue Areas      
   Surgery $400,000 1,500 48,000
   Semiprivate Rooms $200,000 2,000 24,000
   Maternity $150,000 1,000 12,000

The hospital wants to allocate the service department costs to the revenue areas. Housekeeping is
allocated based on square footage; Laundry is allocated based on pounds of laundry. The normal
capacity for Surgery is 200 hours per month; normal capacity for semiprivate rooms is 600 patient days;
and normal capacity for maternity is 200 patient days.

Required:

Determine the overhead rate for the three revenue areas. Allocate the service department costs to the
revenue areas using the step method. Allocate the service department with the largest dollar value first.  
 

Surgery: $2,485.34/hr; SemiPrivate rooms: $461.04/day; Maternity: $941.56/day


 

Hous
    Laund Surg SemiP Mat
ek
Allocatio $80,0 $132,0 $400,0 $200,0 $150,0
Dept
n 00 00 00 00 00
Laun
21,12 -
d to 16/100      
0 21,120
HK
Laun
-
d to 48/100   63,360    
63,360
Surg
Laun
d to -
24/100     31,680  
Semi 31,680
P
Laun
-
d to 12/100       15,840
15,840
Mat
HK -
1,500/4,5
to 33,70   33,707    
00
Surg 7
HK 2,000/4,5 -     44,942  

11-172
Copyright © 2017 McGraw-Hill Education. All rights reserved. No reproduction or distribution without the prior written consent of
McGraw-Hill Education.
to
44,94
Semi 00
2
P
HK -
1,000/4,5              
to 22,47                             22,471
00  
Mat 1
          $497,0 $276,6 $188,3
              0
0 67 22 11

Surgery: $497,067/200 hrs = $2,485.34/hr


SemiPrivate Rooms: $276,622/600 patient days = $461.04/day
Maternity: $188,311/200 patient days = $941.56/day
 
AACSB: Analytical Thinking
AICPA: FN Measurement
Blooms: Analyze
Difficulty: 2 Medium
Gradable: manual
Learning Objective: 11-03 Allocate service department costs using the step method.
Topic: Methods of Allocating Service Department Costs
 

11-173
Copyright © 2017 McGraw-Hill Education. All rights reserved. No reproduction or distribution without the prior written consent of
McGraw-Hill Education.
112. Jack Donaldson owns and operates Jack's Abstracting Service. Jack's two revenue generating operations
(Abstracting Services and Closing Services) are supported by two service departments: Clerical and
Custodial. Costs in the service departments are allocated in the following order using the designated
allocation bases.
Clerical: number of transactions processed.
Custodial: square footage of space occupied.
Average and expected activity levels for next month are as follows:
 
Numbers of Square Expected
 
Transactions Footage     Costs
Abstract services 50 1,800  
Closing services 25 2,200  
Clerical   1,600 $40,000
Custodial 5   10,000

Required:

Use the direct method to allocate the service department costs to the revenue generating departments.
Provide the total costs for the revenue departments.  
 

Abstract: $31,167; Close: $18,833


 

Clerica Abstrac
    Cust Close
l t
Dept Allocation $40,000 $10,000    
Cleri
c to 50/75 -26,667   26,667  
Abstr
Cleri
c to 25/75 -13,333     13,333
Close
Cust
to 1,800/4,000   -4,500 4,500  
Abstr
Cust
to 2,200/4,000               -5,500                   5,500
close
              0           0 $31,167 $18,833
 
AACSB: Analytical Thinking
AICPA: FN Measurement
Blooms: Apply
Difficulty: 2 Medium
Gradable: manual
Learning Objective: 11-02 Allocate service department costs using the direct method.

11-174
Copyright © 2017 McGraw-Hill Education. All rights reserved. No reproduction or distribution without the prior written consent of
McGraw-Hill Education.
Topic: Methods of Allocating Service Department Costs
 

11-175
Copyright © 2017 McGraw-Hill Education. All rights reserved. No reproduction or distribution without the prior written consent of
McGraw-Hill Education.
11-176
Copyright © 2017 McGraw-Hill Education. All rights reserved. No reproduction or distribution without the prior written consent of
McGraw-Hill Education.
113. Jack Donaldson owns and operates Jack's Abstracting Service. Jack's two revenue generating operations
Abstracting Services and Closing Services are supported by two service departments: Clerical and
Custodial. Costs in the service departments are allocated in the following order using the designated
allocation bases.
Clerical: number of transactions processed.
Custodial: square footage of space occupied.
Average and expected activity levels for next month are as follows:
 
Number of Square Expected
 
Transactions Footage     Costs
Abstract services 50 1,800  
Closing services 25 2,200  
Clerical   1,600 $40,000
Custodial 5   10,000

Required:

a. Use the step method to allocate the service department costs to the revenue generating departments.
Assume Clerical costs are allocated before Custodial costs and round all calculations to the nearest
whole dollar. Provide the total costs for the revenue departments.
b. Use the step method to allocate the service department costs to the revenue generating departments
but now assume Custodial costs are allocated before Clerical costs. Provide the total costs for the
revenue departments.  
 

a. Abstract: $30,625; Close: $19,375


b. Abstract: $31,785; Close: $18,215

Clerica
a.   Cust Abstract Close
l
Dept Allocation $40,000 $10,000    
Cleri
c to 5/80 -2,500 2,500    
Cust
Cleri
c to 50/80 -25,000   25,000  
Abstr
Cleri
c to 25/80 -12,500     12,500
Close
Cust
to 1,800/4,000   -5,625 5,625  
Abstr
Cust 2,200/4,000                -6,875                    6,875
to

11-177
Copyright © 2017 McGraw-Hill Education. All rights reserved. No reproduction or distribution without the prior written consent of
McGraw-Hill Education.
Close
              0          0  $30,625 $19,375
Clerica
b.   Cust Abstract Close
l
Dept Allocation $40,000 $10,000    
Cust
to
1,600/5,600 2,857 -2.857    
Cleri
c
Cust
to 1,800/5,600   -3,214 3,214  
Abstr
Cust
to 2,200/5,600   -3,929   3,929
Close
Cleri
c to 50/75 -28,571   28,571  
Abstr
Cleri
c to 25/75 -14,286                            14,286
Close
              0          0 $31,785 $18,215
 
AACSB: Analytical Thinking
AICPA: FN Measurement
Blooms: Apply
Difficulty: 2 Medium
Gradable: manual
Learning Objective: 11-03 Allocate service department costs using the step method.
Topic: Methods of Allocating Service Department Costs
 

11-178
Copyright © 2017 McGraw-Hill Education. All rights reserved. No reproduction or distribution without the prior written consent of
McGraw-Hill Education.
114. Ardvark Industries has two production departments, Assembly and Finishing and three Service
Departments, Personnel, Maintenance, and Cafeteria. Data relevant to Ardvark are:
 
Direc Perso Mainten Cafet Asse Finis
Dept
t Cost nnel ance eria mbly hing
Personn $500,
  0.10   0.70 0.20
el 000
Mainten 420,0
      0.80 0.20
ance 00
Cafeteri 200,0
0.20 0.20   0.30 0.30
a 00
Finishin 150,0
         
g 00
Assembl 380,0
         
y 00

Required:

Allocate the service department costs of Ardvark Industries using the step-method of cost allocation.  
 

Since the Cafeteria provides the largest percentage of service to other service departments, it should be
allocated first in the step-method of allocation.
 

Cafet Perso Mainte Finis Asse


  Total
eria nnel nance hing mbly
Direct $200, $500, $420,00 $150, $380, $1,650,
cost 000 000 0 000 000 000
1st step
-
(.20, 40,00 60,00 60,00
200,0 40,000  
20, .30, . 0 0 0
00
30)
2nd step -
378,0 108,0
(.10, .70,   540,0 54,000  
00 00
.20) 00
-
3rd step $411, $102,
    $514,00  
(.80, .20) 200 800
0
Departm
           
ental
$999, $650, $1,650,
Total $0 $0 $0
200 800 000
 
AACSB: Analytical Thinking
AICPA: FN Measurement
Blooms: Analyze

11-179
Copyright © 2017 McGraw-Hill Education. All rights reserved. No reproduction or distribution without the prior written consent of
McGraw-Hill Education.
Difficulty: 3 Hard
Gradable: manual
Learning Objective: 11-03 Allocate service department costs using the step method.
Topic: Methods of Allocating Service Department Costs
 

11-180
Copyright © 2017 McGraw-Hill Education. All rights reserved. No reproduction or distribution without the prior written consent of
McGraw-Hill Education.
11-181
Copyright © 2017 McGraw-Hill Education. All rights reserved. No reproduction or distribution without the prior written consent of
McGraw-Hill Education.
115. Mena Corporation has two production departments, Assembly and Finishing and three service departments,
Personnel, Maintenance, and Cafeteria. Data relevant to Mena are:
 
Direct Person Mainten Cafet Assem Finis
Dept
cost nel ance eria bly hing
Personne $500,
  0.10   0.70 0.20
l 000
Mainten 420,0
      0.80 0.20
ance 00
200,0
Cafeteria 0.20 0.20   0.30 0.30
00
Finishin 150,0
         
g 00
Assembl 380,0
         
y 00

Assembly and Finishing work on two jobs during the month: Job 100 and 101. Costs are allocated to jobs
based on machine hours in assembly and labor hours in Finishing. The machine and labor hours worked in
each department are as follows:
 
    Assembly Finishing
Job 100 Labor Hours 200 800
  Machine Hours 1,000 200
Job 101 Labor Hours 100 900
  Machine Hours 500 100

Required:

Determine the amount of service department costs to be allocated to Jobs 100 and 101. Mena allocates
service department costs to production departments using the direct-method of allocation.  
 

First, Allocate the Service Department costs to the Production Departments:

  Personnel Maintenance Cafeteria Assembly Finishing


Direct costs 500,000 420,000 200,000 380,000 150,000 $1,650,000
(7/9; 2/9) -500,000     388,889 111,111
(8; 2)   -420,000   336,000 84,000
(3/6; 3/6)     -200,000 100,000 100,000
TOTAL 0 0 0 $1,204,889 $445,111 $1,650,000

Then, allocate the production department costs to Job 100 and 101:
 
  Assembly Finishing Total  

11-182
Copyright © 2017 McGraw-Hill Education. All rights reserved. No reproduction or distribution without the prior written consent of
McGraw-Hill Education.
Labor hours        
Job 100 200 800 1000  
Job 101 100 900 1000  
Total 300 1700 2000  
Machine hours        
Job 100 1000 200 1200  
Job 101 500 100 600  
Total 1500 300 1800  
  Assembly Finishing    
Total service
       
department
Cost in: $824,889 $295,111 $1,120,000  
Machine hours 1,500      
Labor hours   1,700    
Cost per hour $549,93 $173,59    
  Job 100 Job 101 Total  
Machine hours        
1000 × $549.93;
$549,930 $274,965 $824,895  
500 × 549.93
Labor hours        
800 × $173.59;
 $138,872 $156,231    295,103  
900 × $173.59
Total cost **
$688,802 $431,196 $1,119,998
allocated *
***rounded        
 
AACSB: Analytical Thinking
AICPA: FN Measurement
Blooms: Analyze
Difficulty: 3 Hard
Gradable: manual
Learning Objective: 11-02 Allocate service department costs using the direct method.
Topic: Methods of Allocating Service Department Costs
 

11-183
Copyright © 2017 McGraw-Hill Education. All rights reserved. No reproduction or distribution without the prior written consent of
McGraw-Hill Education.
116. Boston Corporation has two production Departments: Assembly and Machining and two service
departments: Personnel and Cafeteria. Direct costs for each department and the proportion of service
costs used by the various departments for the month of July, 2016 are as follows:
 
    Proportion of Services Used by:
Departm Direct Personn Cafeter Machini Assemb
ent costs el ia ng ly
$30,00
Personnel   0.40 0.30 0.30
0
$50,00
Cafeteria 0.20   0.50 0.30
0
Machinin $80,00
       
g 0
$70,00
Assembly        
0

Required:

Compute the allocation of service department costs to producing departments for July, 2016 using the
step-method.  
 

Since the Personnel department provides the largest percentage of service to other service departments,
it should be allocated first in the step-method of allocation.

Personn Cafeteri Machini Assembl


  Total
el a ng y
Direc $230,00
$30,000 $50,000 $80,000 $70,000
t cost 0
Step
1
(.40, -30,000 12,000 9,000 9,000  
.30, .
30)
Step
2      38,75
   -62,000     23,250  
(5/8, 0
3/8)
$230,00
Total            $0          $0 $127,750 $102,250
0
 
AACSB: Analytical Thinking
AICPA: FN Measurement
Blooms: Analyze
Difficulty: 2 Medium
Gradable: manual
Learning Objective: 11-03 Allocate service department costs using the step method.

11-184
Copyright © 2017 McGraw-Hill Education. All rights reserved. No reproduction or distribution without the prior written consent of
McGraw-Hill Education.
Topic: Methods of Allocating Service Department Costs
 

11-185
Copyright © 2017 McGraw-Hill Education. All rights reserved. No reproduction or distribution without the prior written consent of
McGraw-Hill Education.
11-186
Copyright © 2017 McGraw-Hill Education. All rights reserved. No reproduction or distribution without the prior written consent of
McGraw-Hill Education.
117. Data Master is a computer software consulting company. Its three major functional areas are computer
programming, information systems consulting, and software training. Cynthia Moore, a pricing analyst
in the Accounting Department, has been asked to develop total costs for the functional areas. These
costs will be used as a guide in pricing a new contract. In computing these costs, Moore is considering
three different methods of allocating overhead costs-the direct method, the step method, and the
reciprocal method. Moore assembled the following data on overhead from its two service departments,
the Information Systems Department and the Facilities Department.
 
Service
  User Departments  
Departments
Comp
Info
uter
  Syste Facili Consu Trai
Progra Total
ms ties lt ning
m
Budget
ed $50,0 $25,00 $75,00 $110,0 $85,0 $345,0
Overhe 00 0 0 00 00 00
ad
Info
System   400 1,100 600 900 3,000
s (hrs)
Faciliti
200,0 400,00 600,00 800,0 2,000,0
es (Sq  
00 0 0 00 00
ft)

Information systems is allocated on the basis of hours of computer usage; facilities are allocated on the
basis of floor space.

Required:

Allocate the service department costs to the user departments using the reciprocal method. Round to the
nearest dollar.  
 

Computer Programming: $100,929; Consulting: $130,271; Training: $113,800

IS = $50,000 + [(200/2,000) × Fac]


Fac = $25,000 + [(400/3,000) × IS]
IS = $53,209; Fac = $32,095
 

    IS Fac CP Cons Train


Dep Allocatio $50,0 $25,0 $75,00 $110,0 $85,00
t n 00 00 0 00 0
IS
400/3,00
to -7,095 7,095      
0
Fac

11-187
Copyright © 2017 McGraw-Hill Education. All rights reserved. No reproduction or distribution without the prior written consent of
McGraw-Hill Education.
IS -
1,100/3,0
to 19,51   19,510    
00
CP 0
IS
-
to 600/3,00
10,64     10,642  
Con 0
2
s
IS
-
to 900/3,00
15,96       15,962
Trai 0
2
n
Fac
200/2,00
to 3,209 -3,209      
0
IS
Fac
400/2,00
to   -6,419 6,419    
0
CP
Fac
to 600/2,00
  -9,629   9,629  
Con 0
s
Fac
-
to 800/2,00                             12,83
12,83             
Trai 0     8
8
n
                    $100,9 $130,2 $113,8
   
0 0 29 71 00
 
AACSB: Analytical Thinking
AICPA: FN Measurement
Blooms: Apply
Difficulty: 3 Hard
Gradable: manual
Learning Objective: 11-04 Allocate service department costs using the reciprocal method.
Topic: Methods of Allocating Service Department Costs
 

11-188
Copyright © 2017 McGraw-Hill Education. All rights reserved. No reproduction or distribution without the prior written consent of
McGraw-Hill Education.
11-189
Copyright © 2017 McGraw-Hill Education. All rights reserved. No reproduction or distribution without the prior written consent of
McGraw-Hill Education.
118. Jack Donaldson owns and operates Jack's Abstracting Service. Jack's two revenue generating operations
(Abstracting Services and Closing Services) are supported by two service departments: Clerical and
Custodial. Costs in the service departments are allocated in the following order using the designated
allocation bases.
Clerical: number of transactions processed.
Custodial: square footage of space occupied.
Average and expected activity levels for next month are as follows:

Number of Square Expected


 
Transactions  Footage   Costs
Abstract services 50 1,800  
Closing services 25 2,200  
Clerical   1,600 $40,000
Custodial 5   10,000

Required:

Use the reciprocal method to allocate the service department costs to the revenue generating
departments. Provide the total costs for the revenue departments.  
 

Abstract: $31,364; Close: $18,636

Clerical = $40,000 + (1,600/5,600 × Custodial)


Custodial = $10,000 + (5/80 × Clerical)
Clerical = $43,636.36; Custodial = $12,727.27
 

Abstrac
a.   Clerical Cust Close
t
Dept Allocation $40,000 $10,000    
Cleri
c to 5/80 -2,727 2,727    
Cust
Cleri
c to 50/80 -27,273   27,273  
Abstr
Cleri
c to 25/80 -13,636     13,636
Close
Cust
to
1,600/5,600 3,636 -3,636    
Cleri
c
Cust 1,800/5,600   -4,091 4,091  
to

11-190
Copyright © 2017 McGraw-Hill Education. All rights reserved. No reproduction or distribution without the prior written consent of
McGraw-Hill Education.
Abstr
Cust
to 2,200/5,600               -5,000                  5,000
Close
              0          0 $31,364 $18,636
 
AACSB: Analytical Thinking
AICPA: FN Measurement
Blooms: Apply
Difficulty: 3 Hard
Gradable: manual
Learning Objective: 11-04 Allocate service department costs using the reciprocal method.
Topic: Methods of Allocating Service Department Costs
 

11-191
Copyright © 2017 McGraw-Hill Education. All rights reserved. No reproduction or distribution without the prior written consent of
McGraw-Hill Education.
11-192
Copyright © 2017 McGraw-Hill Education. All rights reserved. No reproduction or distribution without the prior written consent of
McGraw-Hill Education.
119. Yellville Regional Hospital is a small hospital with two service departments and three revenue areas:
 
Direct Laundry
Service Dept Costs Sq Ft Pounds
   Housekeeping $80,000 - 16,000
   Laundry $132,000 500  
Revenue Areas      
   Surgery $400,000 1,500 48,000
   Semiprivate Rooms $200,000 2,000 24,000
   Maternity $150,000 1,000 12,000

The hospital wants to allocate the service department costs to the revenue areas. Housekeeping is
allocated based on square footage; Laundry is allocated based on pounds of laundry. The normal
capacity for Surgery is 200 hours per month; normal capacity for semiprivate rooms is 600 patient days;
and normal capacity for maternity is 200 patient days.

Required:

Determine the overhead rate for the three revenue areas. Allocate the service department costs to the
revenue areas using the reciprocal method.  
 

Surgery: $2,495.61/hr; SemiPrivate rooms: $458.75/day; Maternity: $938.13/day


HK = $80,000 + 16/100 × Laund
Laund = $132,000 + 500/5,000 × HK
HK = $102,764; Laundry = $142,276
 

House
    Laund Surg SemiP Mat
k
Allocati $80,00 $132,0 $400,0 $200,0 $150,0
Dept
on 0 00 00 00 00
HK
to 500/5,00 -
10,276      
Laun 0 10,276
d
HK
1,500/5,0 -
to   30,829    
00 30,829
Surg
HK
to 2,000/5,0 -
    41,106  
semi 00 41,106
P
HK
1,000/5,0 -
to       20,553
00 20,553
Mat

11-193
Copyright © 2017 McGraw-Hill Education. All rights reserved. No reproduction or distribution without the prior written consent of
McGraw-Hill Education.
Laun
-
d to 16/100 22,764      
22,764
HK
Laun
-
d to 48/100   68,293    
68,293
Surg
Laun
d to -
24/100     34,146  
Semi 34,146
P
Laun
             -                17,07
d to 12/100           
   17,073   3
Mat
           $499,1 $275,2 $187,6
            0
0 22 52 26

Surgery: $499,122/200 hrs = $2,495.61/hr


SemiPrivate Rooms: $275,252/600 patient days = $458.75/day
Maternity: $187,626/200 patient days = $938.13/day
 
AACSB: Analytical Thinking
AICPA: FN Measurement
Blooms: Analyze
Difficulty: 2 Medium
Gradable: manual
Learning Objective: 11-04 Allocate service department costs using the reciprocal method.
Topic: Methods of Allocating Service Department Costs
 

11-194
Copyright © 2017 McGraw-Hill Education. All rights reserved. No reproduction or distribution without the prior written consent of
McGraw-Hill Education.
120. Franklin Corporation has three operating departments (Fabricating, Assembly, and Finishing) and two
service departments (Custodial and Administrative). The following information has been provided:
 
Custodi Admi Fabrica Assem Finish
 
al n ting bly ing
$250,00 $400,0
Dept Costs -- -- --
0 00
# employees 10 -- 80 100 60
Sq ft -- 15,000 30,000 35,000 20,000
Allocations are based on the
     
following:
Square
    Custodial:        
feet
Number
    Administr of
       
ative: employ
ees

Required:

Franklin has been approached by Sparkle Cleaning to outsource the custodial service. Assuming all
costs are variable, what is the relevant cost of the custodial department to compare with the Sparkle
Cleaning bid?  
 

$267,606

Custodial = $250,000 + (10/(10 + 80 + 100 + 60) × Admin)


Admin = $400,000 + (15,000/(15,000 + 30,000 + 35,000 + 20,000) × Custodial)
Custodial = $267,606
Admin: $400,000 + 40,141 = $440,141 [$400,000 + (.15 × $267,606 = $40,141)]
Custodial: $250,000 + 17,606 = $267,606 [$250,000 + (.04 × $440,141 = $17,606)]

 
AACSB: Analytical Thinking
AICPA: FN Decision Making
Blooms: Analyze
Difficulty: 2 Medium
Gradable: manual
Learning Objective: 11-05 Use the reciprocal method approach for outsourcing decisions.
Topic: Methods of Allocating Service Department Costs
 

11-195
Copyright © 2017 McGraw-Hill Education. All rights reserved. No reproduction or distribution without the prior written consent of
McGraw-Hill Education.
121. The Joplin Company conducts a simple chemical process in Department #1, which produces three
separate items: A, K, and H. A is processed further in Department #2. K is processed further in
Department #3. Product H is a by-product, to be accounted for by the cost reduction method. The
following information relates to September:

Department #1's costs $420,000.


Department #2's costs $150,000.
Department #3's costs $60,000.

A: 25,000 pounds completed; 23,500 pounds sold for $12 per pound.
K: 75,000 pounds completed; 70,000 pounds sold for $7.50 per pound.
H: 10,000 pounds completed; 10,000 pounds sold for $1.50 per pound. (There are shipping costs of
$0.30 per pound.)

There were no September 1st inventories.

Required:

Prepare a schedule to show the computation for the unit costs per pound for Products A, K, and H
assuming Joplin uses the estimated net realizable value method to allocate joint costs to the main
products.  
 

Uni
Adde Alloca
Sales Total t
Uni d ted
Prod Value NRV Costs Cos
ts Costs Costs
uct ts
25,0 $300,0 $150,0 $150,0 $93,79 $243,7 $9,
A
00 00 00 00 3 93 75
75,0 562,50 502,50 314,20 374,20 4.9
K 60,000
00 0 0 7 7 9
10,0 1.5
H 15,000 3,000 12,000 12,000 15,000
00 0

Allocated Costs (AC)

A: $150,000/652,500 × 408,000 = $93,793


K: $502,500/652,500 × 408,000 = $314,207
H: $15,000 - 3,000 = $12,000
 
AACSB: Analytical Thinking
AICPA: FN Measurement
Blooms: Apply
Difficulty: 2 Medium
Gradable: manual

11-196
Copyright © 2017 McGraw-Hill Education. All rights reserved. No reproduction or distribution without the prior written consent of
McGraw-Hill Education.
Learning Objective: 11-07 Allocate joint costs using the net realizable value method.
Topic: Joint Cost Allocation Methods
 
122. Simpson Manufacturing Enterprises uses a joint production process that produces three products at the
split-off point. Joint production costs during April were $720,000. The company uses the net realizable
value method for allocating joint costs. Product information for April was as follows:
 
  Product
  R S T
Gallons produced 2,500 5,000 7,500
Sales prices per
     
gallon:
   At the split-off $100 $80 $20
   After further
$150 $115 $30
processing
Costs to process after
$150,000 $150,000 $100,000
split-off

Required:

a. Assume that all three products are main products and that they can be sold at the split-off point or
processed further, whichever is economically beneficial to Simpson. Allocate the joint costs to the three
products.
b. Assume that Simpson uses the physical quantities method to allocate the joint costs. How much
would be allocated to each of the three products?  
 

a. R: $218,182; S: $370,909; T: $130,909


b. R: $120,000; S: $240,000; T: $360,000

a. R: Sell now: 2,500 × $100 = $250,000; process further: (2,500 × $150) - $150,000 = $225,000; Sell
now
S: Sell now: 5,000 × $80 = $400,000; process further: (5,000 × $115) - $150,000 = $425,000; Process
further
T: Sell now: 7,500 × $20 = $150,000; process further: (7,500 × $30) - $100,000 = $125,000; Sell now

Joint cost allocation:

R: [$250,000/($250,000 + 425,000 + 150,000)] × $720,000 = $218,182


S: [$425,000/($250,000 + 425,000 + 150,000)] × $720,000 = $370,909
T: [$150,000/($250,000 + 425,000 + 150,000)] × $720,000 = $130,909
b. R: [2,500/(2,500 + 5,000 + 7,500)] × $720,000 = $120,000
S: [5,000/(2,500 + 5,000 + 7,500)] × $720,000 = $240,000
T: [7,500/(2,500 + 5,000 + 7,500)] × $720,000 = $360,000

 
AACSB: Analytical Thinking
AICPA: FN Measurement

11-197
Copyright © 2017 McGraw-Hill Education. All rights reserved. No reproduction or distribution without the prior written consent of
McGraw-Hill Education.
Blooms: Apply
Difficulty: 2 Medium
Gradable: manual
Learning Objective: 11-07 Allocate joint costs using the net realizable value method.
Learning Objective: 11-08 Allocate joint costs using the physical quantities method.
Topic: Joint Cost Allocation Methods
 
123. Clean-Burn, Inc. is a small petroleum company that acquires high-grade crude oil from low-volume
production wells owned by individuals and small partnerships. The crude oil is processed in a single
refinery into Two Oil, Six Oil, and impure distillates. Clean-Burn does not have the technology or
capacity to process these products further and sells most of its output each month to major refineries.
There were no inventories on November 1.

Crude oil acquired and placed into


$5,000,000
production
Direct labor and related costs 2,000,000
Refinery overhead 3,000,000

Production and sales


Two Oil, 300,000 barrels produced; 280,000 barrels sold at $20 each.
Six Oil, 240,000 barrels produced; 220,000 barrels sold at $30 each.
Distillates, 120,000 barrels produced and sold at $15 per barrel.

Required:

a. Allocate the joint costs to the products using the physical quantities method.
b. Allocate the joint costs to the products using the net realizable value method.  
 

a. Two Oil: $4,545,455; Six Oil: $3,636,364; Distillates: $1,818,181


b. Two Oil: $4,000,000; Six Oil: $4,800,000; Distillates: $1,200,000

Total joint cost: $5,000,000 + 2,000,000 + 3,000,000 = $10,000,000


a. Two Oil: [300,000/(300,000 + 240,000 + 120,000)] × $10,000,000 = $4,545,455
Six Oil: [240,000/(300,000 + 240,000 + 120,000)] × $10,000,000 = $3,636,364
Distillates: [120,000/(300,000 + 240,000 + 120,000)] × $10,000,000 = $1,818,181
b. Net realizable values: Two Oil: 300,000 × $20 = $6,000,000; Six Oil: 240,000 × $30 = $7,200,000;
Distillates: 120,000 × $15 = $1,800,000; Total = $15,000,000
Two Oil: ($6,000,000/$15,000,000) × $10,000,000 = $4,000,000
Six Oil: ($7,200,000/$15,000,000) × $10,000,000 = $4,800,000
Distillates: ($1,800,000/$15,000,000) × $10,000,000 = $1,200,000

 
AACSB: Analytical Thinking
AICPA: FN Measurement
Blooms: Analyze
Difficulty: 2 Medium
Gradable: manual
Learning Objective: 11-07 Allocate joint costs using the net realizable value method.
Learning Objective: 11-08 Allocate joint costs using the physical quantities method.

11-198
Copyright © 2017 McGraw-Hill Education. All rights reserved. No reproduction or distribution without the prior written consent of
McGraw-Hill Education.
Topic: Joint Cost Allocation Methods
 

11-199
Copyright © 2017 McGraw-Hill Education. All rights reserved. No reproduction or distribution without the prior written consent of
McGraw-Hill Education.
124. Smokey Enterprises buys Liquid Charcoal for $0.80 a gallon. At the end of processing in department 1,
the liquid charcoal splits off into Products U, V, and W. Product U is sold at the split-off point, with no
further processing. Products V and W require further processing before they can be sold; Product V is
processed in Department 2, and Product W is processed in Department 3. Following is a summary of
costs and other related data for the most recent accounting period:
 
  Department
  1 2 3
Cost of liquid charcoal $104,000    
Direct labor 16,000 45,000 65,000
Manufacturing
10,000 27,000 49,000
overhead
  Products
  U V W
Gallons sold 20,000 30,000 50,000
Gallons on hand end of
15,000 0 15,000
period
Sales in dollars $30,000 $96,000 $142,000

There were no beginning inventories and there was no liquid charcoal on hand at the end of the period.
All gallons on hand in ending inventory were complete as to processing. Smokey uses the estimated net
realizable value method of allocating joint costs.

Required:

a. Determine the product cost for U, V, and W, assuming the physical quantity method is used to
allocate joint costs.
b. Determine the product cost for U, V, and W, assuming the net realizable value method is used to
allocate joint costs.  
 

a. U: $35,000; V: $30,000; W: $65,000


b. U: $20,489; V: $37,466; W: $72,045

Joint Cost = $104,000 + 16,000 + 10,000 = $130,000


a. U: [(20,000 + 15,000)/(20,000 + 15,000 + 30,000 + 50,000 + 15,000)] × $130,000 = $35,000
V: [30,000/(20,000 + 15,000 + 30,000 + 50,000 + 15,000)] × $130,000 = $30,000
W: [(50,000 + 15,000)/(20,000 + 15,000 + 30,000 + 50,000 + 15,000)] × $130,000 = $65,000
b. Selling Prices: U: $30,000/20,000 = $1.50; V: $96,000/30,000 = $3.20; W: $142,000/50,000 = $2.84;
Net realizable values: U: (20,000 + 15,000) × $1.50 = $52,500; V: $96,000; W: (50,000 + 15,000) ×
$2.84 = $184,600; Total NRV = $333,100
U: (52,500/333,100) × $130,000 = $20,489
V: (96,000/333,100) × $130,000 = $37,466
W: (184,600/333,100) × $130,000 = $72,045

 
AACSB: Analytical Thinking

11-200
Copyright © 2017 McGraw-Hill Education. All rights reserved. No reproduction or distribution without the prior written consent of
McGraw-Hill Education.
AICPA: FN Measurement
Blooms: Analyze
Difficulty: 2 Medium
Gradable: manual
Learning Objective: 11-07 Allocate joint costs using the net realizable value method.
Learning Objective: 11-08 Allocate joint costs using the physical quantities method.
Topic: Joint Cost Allocation Methods
 
125. The Wang Company conducts a simple chemical process in Department #1, which produces three
separate items: A, B, and C. A is processed further in Department #2. B is processed further in
Department #3. Product C can be sold immediately. The following information relates to October:

Department #1's costs $540,000.


Department #2's costs $120,000.
Department #3's costs $300,000.

A: 25,000 pounds completed; 23,500 pounds sold for $12 per pound.
B: 75,000 pounds completed; 70,000 pounds sold for $7.50 per pound.
C: 50,000 pounds completed; 46,000 pounds sold for $5.00 per pound.

There were no October 1st inventories.

Required:

a. Allocate the joint process costs to Products A, B, and C assuming the estimated net realizable value
method is used.
b. Allocate the joint process costs to products A, B, and C assuming the physical quantities method is
used.  
 

a. A: $140,361; B: $204,693; C: $194,946


b. A: $90,000; B: $270,000; C: $180,000

a. Estimated net realizable values: A: 25,000 × $12 = $300,000 - $120,000 = $180,000; B: 75,000 ×
$7.50 = $562,500 - $300,000 = $262,500; C: 50,000 × $5 = $250,000; Total NRV: $180,000 + 262,500
+ 250,000 = $692,500

A: ($180,000/$692,500) × $540,000 = $140,361


B: ($262,500/$692,500) × $540,000 = $204,693
C: ($250,000/$692,500) × $540,000 = $194,946

b. A: [25,000/(25,000 + 75,000 + 50,000)] × $540,000 = $90,000


B: [75,000/(25,000 + 75,000 + 50,000)] × $540,000 = $270,000
C: [50,000/(25,000 + 75,000 + 50,000)] × $540,000 = $180,000

 
AACSB: Analytical Thinking
AICPA: FN Measurement
Blooms: Analyze
Difficulty: 2 Medium

11-201
Copyright © 2017 McGraw-Hill Education. All rights reserved. No reproduction or distribution without the prior written consent of
McGraw-Hill Education.
Gradable: manual
Learning Objective: 11-07 Allocate joint costs using the net realizable value method.
Learning Objective: 11-08 Allocate joint costs using the physical quantities method.
Topic: Joint Cost Allocation Methods
 
126. The Marketplace Corporation produces two consumer products and a by-product. Zylon is ready for sale
after split-off, while Qytol must be further processed. The by-product is a heavy residue in the bottom of
the vat. The net realizable value of the by-product is credited against the $565,000 joint cost of the
Heating Department. Volume and cost data for February is as follows:

Gallons Selling Additional


 
Produced Price Processing
Zylon 200,000 $2.00 0
Qytol 400,000 1.10 $40,000
By-Product 5,000 0.50 0

Required:

a. Allocate the Heating Department cost to the products using the physical quantities method.
b. Allocate the Heating Department cost to the products using the workback method.  
 

a. Zylon: $187,500; Qytol: $375,000


b. Zylon: $281,250; Qytol: $281,250

a. Joint costs to allocate: $565,000 - (5,000 × $0.50) = $562,500


Zylon: [200,000/(200,000 + 400,000)] × $562,500 = $187,500
Qytol: [400,000/(200,000 + 400,000)] × $562,500 = $375,000
b. Estimated net realizable values: Zylon: 200,000 × $2 = $400,000; Qytol: (400,000 × $1.10) - 40,000
= $400,000
Zylon: [$400,000/($400,000 + 400,000)] × $562,500 = $281,250
Qytol: [$400,000/($400,000 + 400,000)] × $562,500 = $281,250

 
AACSB: Analytical Thinking
AICPA: FN Measurement
Blooms: Analyze
Difficulty: 2 Medium
Gradable: manual
Learning Objective: 11-07 Allocate joint costs using the net realizable value method.
Learning Objective: 11-08 Allocate joint costs using the physical quantities method.
Topic: Joint Cost Allocation Methods
 

11-202
Copyright © 2017 McGraw-Hill Education. All rights reserved. No reproduction or distribution without the prior written consent of
McGraw-Hill Education.
127. The Delicious Canning Company processes tomatoes into ketchup, tomato juice, and canned tomatoes.
During the summer, the joint costs of processing the tomatoes were $420,000. There was no beginning
or ending inventories for the summer. Production and sales value information for the summer were as
follows:
 
Product Cases Additional Costs Selling Price
Ketchup 100,000 $3.00 per case $28 per case
Juice 150,000 5.00 per case $25 per case
Canned 200,000 2.50 per case 10 per case

Required:

a. Determine the amount allocated to each product if the estimated net realizable value method is used.
b. Determine the amount allocated to each product if the physical units method is used.  
 

a. Ketchup: $150,000; Juice: $180,000; Canned: $90,000


b. Ketchup: $93,333; Juice: $140,000; Canned: $186,667

a. Estimated net realizable value: Ketchup: 100,000 × ($28 - $3) = $2,500,000; Juice: 150,000 × ($25 -
$5) = $3,000,000; Canned: 200,000 × ($10 - $2.50) = $1,500,000
Ketchup: [$2,500,000/($2,500,000 + 3,000,000 + 1,500,000)] × $420,000 = $150,000
Juice: [$3,000,000/($2,500,000 + 3,000,000 + 1,500,000)] × $420,000 = $180,000
Canned: [$1,500,000/($2,500,000 + 3,000,000 + 1,500,000)] × $420,000 = $90,000
b. Use number of cases for physical quantity.
Ketchup: [100,000/(100,000 + 150,000 + 200,000)] × $420,000 = $93,333
Juice: [150,000/(100,000 + 150,000 + 200,000)] × $420,000 = $140,000
Canned: [200,000/(100,000 + 150,000 + 200,000)] × $420,000 = $186,667

 
AACSB: Analytical Thinking
AICPA: FN Measurement
Blooms: Analyze
Difficulty: 2 Medium
Gradable: manual
Learning Objective: 11-07 Allocate joint costs using the net realizable value method.
Learning Objective: 11-08 Allocate joint costs using the physical quantities method.
Topic: Joint Cost Allocation Methods
 

11-203
Copyright © 2017 McGraw-Hill Education. All rights reserved. No reproduction or distribution without the prior written consent of
McGraw-Hill Education.
128. The Joplin Company conducts a simple chemical process in Department #1, which produces three
separate items: A, K, and H. A is processed further in Department #2. K is processed further in
Department #3. Product H is a by-product, to be accounted for by the other revenue method. The
following information relates to September:

Department #1's costs $420,000.


Department #2's costs $150,000.
Department #3's costs $60,000.

A: 25,000 pounds completed; 23,500 pounds sold for $12 per pound.
K: 75,000 pounds completed; 70,000 pounds sold for $7.50 per pound.
H: 10,000 pounds completed; 10,000 pounds sold for $1.50 per pound. (There are shipping costs of
$0.30 per pound.)

There were no September 1st inventories.

Required:

Prepare a schedule to show the computation for the unit costs per pound for Products A, K, and H
assuming Joplin uses the physical quantities method to allocate joint costs to the main products.  
 

Allocate
Added Total Unit
d
Produc Units Costs Costs Costs
Costs
t
A 25,000 $150,000 $105,000 $255,000 $10.20
K 75,000 60,000 315,000 375,000 5.00

Joint costs: $420,000 - 0 = $420,000


Allocated costs: A: [25,000/(25,000 + 75,000)] × $420,000 = $105,000;
K: [75,000/(25,000 + 75,000)] × $420,000 = $315,000
 
AACSB: Analytical Thinking
AICPA: FN Measurement
Blooms: Apply
Difficulty: 2 Medium
Gradable: manual
Learning Objective: 11-08 Allocate joint costs using the physical quantities method.
Topic: Joint Cost Allocation Methods
 

11-204
Copyright © 2017 McGraw-Hill Education. All rights reserved. No reproduction or distribution without the prior written consent of
McGraw-Hill Education.
129. Highlands, Inc. operates a sawmill facility. The company accounts for the sawdust that results from the
primary sawing operation as a by-product. The sawdust is sold to another company at a price of $1.00
per hundred cubic feet. Normally, sales revenue from the sawdust is $21,200 per month. The sawdust is
charged to inventory at $2.20 per hundred cubic feet, although there is no direct cost to process it.
As an alternative, Highlands can rent equipment that will process the dust into imitation logs for
fireplaces. These logs sell for $25.00 per hundred to wholesalers, who package and add scent to them.
75 logs can be produced from 100 cubic feet of sawdust.
Cost of the equipment to produce these logs and the additional personnel required to operate the
equipment are $360,000 per month, regardless of the output.

Required:

A) Should Highlands sell the sawdust for $1.00 per hundred cubic feet or process it into imitation logs?
Support your answer with the appropriate calculations.  
 

Process them further.

Sell now at $100 per hundred = $21,200


Process further = [$25($21,200/$1.00)(75/100)] - $360,000 = $397,500 - 360,000 = $37,500
Decision: Process further for an incremental income of $37,500 - $21,200 = $16,300

 
AACSB: Analytical Thinking
AICPA: FN Decision Making
Blooms: Analyze
Difficulty: 1 Easy
Gradable: manual
Learning Objective: 11-09 Explain how cost data are used in the sell-or-process-further decision.
Topic: Deciding Whether to Sell Goods Now or Process Them Further
 

11-205
Copyright © 2017 McGraw-Hill Education. All rights reserved. No reproduction or distribution without the prior written consent of
McGraw-Hill Education.
11-206
Copyright © 2017 McGraw-Hill Education. All rights reserved. No reproduction or distribution without the prior written consent of
McGraw-Hill Education.
130. Voorhees Manufacturing Corporation produces three products in a joint process.
Additional information is as follows:
 
  O P Q Total
Units
42,000 50,000 8,000 100,000
produced
Sales value at
$250,000 $50,000 $20,000 $320,000
split off
Additional
costs if
$18,000 $30,000 $10,000 $58,000
processed
further
Sales value if
processed $290,000 $70,000 $25,000 $385,000
further
Joint costs       $300,000
Product
weights in 84,000 150,000 8,000 242,000
pounds

Required:

(a) Determine which products should be sold at split-off and which should be processed further.
(b) Assuming Voorhees makes decisions that are in its best interests for overall profitability, what
would be the company's gross margin?  
 

(a) Only product O should be processed further. It is the only product where the incremental processing
costs are less than the incremental revenues.
 

  O P Q
Sales value at split-off $250,000 $50,000 $20,000
Sales value if
$290,000 $70,000 $25,000
processed Further
Additional processing
   18,000  30,000   10,000
cost
Net revenue of
272,000 40,000 15,000
processing
Net advantage
(disadvantage of $22,000 ($10,000) ($5,000)
processing further)

(b) If Voorhees processes O further and sells P and Q at split-off, its gross margin will be $42,000.
 
Sales ($290,000 + $50,000 + $20,000) $360,000

11-207
Copyright © 2017 McGraw-Hill Education. All rights reserved. No reproduction or distribution without the prior written consent of
McGraw-Hill Education.
Additional processing costs $18,000
Joint costs $300,000
Gross margin  $42,000
 
AACSB: Analytical Thinking
AICPA: FN Measurement
Blooms: Analyze
Difficulty: 2 Medium
Gradable: manual
Learning Objective: 11-09 Explain how cost data are used in the sell-or-process-further decision.
Topic: Deciding Whether to Sell Goods Now or Process Them Further
 

11-208
Copyright © 2017 McGraw-Hill Education. All rights reserved. No reproduction or distribution without the prior written consent of
McGraw-Hill Education.
131. Ridgeline Enterprises produces three products in a joint process. Products A and B were processed
further. Additional information is as follows:

  A B C Total
Units
42,000 50,000 8,000 100,000
produced
Sales value at
$250,000 $30,000 $20,000 $300,000
split-off
Additional
costs if
$18,000 $30,000 $0 $48,000
processed
further
Sales value if
processed $290,000 $70,000 $0 $360,000
further
Joint Costs       $200,000
Product
Weight in 168,000 300,000 32,000 500,000
pounds

Required:

(a) Allocate the joint costs, assuming that all products are joint products and joint-costs are allocated
using the physical-measures method.
(b) Allocate the joint costs using the physical-measures method, assuming that product C is considered
a by-product, whose sales value is deducted from the total joint costs.  
 

(a)
 

  A B C Total
Product
Weight in 168,000 300,000 32,000 500,000
pounds
Percent 33.6% 60% 6.4% 100.00%
Cost allocated $67,200 $120,000 $12,800 $200,000

(b)

  A B Total
Product Weight in
168,000 300,000 468,000
pounds
Percent 35.9% 64.1% 100%
Cost allocated $64,620 $115,380 $180,000

11-209
Copyright © 2017 McGraw-Hill Education. All rights reserved. No reproduction or distribution without the prior written consent of
McGraw-Hill Education.
 
AACSB: Analytical Thinking
AICPA: FN Measurement
Blooms: Analyze
Difficulty: 2 Medium
Gradable: manual
Learning Objective: 11-09 Explain how cost data are used in the sell-or-process-further decision.
Topic: Deciding Whether to Sell Goods Now or Process Them Further
 

11-210
Copyright © 2017 McGraw-Hill Education. All rights reserved. No reproduction or distribution without the prior written consent of
McGraw-Hill Education.
11-211
Copyright © 2017 McGraw-Hill Education. All rights reserved. No reproduction or distribution without the prior written consent of
McGraw-Hill Education.
132. Geneva Powder Company produces body powders in batches. Each type of powder can be sold in its
current condition or processed further and specialized for high priced department stores. Data
concerning the various products appear below. Joint processing costs are $200,000.

Price
per
Further
Numbe Pound Price after
Processing
Type of r of at Processing
Costs
Powder Pounds Split- Further
Off
Cosmetic
200,000 $10 $150,000 $11.50
Powder
Medicated
400,000 $8 $60,000 $8.40
Powder
Baby
50,000 $5 $80,000 $5.50
Powder

Required:

(a) Determine which products should be sold at split-off and which should be processed further.
(b) Geneva Powder is approached by the Regis Department Store chain. Regis would like Geneva
Powder to process regular powder into a special powder for its cosmetics department. At what price per
pound would Geneva Powder be economically indifferent between selling the powder at the split-off
point and processing it further for Regis?  
 

(a)

Net
 
Revenue Revenue Revenue
Processin
Product at Split- to Process After
g Costs
Off Further Processin
g
Cosmetic $2,000,00 $2,300,00 $2,150,00
$150,000
Powder 0 0 0
Medicate $3,200,00 $3,360,00 $3,300,00
$60,000
d Powder 0 0 0
Baby
$250,000 $275,000 $80,000 $195,000
Powder

The company should process the Cosmetic and Medicated Powders further, since the net revenue after
further processing exceeds the revenue at split-off. Baby Powder should be sold at the split-off point.
(b) Geneva Powder would be economically indifferent at a selling price of $6.60 for the regular
powder. There is currently an advantage of $55,000 for the company to sell Baby Powder at the split-off
point ($250,000 - 195,000). With 50,000 pounds of powder produced, the additional selling price per

11-212
Copyright © 2017 McGraw-Hill Education. All rights reserved. No reproduction or distribution without the prior written consent of
McGraw-Hill Education.
pound needed to generate the same net revenues by processing further is $1.10 ($55,000/50,000). The
current selling price after further processing is $5.50. The selling price to be economically indifferent
would be $5.50 + $1.10 = $6.60.

  Sell at Split-Off Process Further


50,000 50,000 ×
Revenues $250,000 $330,000
× $5 $6.60
Processing
         80,000
costs
Net
      $250,000
Revenues
 
AACSB: Analytical Thinking
AICPA: FN Measurement
Blooms: Analyze
Difficulty: 2 Medium
Gradable: manual
Learning Objective: 11-09 Explain how cost data are used in the sell-or-process-further decision.
Topic: Deciding Whether to Sell Goods Now or Process Them Further
 
133. Indicate whether the following costs would be treated as joint-product costs or costs incurred after the
split-off point. Use J for joint product costs and S for costs incurred after the split-off point.
 
Cost of planting, growing and picking pineapples in a pineapple
(a)                     
factory
(b) Costs of processing apples at a cider mill                      
(c) Costs of processing pineapples into juice and sliced pineapples                      
(d) Depreciation on oil rigs for an oil producer                      
Costs of running a fishing boat used to catch varieties of fish,
(e)                      
lobsters, etc.
(f) Labor costs, of “shucking” clams to produce clam chowder                      
Costs of chopping onions to be used in spaghetti sauce and soup in a
(g)                      
food manufacturer
Cost of processing rejected meat parts into hot dogs in a meat
(h)                      
processing plant
Cost of processing wood and sawdust into particle board in a
(i)                      
sawmill
(j) Ingredients and packaging added to batches of spaghetti in (g) above __________
(k) Costs of refining gasoline in (d) above                      
(l) Processing of pulp into paperboard in a paper manufacturer                      
(m) Utility costs of processing timber for a lumber manufacturer                      
 
 

(a) J, (b) S, (c) J, (d) J, (e) J, (f) S, (g) J, (h) S, (i) S, (j) S, (k) S, (l) S, (m) J

 
AACSB: Reflective Thinking
AICPA: FN Decision Making

11-213
Copyright © 2017 McGraw-Hill Education. All rights reserved. No reproduction or distribution without the prior written consent of
McGraw-Hill Education.
Blooms: Remember
Difficulty: 1 Easy
Gradable: manual
Learning Objective: 11-09 Explain how cost data are used in the sell-or-process-further decision.
Topic: Deciding Whether to Sell Goods Now or Process Them Further
 

11-214
Copyright © 2017 McGraw-Hill Education. All rights reserved. No reproduction or distribution without the prior written consent of
McGraw-Hill Education.
11-215
Copyright © 2017 McGraw-Hill Education. All rights reserved. No reproduction or distribution without the prior written consent of
McGraw-Hill Education.
134. The Macon Industries started the production of K1 (its main product) and S2 (its by-product) on January
2, 2016. During 2016, 7,500 units of K1 and 1,500 units of S2 were produced. In 2016, 6,000 units of
K1 and 1,000 units of S2 were sold at $57.00 and $1.10 per unit, respectively. Production was halted at
the end of 2016 and the inventory was sold in 2017 at the normal selling prices. The joint production
costs were $240,000 and are entirely avoidable. The separable costs to produce K1 were $2.60 per unit
and to produce S2 were $0.45 per unit. Operating expenses were $60,000 in 2016 and $12,000 in 2017.

Required:

a. Prepare an income statement for 2016 and 2017 assuming the "other revenue" method of accounting
for by-products is used.
b. Prepare an income statement for 2016 and 2017, assuming the "cost reduction" method of accounting
for by-products is used.  
 

a.     2016 2017
  Sales (57 × 6,000)  $342,000  
    (57 × 1,500)    $85,500
Cost of [($240,000/7,500
  goods ) × 6,000] +   207,600  
sold $15,600
[($240,000/7,500
    ) × 1,500] +     51,900
$3,900
Gross
         134,400        33,600
Margin
Operating
           60,000        12,000
expenses
Other ($1.10 - 0.45) ×
    650  
income 1,000
($1.10 - 0.45) ×
            325
500
Net
     $75,050  $21,925
income
b
    2016 2017
.
  Sales (57 × 6,000)  $342,000  
    (57 × 1,500)    $85,500
Cost of [($240,000 -
  goods 975)/7,500 ×       206,820  
sold 6,000] + $15,600
[($240,000 -
    975)/7,500 ×         51,705
1,500] + $3,900
  Gross        135,180       33,795

11-216
Copyright © 2017 McGraw-Hill Education. All rights reserved. No reproduction or distribution without the prior written consent of
McGraw-Hill Education.
Margin
Operating
       60,000   12,000
expenses
Net
     $75,180  $21,795
income
 
AACSB: Analytical Thinking
AICPA: FN Measurement
Blooms: Analyze
Difficulty: 3 Hard
Gradable: manual
Learning Objective: 11-10 Account for by-products.
Topic: Deciding What to Do with By-Products
 
135. What is the difference between an intermediate cost center and a final cost center?  
 

Any cost center whose costs are charged to other departments in the organization is called an
intermediate cost center, whereas a final cost center is one whose costs are not allocated to another cost
center.

 
AACSB: Analytical Thinking
AICPA: FN Measurement
Blooms: Remember
Difficulty: 2 Medium
Gradable: manual
Learning Objective: 11-01 Explain why service costs are allocated.
Topic: Methods of Allocating Service Department Costs
 
136. Describe the difference between the direct method of service department allocation, the step method,
and the reciprocal method. 
 

The direct method allocates the service department costs only to the producing or operating
departments. The step method allocates service department costs to other service departments as well as
the producing departments, but once a service department has been allocated, no further allocations are
made back to that department. The step method recognizes only some of the services provided to a
department. The reciprocal method recognizes all of the services provided to a department.

 
AACSB: Analytical Thinking
AICPA: FN Measurement
Blooms: Remember
Difficulty: 2 Medium
Gradable: manual
Learning Objective: 11-02 Allocate service department costs using the direct method.
Learning Objective: 11-03 Allocate service department costs using the step method.
Learning Objective: 11-04 Allocate service department costs using the reciprocal method.
Topic: Methods of Allocating Service Department Costs
 

11-217
Copyright © 2017 McGraw-Hill Education. All rights reserved. No reproduction or distribution without the prior written consent of
McGraw-Hill Education.
137. Why does the sequence in which service departments are allocated make a difference when using the
step method but not when using the reciprocal method? 
 

The sequence makes a difference when using the step method because once a service department's costs
have been allocated, no costs are allocated back to that department. The departments that are among the
first to be allocated will not be charged for service department costs of departments that are later in the
sequence. The sequence does not matter when using the reciprocal method because the costs are
allocated simultaneously.

 
AACSB: Analytical Thinking
AICPA: FN Measurement
Blooms: Apply
Difficulty: 2 Medium
Gradable: manual
Learning Objective: 11-03 Allocate service department costs using the step method.
Learning Objective: 11-04 Allocate service department costs using the reciprocal method.
Topic: Methods of Allocating Service Department Costs
 
138. Which of the three service department allocation methods should be used for decision making? Explain
your reasoning. 
 

The only method that is appropriate to use for decision making is the reciprocal method. The reciprocal
method attempts to determine all of the costs of a service department. The step method includes only
some of the services from other service departments; the direct method ignores all of the services from
other service departments.

 
AACSB: Analytical Thinking
AICPA: FN Decision Making
Blooms: Apply
Difficulty: 2 Medium
Gradable: manual
Learning Objective: 11-05 Use the reciprocal method approach for outsourcing decisions.
Topic: Methods of Allocating Service Department Costs
 
139. What are some of the reasons that joint costs are allocated? 
 

Cost allocations are often used to determine departmental or division costs for evaluating executive
performance and compensation. Manufacturing companies must allocate joint costs to determine the
inventory value of the products that result from the joint process. Companies subject to rate regulation
may need to use these allocations when determining the regulated rates and negotiating contracts.

 
AACSB: Analytical Thinking
AICPA: FN Decision Making
Blooms: Apply

11-218
Copyright © 2017 McGraw-Hill Education. All rights reserved. No reproduction or distribution without the prior written consent of
McGraw-Hill Education.
Difficulty: 2 Medium
Gradable: manual
Learning Objective: 11-06 Explain why joint costs are allocated.
Topic: Methods of Allocating Service Department Costs
 
140. Explain the difference between the net realizable value method for joint cost allocation and the netback
(or workback) method. 
 

The net realizable value method is used when products are salable at the split-off point. The netback,
also called workback or estimated net realizable value, method is used when some or all of the products
must be go through additional processing before they are salable. In this case, an estimate of the net
realizable value is computed by taking the final sales value minus the additional processing costs.

 
AACSB: Analytical Thinking
AICPA: FN Measurement
Blooms: Remember
Difficulty: 2 Medium
Gradable: manual
Learning Objective: 11-07 Allocate joint costs using the net realizable value method.
Topic: Joint Cost Allocation Methods
 
141. In a sell-or-process-further decision (a) what are the relevant data to be considered and (b) what is the
decision process associated with the split-off point?  
 

(a) As an application of the differential analysis approach, the relevant data to be considered are (1) the
additional revenue after further processing and (2) the additional costs of processing further.
(b) We can summarize the sell-or-process-further decision as:
 

Sell at split-off if:


Sales value after processing, less
Sales value at split-off
additional processing costs
>
Process further if:
Sales value after processing, less
Sales value at split-off
additional processing costs
<
 
AACSB: Analytical Thinking
AICPA: FN Measurement
Blooms: Remember
Difficulty: 2 Medium
Gradable: manual
Learning Objective: 11-09 Explain how cost data are used in the sell-or-process-further decision.
Topic: Deciding Whether to Sell Goods Now or Process Them Further
 

11-219
Copyright © 2017 McGraw-Hill Education. All rights reserved. No reproduction or distribution without the prior written consent of
McGraw-Hill Education.
142. Describe two methods of accounting for by-products. What effects do these methods have on the
allocation of the joint cost to the main products?  
 

One method is to assign a value to the by-products equal to the net realizable value of the by-product.
This method reduces the joint cost by the amount assigned to the by-products. The second method is to
assign no value to the by-products, but to treat the proceeds of the sale of the by-product as other
revenue. In this method there is no reduction in the joint cost.

 
AACSB: Analytical Thinking
AICPA: FN Measurement
Blooms: Remember
Difficulty: 2 Medium
Gradable: manual
Learning Objective: 11-10 Account for by-products.
Topic: Deciding What to Do with By-Products
 

11-220
Copyright © 2017 McGraw-Hill Education. All rights reserved. No reproduction or distribution without the prior written consent of
McGraw-Hill Education.
11-221
Copyright © 2017 McGraw-Hill Education. All rights reserved. No reproduction or distribution without the prior written consent of
McGraw-Hill Education.
143. Boswell Consulting has two service departments: S1 and S2 and three production departments: P1, P2,
and P3. Data for a recent month follow:
 
  S1 S2 P1 P2 P3
Service
Provide   0.10 0.20 0.40 0.30
d to:
  0.30   0.20 0.40 0.10
$200,00 $100,00 $600,00 $800,00 $1,000,00
Costs
0 0 0 0 0

Required:

(a) Determine the allocations to the production departments when the reciprocal method is used.
(b) Briefly describe why the reciprocal method is theoretically preferable to other methods of
allocation.  
 

(a)
 

Equations: S1 = $200,000 + .30S2


  S2 = $100.000 + .10 S1

S1 = $200,000 + . S2 = $100,00 + .
30(100,000 + .10S1) 10(200,000 + .30S2)
S1 = $200,000 + 30,000 + . S2 = $100,000 + 20,000 + .
03(S1) 03(S2)
.97S1 = 230,000 .97S2 = $120,000
S1 = $237,113 S2 = $123,711

Allocati
S1 S2 P1 P2 P3
on
Cost
before $200,00 $100,00 $600,00 $800,00 $1,000,00
allocatio 0 0 0 0 0
n:
Allocate -
23,711 47,423 94,845 71,134
S1: 237,113
($37,11 $123,71 $647,42 $894,84 $1,071,13
Subtotal
3) 1 3 5 4
Allocate -
 37,113   24,742   49,485      12,371
S2: 123,711
Balance       $0        $0 $672,16 $944,33 $1,083,50
after 5 0 5

11-222
Copyright © 2017 McGraw-Hill Education. All rights reserved. No reproduction or distribution without the prior written consent of
McGraw-Hill Education.
allocatio
n

(b) While there is no difference in the total amount of cost that would be allocated, the difference lies in
the treatment of inter-service departmental costs. The direct method ignores any service provided from
one service department to another; allocations begin from the service department that has provided the
greatest proportion of its services to other departments, or that services the greatest number of other
service departments. The step-method makes a partial recognition, but does not allocate back to service
departments on higher "steps." The reciprocal method fully recognizes inter-service department
activities.
 
AACSB: Reflective Thinking
AICPA: FN Measurement
Blooms: Analyze
Difficulty: 3 Hard
Gradable: manual
Learning Objective: 11-04 Allocate service department costs using the reciprocal method.
Topic: Methods of Allocating Service Department Costs
 

11-223
Copyright © 2017 McGraw-Hill Education. All rights reserved. No reproduction or distribution without the prior written consent of
McGraw-Hill Education.
11-224
Copyright © 2017 McGraw-Hill Education. All rights reserved. No reproduction or distribution without the prior written consent of
McGraw-Hill Education.
144. Morgan and Regis Consultants is a large, international consulting organization. The company provides
consulting services in the computer and Internet areas. The company also has several divisions that
provide manufacturing of various computer parts. The company has five divisions which are all profit
centers. Each division includes allocated corporate costs in its annual budget. The budget for the coming
year for the Building and Grounds Service department is $6,000,000. Included in this budget is the
maintenance of all corporate buildings, depreciation, cleaning, insurance, and all other facility-related
maintenance costs. The company uses a weighted method of allocating facility costs based on the type
of space maintained by each division. Space ranges from manufacturing warehouses, which are least
expensive to maintain, to computer mainframe space, which requires specialized temperature controls,
air conditioning, and maintenance. The company has decided to use a weighting system assigning the
following relative weights to each type of space: 1 for warehouse, 3 for office, and 5 for computer
space. Below, find data relating to the five divisions and the square footage of each type of space.
Currently, Division 5, the Internet consulting division, is the largest in sales volume and profits for the
company, which has been growing at the rate of 20% per year, while divisions 3 and 4 have been
struggling due to declining margins on technology products.
 
    Type of space  
  Office Computer Warehouse Total
Weighting 3 5 1  
Division 1 10000 0 0 10000
Division 2 31000 10000 5000 46000
Division 3 15000 12000 32000 59000
Division 4 15000 10000 50000 75000
Division 5 30000 30000 0 60000
Total 101000 62000 87000 250000

Required:

(a) As director of corporate budgeting, you are required to send to each division its facility allocation
for the coming year. Prepare a schedule showing how the budget of $6,000,000 will be allocated to each
division.
(b) Describe potential motivational problems brought on by these allocations. (Based on an actual
company)  
 

(a) First, determine the number of weighted square feet for each category.
 

    Type of space  
  Office Computer Warehouse Total
Weighting 3 5 1  
Division 1 30000 0 0 30000
Division 2 93000 50000 5000 148000
Division 3 45000 60000 32000 137000

11-225
Copyright © 2017 McGraw-Hill Education. All rights reserved. No reproduction or distribution without the prior written consent of
McGraw-Hill Education.
Division 4 45000 50000 50000 145000
Division 5 90000 150000         0 240000
Total 303000 310000 87000 700000

Allocation rate $6,000,000 = $8.57 per weighted


= 700,000 square foot

Then, allocate according to the allocation rate per weighted square foot:
 
Warehous
  Office Computer Total
e
Divisio
$257,143 $0 $0 $257,143
n1
Divisio
$797,143 $428,571 $42,857 $1,268,571
n2
Divisio
$385,714 $514,286 $274,286 $1,174,286
n3
Divisio
$385,714 $428,572 $428,571 $1,242,857
n4
Divisio
   $771,429 $1,285,714            $0 $2,057,143
n5
Total $2,597,143 $2,657,143 $745,714 $6,000,000

(b) The manager of division 5 may find the allocations unfair. Because the division uses more computer
and office space, but no warehouse space, its allocation is 62% more than the next highest allocation
and eight times more than Division 1. The manager may argue that the Division is being penalized for
its rapid growth, while the manufacturing divisions receive a much lower allocation, because most of
their space is warehouse space. All divisions, except Division 1, may complain about that division's low
allocation. Managers receiving higher allocations may argue that computer space is not 5 times more
expensive than warehouse space. This problem underscores the arbitrary nature of allocations and the
motivational issues that may arise from them.
 
AACSB: Reflective Thinking
AICPA: FN Measurement
Blooms: Analyze
Difficulty: 2 Medium
Gradable: manual
Learning Objective: 11-01 Explain why service costs are allocated.
Topic: Methods of Allocating Service Department Costs
 

11-226
Copyright © 2017 McGraw-Hill Education. All rights reserved. No reproduction or distribution without the prior written consent of
McGraw-Hill Education.
11-227
Copyright © 2017 McGraw-Hill Education. All rights reserved. No reproduction or distribution without the prior written consent of
McGraw-Hill Education.
145. Castro Corporation has one service department and three producing departments. The budget for the
following year allocates the service department costs to the producing departments based on the number
of employees in each department. Currently, the budget for the service department is $2,400,000 and the
number of employees in each department is as follows:

Department 1: 100
Department 2: 50
Department 3: 150

During the year, due to sudden expanded growth, Department 2 has to add 50 new employees; however
the service department costs have not increased due to budget constraints.

Required:

(a) What were the expected service department allocations at the beginning of the year to each
production department?
(b) What will be the actual allocations based on the number of employees each department has at year
end?
(c) Comment on the reasonableness of the situation. What are the potential causes of any problems
created by this allocation method?  
 

(a) Based on the budget, the allocations would be:


 

          Budget        
$2,400,000 = $8,000 per
Number of
     300 employee
employees

Department 1: 100 × $8,000 = $800,000


Department 2: 50 × $8,000 = $400,000
Department 3: 150 × $8,000 = $1,200,000
      Total $2,400,000

Based on the actual number of employees at year end, the allocations would be:

          Budget        
$2,400,000
Number of = $6,857.14
     350
employees

Department 1: 100 × $6,857.14 $685,714


Department 2: 100 × $6,857.14 $685,714
Department 3: 150 × $6,857.14 $1,028,572
      Total $2,400,000

11-228
Copyright © 2017 McGraw-Hill Education. All rights reserved. No reproduction or distribution without the prior written consent of
McGraw-Hill Education.
(c) The managers of Departments 1 and 3 both have a budget reduction, even though they took no
actions. The manager of Department 2 has a 71 percent increase in allocated cost with a 100 percent
increase in the number of employees. The managers of Departments 1 and 3 both benefit from the
actions of the manager in Department 2, even though they took no action. Furthermore, Department 2 is
being penalized for strong growth, which may cause a morale problem with Department 2's manager.
This problem could be created because the cost allocation base is inappropriate. If the number of
employees were an appropriate allocation base, one would expect the costs in the service department to
increase with such a large increase in the base.
 
AACSB: Reflective Thinking
AICPA: FN Measurement
Blooms: Analyze
Difficulty: 2 Medium
Gradable: manual
Learning Objective: 11-01 Explain why service costs are allocated.
Topic: Methods of Allocating Service Department Costs
 

11-229
Copyright © 2017 McGraw-Hill Education. All rights reserved. No reproduction or distribution without the prior written consent of
McGraw-Hill Education.
146. Liberty Credit Checks produces two styles of credit reports: personal and corporate. The difference
between the two is the amount of background information and data collection required. The corporate
report uses more skilled personnel because additional checking and data are required. The relevant
figures for the year just completed follow: Total support service costs to be allocated are $3,200,000.
 
Allocation base Individual Corporate
Data purchased $40,000 $80,000
Research hours 24,000 30,000
Interview hours 1,000 10,000
Number of reports 16,000 3,000

Required:

(a) Which method would be preferred by each manager? Which method would be least preferred?
(b) Provide arguments that each manager would make for his/her preferred method. How would each
manager argue against his/her least preferred method?  
 

(a) This question underscores the arbitrary nature of cost allocation and underscores the fact that no
method will please all of the recipients of allocated costs. In this case, the manager of the individual
department would most favor interview hours, where his/her total percentage of the allocation is only
9%. This manager would least favor number of reports, where the department would receive a total of
84% of the allocation. The manager of the corporate department would most favor number of reports,
with a 16% total allocation and least favor interview hours, where over 90% of the cost would be
allocated to the corporate department.
(b) Arguments for:
Number of reports: The corporate manager would argue that the final output represents the cost effort
of each department. Since corporate provided far fewer total reports, that department would receive a
relatively small allocation.
Interview hours: The individual manager would argue that interview hours are the most valuable source
of information and that they reflect an appropriate use of his/her department resources.
Arguments Against:
Number of reports: The individual manager would argue that just because there are more individual
customers generating the reports, the number of reports does not reflect the work done and cost incurred
by each department. The number of reports reflects great productivity and the department should not be
penalized for being productive.
Interview hours: The corporate manager would argue that while his/her department does more
interviewing, that is not a fair reflection of the resources consumed, since the number of interviews is
relatively small, but the hours put into them are extensive. The manager would argue that other methods
of gathering information more appropriately reflect the resource use of the Department.

 
AACSB: Reflective Thinking
AICPA: FN Measurement
Blooms: Analyze
Difficulty: 2 Medium
Gradable: manual
Learning Objective: 11-01 Explain why service costs are allocated.

11-230
Copyright © 2017 McGraw-Hill Education. All rights reserved. No reproduction or distribution without the prior written consent of
McGraw-Hill Education.
Topic: Methods of Allocating Service Department Costs
 

11-231
Copyright © 2017 McGraw-Hill Education. All rights reserved. No reproduction or distribution without the prior written consent of
McGraw-Hill Education.
11-232
Copyright © 2017 McGraw-Hill Education. All rights reserved. No reproduction or distribution without the prior written consent of
McGraw-Hill Education.
147. Portofino Manufacturing Corporation manufactures three products in a joint process. Additional
information is as follows:
 
  J K L Total
Units
16,000 4,000 2,000 22,000
produced
Sales value
$300,000 $100,000 $20,000 $420,000
at split-off
Addition
costs if
  Processed $48,000 $20,000 $6,000 $74,000
further
Sales value if
processed
$340,000 $160,000 $40,000 $540,000
   Further
Joint costs       $120,000

Required:

(a) Allocate the joint costs to the three products using the net realizable value method.
(b) Determine which products should be sold at split-off and which products should be processed
further.  
 

  J K L Total
Sales
$340,000 $160,000 $40,000 $540,000
revenues
Less
additional
processing
48,000 20,000 6,000 74,000
costs to point
of
marketability
a) NRV at
$292,000 $140,000 $34,000 $466,000
split-off
Percent 62.66% 30.04% 7.3% 100.00%
Allocation $75,192 $36,048 $8,760 $120,000
b)        
    J K L
Sales value at
  $300,000 $100,000 $20,000
split-off
Sales value if
processed   $340,00 $160,00
$40,000
   Further 0 0
Processing   $48,000 $20,000 $6,000

11-233
Copyright © 2017 McGraw-Hill Education. All rights reserved. No reproduction or distribution without the prior written consent of
McGraw-Hill Education.
costs
NRV of
  $292,000 $140,000 $34,000
processing
Net benefit of
further  
($8,000) $40,000 $14,000
   processing

Only Products K and L should be processed further. Product J should be sold at split-off.
 
AACSB: Reflective Thinking
AICPA: FN Decision Making
AICPA: FN Measurement
Blooms: Analyze
Difficulty: 2 Medium
Gradable: manual
Learning Objective: 11-09 Explain how cost data are used in the sell-or-process-further decision.
Topic: Deciding Whether to Sell Goods Now or Process Them Further
 

11-234
Copyright © 2017 McGraw-Hill Education. All rights reserved. No reproduction or distribution without the prior written consent of
McGraw-Hill Education.
148. Dawson Corporation produces a product called Blocker, which gives rise to a by-product called Spotter.
The only costs associated with Spotter are additional processing costs of $4 for each unit. Dawson
accounts for Spotter's sales first by deducting its separable costs from its sales and then by deducting
this net amount from the cost of sales of Blocker. This year, 9,600 units of Spotter were produced. They
were all sold for $8 each. Company operating expenses were $250,000 for the year. Sales revenue and
cost of goods sold for Blocker were $1,600,000 and $800,000 respectively. (CPA adapted)

Required:

(a) Calculate the company's gross margin under the current accounting method.
(b) Assume the company changes its accounting method and accounts for the by-product's net
realizable value as "other revenue." Calculate the gross margin under the new method.
(c) Under what circumstances would method a or b be preferred?  
 

(a)
 

Net Realizable Value of Spotter [($8 - 4) ×


$38,400
9,600]
Cost of Goods Sold of Blocker ($800,000 -
761,600
38,400)
Gross Margin of Blocker ($1,600,000 -
$838,400
$761,600)

Sales $1,600,000
Other Revenue         38,400
Total Revenue $1,638,400
Cost of Goods Sold      800,000
Gross Margin    $838,400

(c) While either method will result in the same gross margin for the company, treating the net realizable
value as other revenue is the simpler method. Technically, the matching principle is violated if all
products are not sold in the accounting period. However, because the effect is normally considered
immaterial, it is used by many companies. Deducting the net realizable value from the cost of the main
products can be more complicated if all main products are not sold. If all the main products produced in
the period have not been sold, the net realizable value of by-products should be prorated to the main
product inventories and cost of goods sold.
 
AACSB: Reflective Thinking
AICPA: FN Decision Making
AICPA: FN Measurement
Blooms: Analyze
Difficulty: 2 Medium
Gradable: manual
Learning Objective: 11-07 Allocate joint costs using the net realizable value method.
Topic: Joint Cost Allocation Methods

11-235
Copyright © 2017 McGraw-Hill Education. All rights reserved. No reproduction or distribution without the prior written consent of
McGraw-Hill Education.
 

11-236
Copyright © 2017 McGraw-Hill Education. All rights reserved. No reproduction or distribution without the prior written consent of
McGraw-Hill Education.
11-237
Copyright © 2017 McGraw-Hill Education. All rights reserved. No reproduction or distribution without the prior written consent of
McGraw-Hill Education.
149. Bartoff Foods produces three supplemental food products simultaneously through a refining process
costing $186,000. The joint products, Bulkup and Bodybuilder, have a final selling price of $8 per
pound and $10 per pound, respectively, after additional processing costs of $2 per pound for each
product incurred after the split-off point. Quicksnack, a by-product, is sold at the split-off point for $6
per pound. The production of Bulkup results in 20,000 pounds with a caloric value of 6,000 calories per
pound. The production of Bodybuilder, which is very high in carbohydrates, has a caloric value of
12,500 calories per pound. 10,000 pounds of Bodybuilder are produced. Quicksnack has a caloric value
of 2,000 calories a pound and 2,000 pounds are produced. (CMA adapted)

Required:

(a) Allocate the joint product costs using the net-realizable-value method, assuming that Quicksnack is
accounted for as a by-product, with its net realizable value deducted from the cost of the main products.
(b) Allocate the joint product costs using the physical measures method, assuming that Quicksnack is
accounted for as a by-product, with its sales revenue accounted for as "other revenue."
(c) Compute Bartoff Food's gross margin under requirements a and b.  
 

(a)
 

  Bulkup Bodybuilder Quicksnack


Pounds
20,000 10,000 2,000
produced
Selling price
$8 $10 $6
per pound
Total sales $160,000 $100,000 $12,000
Calories per
6,000 12,500 2,000
pound
Total
120,000,000 125,000,000 4,000,000
calories
Processing
costs per $2 $2  
pound

Assumption 1: By-product revenue deducted from the costs of the main products.
Costs allocated according to net realizable value.
Total costs to be allocated = $186,000 - $12,000 = $174,000.
 
Net
Separab Perce Allocat
Product Sales realizab
le nt ed
le
  Value Costs Value   Cost
$40,000
$160,00 $120,00 $104,40
Bulkup (20,000 60%
0 0 0
× $2)

11-238
Copyright © 2017 McGraw-Hill Education. All rights reserved. No reproduction or distribution without the prior written consent of
McGraw-Hill Education.
  20,000
Bodybuild $100,00  $80,00    69,60
(10,000 40%
er 0 0 0
× $2)
$260,00 $200,00 $174,00
  $60,000 100%
0 0 0

(b) By-product revenue recorded as "other revenue."


 
Product Calories Percent Allocated
  Cost    
Bulkup 120,000,000 48.98% $91,103
Bodybuilder 125,000,000  51.02%  $94,897
Total 245,000,000 100.00% $186,000

Note: Some students may assume that physical measures method uses total pounds produced.
(c) Regardless of the method chosen, the gross margin will remain the same.
Requirement a:
 
Sales revenue: Main products $260,000
Cost of goods sold:  
Main products: $186,000 in joint-costs plus  
        60,000 in separable costs - 12,000  
          NRV of by-products   234,000
Gross margin $26,000
Requirement b:  
Sales revenue:  
    Main products $260,000
    By-product     12,000
Total revenue: $272,000
Cost of main products sold: $186,000 +  
  60,000 of separable costs    246,000
  Gross margin $26,000
 
AACSB: Reflective Thinking
AICPA: FN Decision Making
AICPA: FN Measurement
Blooms: Analyze
Difficulty: 2 Medium
Gradable: manual
Learning Objective: 11-07 Allocate joint costs using the net realizable value method.
Learning Objective: 11-08 Allocate joint costs using the physical quantities method.
Learning Objective: 11-10 Account for by-products.
Topic: Joint Cost Allocation Methods
 

11-239
Copyright © 2017 McGraw-Hill Education. All rights reserved. No reproduction or distribution without the prior written consent of
McGraw-Hill Education.
150. Timberland Corporation produces three products from a joint process: One-X, Two-Y, and Three-Z.
Each product can be processed further and sold for more. Data on the processes are as follows:

Product One-X Two-Y Three-Z Total


Units
16,000 8,000 4,000 28,000
produced
Joint costs $60,000 (a) (b) $120,000
Sales value at
(c) (d) $30,000 $200,000
split-off
Additional
Processing $14,000 $10,000 $6,000 $30,000
Costs
Sales value if
Processed $140,000 $60,000 $40,000 $240,000
Further

The amount of joint costs for One-X is the amount that has been allocated.

Required:

Determine the values for the lettered spaces. (CPA adapted)  


 

The ratio of sales value at split-off to total sales value at split-off equals the joint-cost ratio.
First, allocate costs to Three-Z: Sales value at split-off divided by total sales value at split-off ×
$120,000 joint costs. $30,000/$200,000 × $120,000 = $18,000 b = $18,000
Joint costs allocated to Two-Y: $120,000 total; $60,000 to One-X: $18,000 to Three-Z = $42,000
a = $42,000
One-X sales value at split-off: (c/$200,000) × $120,000 = $60,000
c = $100,000
Two-Y sales value at split-off: (d/$200,000) × 120,000 = $42,000
d = 35% of $200,000
d = $70,000

 
AACSB: Reflective Thinking
AICPA: FN Decision Making
AICPA: FN Measurement
Blooms: Analyze
Difficulty: 3 Hard
Gradable: manual
Learning Objective: 11-09 Explain how cost data are used in the sell-or-process-further decision.
Topic: Deciding Whether to Sell Goods Now or Process Them Further
 

11-240
Copyright © 2017 McGraw-Hill Education. All rights reserved. No reproduction or distribution without the prior written consent of
McGraw-Hill Education.
11-241
Copyright © 2017 McGraw-Hill Education. All rights reserved. No reproduction or distribution without the prior written consent of
McGraw-Hill Education.
151. Penny's Pineapples is a pineapple grower. After cultivating, fertilizing, growing, and picking
pineapples, the company sells whole pineapples to food processors. The company is considering adding
a processing line where sliced pineapples and pineapple juice, along with a "mash" used for animal feed
will be the final products. Projected information about the costs follows:
 
Final
Units Separable
Product selling
produced costs
price
  Per unit    
Sliced $3.00 per
900,000 cans $600,000
pineapple can
Pineapple 400,000 $1.75 per
$150,000
juice bottles bottle
500,000 $.50 per
Mash $120,000
pounds pound

Joint product costs of cultivating, fertilizing and picking pineapples total $1,000,000.

Required:

(a) Determine the amount of separable costs allocated to each product using the net realizable value
method.
(b) Determine the final cost per unit for each product.
(c) Determine the gross margin for each product.
(d) A fertilizer manufacturer approaches Penny Martin, the President of the company, and asks to buy
the rinds and other excess materials currently used to produce Mash. He would be willing to pay $0.30
per pound for these materials. What advice would you give Penny?  
 

(a)
 

Final Separa Net Costs


Sales ble Realizab Perce Allocate
Produc Value Costs le Value nt d
t
Sliced
$2,700,0 $600,00 $2,100,0 75.54
Pineapp $755,400
00 0 00 %
le
Pineapp $150,00 19.78
$700,000 $550,000 $197,800
le Juice 0 %
 $250,00 $120,00   $130,00   4.68    $46,80
Mash
0 0 0 % 0
$3,650,0 $870,00 $2,780,0 100.00 $1,000,0
Total
00 0 00 % 00

11-242
Copyright © 2017 McGraw-Hill Education. All rights reserved. No reproduction or distribution without the prior written consent of
McGraw-Hill Education.
(b)
 
Numb
Separab Per
  Allocated Total er of
le unit
units
Sliced
$1,355,40 900,00 $1,5
Pineappl $755,400 $600,000
0 0 1
e
Pineappl 400,00 $0.8
$197,800 $150,000 $347,800
e Juice 0 7
500,00 $0.3
Mash    $46,800 $120,000  $166,800
0 3
$1,000,00 $1,870,00
  $870,000    
0 0

(c)
 
Selling Cost Gross
Product Percentage
price per unit profit
Sliced
$3.00 $1.51 $1.49 49.67%
Pineapple
Pineapple
$1.75 $0.87 $0.88 50.29%
Juice
 Mash $0.50 $0.33 $0.17 34.00%

(d) Penny should sell the product to the fertilizer manufacturer. Company profits will increase $20,000:
 
Revenue at split-off
(500,000 × $0.30) $150,000
Revenue to process  
Further (500,000 × $0.50) $250,000
Processing costs $120,000
Net profit of processing $130,000
Net benefit to sell at  
   split-off $20,000
 
AACSB: Reflective Thinking
AICPA: FN Decision Making
AICPA: FN Measurement
Blooms: Analyze
Difficulty: 3 Hard
Gradable: manual
Learning Objective: 11-07 Allocate joint costs using the net realizable value method.
Topic: Joint Cost Allocation Methods
 

11-243
Copyright © 2017 McGraw-Hill Education. All rights reserved. No reproduction or distribution without the prior written consent of
McGraw-Hill Education.
11-244
Copyright © 2017 McGraw-Hill Education. All rights reserved. No reproduction or distribution without the prior written consent of
McGraw-Hill Education.
152. Fantasy Manufacturing produces three products in a joint operation. Information regarding the products
appears below:

  Item 1 Item 2 Item 3 Total


Units
20,000 25,000 10,000 55,000
Produced
Sales Value at
$150,000 $50,000 $20,000 $220,000
Split-off
Additional
costs if
$10,000 $30,000 $5,000 $45,000
Processed
further
Sales Value if
Processed $170,000 $90,000 $28,000 $288,000
Further
Joint Costs       $100,000

Required:

(a) Allocate the joint costs using the relative sales value at split-off method.
(b) Allocate the joint costs using the constant gross margin percentage method.  
 

(a)

  Item 1 Item 2 Item 3 Total


Units
20,000 25,000 10,000 55,000
Produced
Sales Value at
$150,000 $50,000 $20,000 $220,000
Split-off
Percentage 68.2% 22.7% 9.1 % 100%
Joint Costs
$68,200 $22,700 $9,100 $100,000
Allocated

(b)
 
  Item 1 Item 2 Item 3 Total
$170,00 $90,00 $28,00 $288,00
Sales Value
0 0 0 0
Joint Costs       100,000
Additional
         45,000
Processing Costs
$143,00
Gross Margin      
0

11-245
Copyright © 2017 McGraw-Hill Education. All rights reserved. No reproduction or distribution without the prior written consent of
McGraw-Hill Education.
Gross margin
Percentage
     
49.65%
($143,000/$288,00
0)

  Item 1 Item 2 Item 3 Total


Sales Value $170,000 $90,000 $28,000 $288,000
Joint Costs 75,591 15,313 9,096 100,000
Additional
Processing 10,000 30,000 5,000    45,000
Costs
Gross Margin $84,409 $44,687 $13,904 $143,000
Gross margin
49.6523 49.6523 49.6523 49.6523
Percentage
 
AACSB: Reflective Thinking
AICPA: FN Measurement
Blooms: Analyze
Difficulty: 3 Hard
Gradable: manual
Learning Objective: 11-09 Explain how cost data are used in the sell-or-process-further decision.
Topic: Deciding Whether to Sell Goods Now or Process Them Further
 

11-246
Copyright © 2017 McGraw-Hill Education. All rights reserved. No reproduction or distribution without the prior written consent of
McGraw-Hill Education.

You might also like